File:  [Local Repository] / db / baza / okmar15.txt
Revision 1.22: download - view: text, annotated - select for diffs - revision graph
Thu Aug 23 11:27:05 2018 UTC (5 years, 9 months ago) by rubashkin
Branches: MAIN
CVS tags: HEAD
*** empty log message ***

Чемпионат:
Окский марафон - 2015 (Пущино)

URL:
/znatoki/boris/reports/201501Pushchino.html

Дата:
31-Jan-2015

Инфо:
В качестве первых двух туров марафона использовался синхрон "Азовский
бриз", а в качестве последних полутора - синхрон "Кубок 359".

Тур:
3 тур. "ЛУК" (Архангельск)

Вопрос 1:
Внимание, в вопросе есть замены.
   В тексте песни группы "Ундервуд" (куда же без нее на марафоне!)
строчка "Новый год от ИКСА до ИГРЕКА" рифмуется со строкой "Новый год,
приготовьтесь, будет круто!". Что мы заменили на ИКС и ИГРЕК, если они
отличаются друг от друга всего одной буквой?

Ответ:
Салат и салют.

Комментарий:
   Новый год от салата до салюта.
   Новый год, дед Мороз не так уж стар.
   Новый год, приготовьтесь, будет круто.
   А пока-пока-пока, а пока-пока-пока,
   Все ждут двенадцатый удар.
   Всё как в жизни - начинаем салатом, заканчиваем салютом. Команда
"ЛУК" приветствует вас в новом 2015 году и как бы намекает:
приготовьтесь, будет действительно круто.

Источник:
   1. http://morepesen.ru/34810-undervud-novogodnij-car.html
   2. http://www.youtube.com/watch?v=J0ganCpggo0

Автор:
Иван Нефёдов

Вопрос 2:
В романе Александра Иличевского театральный режиссер Штейн держит дома
человеческий череп, который появился у него в доме в 1931 году. Штейн
уверяет мальчишек, пришедших в гости, что это череп известного человека,
приводя в доказательство подлинности отсутствие... Чего?

Ответ:
Носа.

Комментарий:
Штейн неохотно отвечал мальчишкам: "Череп Гоголя. Вот здесь не хватает
носа. Где нос? Нос пошел гулять". В 1931 году перезахоранивали Николая
Васильевича.

Источник:
Александр Иличевский. Перс. - М.: АСТ: Астрель, 2010. - С. 325.

Автор:
Иван Нефёдов

Вопрос 3:
[Ведущему: кавычки в вопросе не озвучивать.]
   Прослушайте список, в котором две буквы в одном из слов лишние:
"Динозавры", "Хищники", "Черный налим", "Грызуны", "Пингвины", "Крючки".
Такая российская классификация была предложена в одном из выпусков
журнала "Forbes" за 2012 год. Шесть видов чего были обозначены таким
образом?

Ответ:
Коррупция.

Зачет:
Взяточничество, денежные вымогательства, взятки, способы "дать на лапу".

Комментарий:
Не налим, а нал. Причем черный. Если вдруг интересно: "Динозавры" -
коррупционные практики, постепенно уходящие в прошлое. "Хищники" -
коррупционные схемы, основанные на вымогании денег у бизнеса органами
государственного контроля. "Черный нал" - понятно. "Грызуны" -
использование ресурсов компании для личного обогащения. "Пингвины" -
случаи использования в коррупционных целях дружбы и прочих личных
отношений. "Крючки" - использование в своих интересах рычагов
воздействия на сотрудников, партнеров и ответственных лиц через сбор
конфиденциальной и компрометирующей информации.

Источник:
http://www.forbes.ru/svoi-biznes/biznes-i-vlast/80739-6-vidov-vzyatok-v-rossii

Автор:
Иван Нефёдов

Вопрос 4:
В апреле 1998 года двое американцев решили своеобразным образом
поддержать налоговую реформу. Они выплыли на середину портовой гавани
одного из прибрежных городов. Достали старый сундук, положили в него
налоговый кодекс и выбросили всё это за борт. Мы не просим вас назвать
город, рядом с которым происходило это мероприятие. Ответьте, какое
короткое слово было написано на сундуке.

Ответ:
Чай.

Зачет:
Tea.

Комментарий:
Своего рода символ Бостонского чаепития 1773 года.

Источник:
   1. http://hpn.asu.edu/archives/Apr98/0204.html
   2. http://www.historynewsnetwork.org/article/94858
   3. http://en.wikipedia.org/wiki/Boston_Tea_Party

Автор:
Иван Нефёдов, Александр Усачёв

Вопрос 5:
В сентябре 2011 года сайт smithsonian.com [смитсОниан точка ком]
составил список самых главных произведений мировой литературы, в который
среди прочих вошли поэма Гомера "Маргит", роман Хемингуэя о Первой
мировой, пьеса Шекспира "Карденио", роман Джейн Остин "Сэндитон" и роман
Германа Мелвилла "Остров Креста". Напишите слово, которое мы пропустили
в предыдущем предложении.

Ответ:
Утерянных.

Зачет:
Потерянных, утраченных и т.п. по смыслу.

Комментарий:
Это были книги, которые мы уже никогда не сможем прочесть, - список
самых главных утерянных произведений мировой литературы.

Источник:
http://www.smithsonianmag.com/arts-culture/The-Top-10-Books-Lost-to-Time.html

Автор:
Иван Нефёдов

Вопрос 6:
[Ведущему: перечислять фразы с соответствующими паузами после них.]
   Статья Александра Харламова от 20 ноября 2012 года была посвящена
известному футбольному клубу и состояла из нескольких абзацев,
начинавшихся следующими фразами: "львиная доля", "уже довольно давно",
"честолюбивый тренер", "ершистость", "сумасшедшая победная серия", "как
ни крути", "убедительная игра". Назовите фамилию тренера, успешно
возглавлявшего клуб в тот момент.

Ответ:
Луческу.

Комментарий:
По первым буквам абзацев.

Источник:
http://www.sports.ru/football/144829723.html

Автор:
Иван Нефёдов

Вопрос 7:
У группы "Сатана печет блины" есть песня с не совсем приличным
названием. В ней поется о нерадивом хозяине, который не заботился о
своих животных, и за это его не простят ни еж, ни пес, ни енот, но вот
ОНА, может быть, простит. Назовите ЕЕ.

Ответ:
Утка.

Зачет:
Уточка.

Источник:
"Сатана печет блины" - "Простит утка".

Автор:
Денис Овчинников

Вопрос 8:
Прослушайте цитату из книги Терри Пратчетта "Пятый элефант": "Господин
Ваймс, это мой фамильный топор. Он принадлежит моей семье почти
девятьсот лет, понятно? Конечно, пару раз меняли лезвие. Несколько раз -
топорище. Меняли конструкцию металлических частей, подновляли узоры...
но разве от этого он перестал быть девятисотлетним фамильным топором?".
Этот топор упоминается в статье Википедии, посвященной проблеме,
появившейся еще во времена античности. Напишите название этой статьи.

Ответ:
Корабль Тесея.

Зачет:
Парадокс Тесея.

Комментарий:
Согласно греческому мифу, пересказанному Плутархом, корабль, на котором
Тесей вернулся с Крита в Афины, долгое время хранился афинянами. При
починке в нем постепенно заменяли доски, до тех пор пока среди философов
не возник спор, тот ли это еще корабль или уже другой, новый?

Источник:
   1. http://ru.wikipedia.org/wiki/Корабль_Тесея
   2. Терри Пратчетт. Пятый элефант.
http://www.e-reading.link/bookreader.php/46326/Pratchett_24_Pyatyii_elefant.html

Автор:
Денис Овчинников

Вопрос 9:
(pic: 20150134.jpg)
   Внимание, в вопросе есть замена.
   В одном из эпизодов мультсериала "Южный парк" утверждается, что
каждый раз, когда Морган Фриман что-то объясняет, он получает АЛЬФУ. У
главного героя мультфильма 1971 года, которого несправедливо обвиняли в
убийстве, тоже были АЛЬФЫ. Назовите АЛЬФУ.

Ответ:
Веснушка.

Комментарий:
Мультфильм 1971 года - "Рыжий, рыжий, конопатый".

Источник:
   1. Мультсериал "Южный парк", s16e14.
   2. http://ru.wikipedia.org/wiki/Рыжий,_рыжий,_конопатый

Автор:
Денис Овчинников

Вопрос 10:
(pic: 20150135.jpg)
   Перед вами предмет, принадлежащий одному из персонажей мультсериала
"Папский городок". Назовите профессию этого персонажа двумя словами.

Ответ:
Швейцарский гвардеец.

Комментарий:
Швейцарская гвардия - один из видов вооруженных сил Ватикана, используют
алебарды как парадно-церемониальное оружие. В мультфильме лезвие
заменено на швейцарский нож.

Источник:
http://ru.wikipedia.org/wiki/Швейцарская_гвардия

Автор:
Денис Овчинников

Вопрос 11:
(pic: 20150136.jpg)
   Художница Джессика Хэйджи с помощью такой схемы отметила то, к каким
последствиям в повседневной жизни приводят все возможные парные
сочетания ИХ. Например, AC для нее - "проституция", CF - "реклама", FD -
"социальное пособие" DB - "суббота", BG - "толстяк в микроплавках", GF -
"второе место", FE - "язвительность". Назовите ИХ тремя словами, два из
которых начинаются на одну и ту же букву.

Ответ:
Семь смертных грехов.

Комментарий:
Примеров, кстати, семь. Вот грехи: A - блуд, B - чревоугодие, C -
алчность, D - леность, E - гнев, F - зависть, G - гордыня. А вот список
сочетаний:
   AB - съедобные трусы,
   AC - проституция,
   AD - быстро перепихнуться,
   AE - домашнее насилие,
   AF - адюльтер,
   AG - брак по расчету,
   BC - последний пончик,
   BD - суббота,
   BE - булимия,
   BF - быстрый метаболизм,
   BG - толстяк в микроплавках,
   CD - аферы "разбогатей сейчас",
   CE - грабеж,
   CF - реклама,
   CG - символы статуса,
   DE - пассивная агрессия,
   DF - социальное пособие,
   DG - халтурщики,
   EF - язвительность,
   EG - бокс,
   FG - второе место.

Источник:
http://esquire.ru/thisisindexed

Автор:
Иван Нефёдов

Вопрос 12:
   <раздатка>
   Статья в журнале "Esquire" о том, как фотограф Мэттью Таун на
протяжении нескольких лет снимает портреты учеников летной школы на юге
Англии, называется "Как я провел этим летом".
   </раздатка>
   [Ведущему: читать "лётной".]
   Статья в журнале "Esquire" о том, как фотограф Мэттью Таун на
протяжении нескольких лет снимает портреты учеников летной школы на юге
Англии, называется "Как я провел этим летом". В предложенном вам тексте
мы дважды пропустили две буквы. Напишите пропавшие буквы в любом
порядке.

Ответ:
А и Б.

Зачет:
Б и А.

Комментарий:
А упало, Б пропало. Школа балетная.

Источник:
http://esquire.ru/photo/ballet-boys

Автор:
Иван Нефёдов

Вопрос 13:
Внимание, в вопросе есть замены.
   Действие американской компьютерной игры "Singularity" происходит в
секретном советском научном комплексе. В одной из лабораторий можно
увидеть Периодический ИКС из ИГРЕКИ. Напишите слова, которые заменены на
ИКС и ИГРЕК.

Ответ:
Стол, элемент.

Комментарий:
(pic: 20150137.jpg)
   Разработчики пытались воссоздать атмосферу советской лаборатории и
использовали русский язык. "Периодический стол из элементы" - неудачный
перевод "Periodical table of elements".

Источник:
   1. http://sadtranslations.livejournal.com/1151914.html
   2. http://ru.wikipedia.org/wiki/Singularity_(игра)

Автор:
Денис Овчинников

Вопрос 14:
В романе ХалЕда ХоссейнИ пожилая мать в разговоре со своим сыном
упоминает Джеймса, Джорджа, Роберта, Джона. Саму себя она связывает с Лу
и задает логичный вопрос: "Как мужчинам удалось монополизировать ИХ?".
Догадавшись о чем идет речь, назовите любую из фамилий упомянутых
мужчин.

Ответ:
Гериг.

Зачет:
Паркинсон, Хангтингтон, Грейвз, Даун.

Комментарий:
"Джеймс Паркинсон, Джордж Хантингтон, Роберт Грейвз, Джон Даун. А теперь
еще и этот мой Лу Гериг. Как мужчинам удалось монополизировать и
названия болезней?".

Источник:
Халед Хоссейни. И эхо летит по горам. - М.: Фантом Пресс, 2013. - С.
370.

Автор:
Иван Нефёдов

Вопрос 15:
Название статьи в русскоязычном журнале "Esquire" [эсквайр] с портретами
молодых американок, одетых подобно своим куклам, на слух совпадает с
названием государства, которое по площади территории занимает двадцатое
место в мире и на гербе которого присутствуют волны. Воспроизведите
название этой статьи из двух слов.

Ответ:
"Республика чад".

Комментарий:
США по форме правления, так же как и Чад, является президентской
республикой. Волны - символ озера Чад.

Источник:
   1. http://esquire.ru/photo/american-girls
   2. http://ru.wikipedia.org/wiki/Чад

Автор:
Иван Нефёдов

Вопрос 16:
В компьютерной игре "The binding of Isaac" [зэ бАйндинг оф Айзэк]
("Жертвоприношение Исаака") некоторые артефакты являются реально
существующими предметами, например, Библия, карты Таро или Спиритическая
доска. На одном из таких артефактов нарисована буква "А", и при его
использовании появляется несколько бомб. Назовите этот артефакт тремя
словами.

Ответ:
Поваренная книга анархиста.

Комментарий:
Отсылка к "Поваренной книге анархиста" - пособию по использованию
обычных исходных материалов, веществ и предметов для изготовления в
домашних условиях наркотических веществ, оружия, взрывных устройств,
ядов; буква "А" в круге - символ анархии.

Источник:
   1. http://ru.bindingofisaac.wikia.com/wiki/Предметы
   2. http://ru.wikipedia.org/wiki/Поваренная_книга_анархиста

Автор:
Денис Овчинников

Вопрос 17:
Бернард Шоу сравнил ЕЕ со спасательным кругом, который "бросают пловцу,
когда тот уже благополучно достиг берега". Совпадение или нет, но,
начиная с 2010 года, Малую ЕЕ можно "наблюдать" в декабре в одной из
северных республик РФ. Назовите ЕЕ двумя словами.

Ответ:
Нобелевская премия.

Комментарий:
Компания "Нобель Ойл" учредила в республике Коми "Малую Нобелевскую
премию" для поддержки учащихся и учителей. Процедура награждения
Нобелевской премией происходит ежегодно, 10 декабря, в столицах двух
стран - Швеции и Норвегии. Ну и как всем известно, Бернард Шоу получил
свою Нобелевку по литературе в 1925 году.

Источник:
   1. http://ru.wikipedia.org/wiki/Нобелевская_премия
   2. http://www.komi-nobel.ru

Автор:
Иван Нефёдов

Вопрос 18:
В романе Александра Иличевского подросток мечтательно описывает тюльпан
Эйхлера - персидский экземпляр, с ослепительно черным зеркалом, за
луковичку которого в Голландии перед восстанием гёзов могли расплатиться
каретой с лошадьми. Назовите цвет тюльпана абсолютно точно.

Ответ:
Аленький.

Зачет:
Алый. Незачет: Другие оттенки красного и сам красный.

Комментарий:
Вот такая аллюзия на известную сказку, богатый купец в которой
отправлялся "по торговым делам за море". А младшая дочь, видимо, была не
только самая любимая, но и самая прозорливая. В сказке "Красавица и
чудовище" в качестве "аленького цветка" была роза.

Источник:
Александр Иличевский. Перс. - М.: АСТ: Астрель, 2010. - С. 324.

Автор:
Иван Нефёдов

Вопрос 19:
В компьютерной игре "The binding of Isaac" [зэ бАйндинг оф Айзэк]
("Жертвоприношение Исаака") некоторые артефакты являются реально
существующими предметами, например, Библия, карты Таро или Спиритическая
доска. Один из артефактов представляет собой ключ, похожий на ИКС, и
дает возможность открыть почти все двери. Назовите ИКС.

Ответ:
Скелет.

Комментарий:
Словосочетание "Skeleton Key" (он же "passingkey") обозначает
специальный ключ, сделанный таким образом, чтобы открывать все замки в
определенном месте, например, в гостиницах. Данное словосочетание можно
перевести дословно как "ключ-скелет".

Источник:
   1. http://ru.bindingofisaac.wikia.com/wiki/Предметы
   2. http://en.wikipedia.org/wiki/Skeleton_key

Автор:
Денис Овчинников

Вопрос 20:
[Ведущему: кавычки в вопросе не озвучивать.]
   Отмечая новаторский подход режиссера фильма "Добро пожаловать в
Зомбиленд", Кирилл Андреев парадоксально замечает, что кинолента придает
АЛЬФУ "ИКСАМ". "АЛЬФА" - произведение средневекового автора, а "ИКСЫ"
появились на следующий год после выхода упомянутого фильма. Какие четыре
слова мы заменили на АЛЬФУ ИКСОВ?

Ответ:
Новая жизнь ходячих мертвецов.

Комментарий:
Новая жизнь (итал. La Vita Nuova) - сборник произведений, написанных
Данте Алигьери в 1283-1293 гг. Фильм "Добро пожаловать в зомбиленд"
вышел на экраны в октябре 2009 года и первоначально планировался как
пилотный эпизод сериала. Через год в октябре 2010 года состоялась
премьера сериала "Ходячие мертвецы". "Новаторский" - подсказка.

Источник:
   1. http://www.film.ru/articles/pod-druzhnoe-rychanie
   2. http://www.kinopoisk.ru/film/427122/
   3. http://en.wikipedia.org/wiki/The_Walking_Dead_(TV_series)

Автор:
Александр Усачёв

Вопрос 21:
Комментируя успешный пуск ракеты с атомной субмарины Б-138 "Обнинск"
проекта 671РТМК, посетитель сайта topwar.ru [топвар точка ру] упомянул
ПЕРВУЮ и ВТОРОГО. Догадавшись, кто такая ПЕРВАЯ, ответьте, кем был
ВТОРОЙ в произведении 1884 года.

Ответ:
Идеалистом.

Комментарий:
Кодовое название подлодок проекта 671РТМК - "Щука". Выражение - "щука в
воде, чтобы карась не дремал". В 1884 году был опубликован рассказ
Салтыкова-Щедрина "Карась-идеалист".

Источник:
   1. http://topwar.ru/64259-s-apl-obninsk-proizveden-uspeshnyy-pusk-krylatoy-rakety-iz-podvodnogo-polozheniya.html
   2. http://dic.academic.ru/dic.nsf/michelson_old/65/
   3. http://az.lib.ru/s/saltykow_m_e/text_0480.shtml

Автор:
Александр Усачёв

Вопрос 22:
Тайфун "Хагупит", обрушившийся на Филиппины в начале декабря 2014 года,
принес множество разрушений и стал серьезным испытанием для всех жителей
островного государства. Особенно страшными выдались первые удары стихии,
пришедшиеся на города в восточной части страны. Многие филиппинцы
получили тяжелые ранения, были и погибшие. Одно из значений слова
"хагупит" на тагальском языке - ОН. Назовите ЕГО.

Ответ:
Кнут.

Зачет:
Хлыст. Незачет: Плеть (слово женского рода).

Комментарий:
"Обрушившийся", "удары", "испытание" - прямые отсылки к наказанию или
пытке кнутом. Тяжелые ранения и последующая смерть были обычным делом
при этом. Также некоторые считают, что первые удары были наиболее
страшными и тяжелыми, так как кнут был еще твердым и не размокшим.

Источник:
   1. http://ru.euronews.com/2014/12/07/more-than-a-million-take-shelter-as-typhoon-hagupit-hits-the-philippines
   2. http://www.rg.ru/2014/12/08/taifun-site.html

Автор:
Александр Усачёв

Вопрос 23:
Автор статьи на сайте "РБК", комментируя большое количество косвенных
аргументов в пользу возвращения на пост главного тренера "Челси" летом
2013 года Жозе Моуринью, имеющего действующий контракт с мадридским
"Реалом", называет эту ситуацию "защитой Чубакки". В одном из разделов
Википедии "защита Чубакки" стоит между "Святой Русью" и "серым
кардиналом". Что мы заменили словами "защита Чубакки"?

Ответ:
Секрет Полишинеля.

Комментарий:
Это всё фразеологизмы, а ситуация с Моуринью превратилась к концу мая в
"секрет Полишинеля".

Источник:
   1. http://sport.rbc.ru/football/newsline/17/05/2013/391651.shtml
   2. http://ru.wikipedia.org/wiki/Секрет_Полишинеля

Автор:
Иван Нефёдов

Вопрос 24:
Наблюдая, как часть солнца начинает пропадать во время затмения, а рядом
сидящая девушка переписывается на онлайн-форуме, герой романа ХалЕда
ХоссейнИ сравнивает солнце с НИМ. Назовите ЕГО двумя словами.

Ответ:
Логотип "Apple".

Комментарий:
"От солнца с северного бока уже откушен ломтик, и теперь оно похоже на
логотип "Эппл" на ноутбуке Талии, - она время от времени рапортует на
какой-то онлайн-форум".

Источник:
Халед Хоссейни. И эхо летит по горам. - М.: Фантом Пресс, 2013. - С.
370.

Автор:
Иван Нефёдов

Тур:
4 тур. "IDDQD-UNR.I.P" (Москва - Раменское)

Вопрос 1:
Говоря об одной из созданных им телепередач, Сергей Супонев упомянул
"самый трудный возраст", а также продолжительный хронометраж каждого из
ее выпусков. Напишите название этой передачи, которое, по одной из
версий, появилось благодаря популярной головоломке.

Ответ:
"Марафон-15".

Комментарий:
По словам Супонева, "15 - самый трудный возраст", а 15 сюжетов в каждой
из программ - "это марафон". "Марафон-15" - телепередача для подростков,
выходившая в эфир с 1989 по 1998 год, каждый выпуск которой состоял из
15 отдельных сюжетов. Пятнашки, или "Игра в 15", - головоломка,
связанная с выстраиванием последовательности из пятнадцати костяшек.

Источник:
   1. http://ru.wikipedia.org/wiki/Марафон-15
   2. http://ru.wikipedia.org/wiki/Пятнашки

Автор:
Виктор Бойков (Раменское)

Вопрос 2:
В пародийном фильме один герой обещает раздавить другого, а затем
обвести мелком. Какие два слова мы пропустили в предыдущем предложении?

Ответ:
Против тараканов.

Зачет:
От тараканов; для тараканов; как таракана.

Комментарий:
В ходе расследований криминалисты для большей наглядности обводят трупы
мелом. "Мелок от тараканов" - популярное средство борьбы с данным видом
насекомых, на которых, кстати, намекает прозвучавшее в вопросе слово
"раздавить". Авторская группа надеется, что по итогам тура ее не
постигнет участь одного из героев данного вопроса.

Источник:
   1. http://www.abur.tv/news/prigodnick/508204-prosto-gavrilo-i-kubok-mogila-2013.html
(2:07:25)
   2. http://www.sanobrabotka.com/blog/2013/01/16/preparaty-protiv-tarakanov/

Автор:
Виктор Бойков (Раменское)

Вопрос 3:
Юрий Денисов, говоря об известных исследователях, приходит к выводу, что
их впечатления от объекта соперничества в сравнении оказались ТАКИМИ.
Автор этого вопроса считает, что ТАКИМИ можно назвать впечатления
каждого из этих соперников по отдельности. Какими ТАКИМИ?

Ответ:
Полярными.

Комментарий:
Речь идет о полярных исследователях Руале Амундсене и Роберте Скотте,
соперничавших за звание первооткрывателя Южного полюса. Амундсен
описывает Антарктиду восторженно, Скотт же называет ее "ужасным местом".

Источник:
   1. http://db.chgk.info/question/spbvuz07.9/11
   2. http://slovari.yandex.ru/полярный/правописание/

Автор:
Евгений Кравченко (Москва)

Вопрос 4:
Внимание, в вопросе есть замены.
   Рекламный слоган российского туристического агентства гласит:
   Скуку, сырость, холод не КОРИ -
   На солнечном КОРИНФЕ отдыхай!
   Какое слово мы заменили словом "КОРИНФ"?

Ответ:
ХайнАнь.

Комментарий:
В оригинале стихотворный слоган звучит следующим образом:
   Скуку, сырость, холод не хай -
   На солнечном ХайнАне отдыхай!
   ХайнАнь - популярный курортный остров у побережья КНР. Согласно
"Толковому словарю живого великорусского языка" В.И. Даля, одним из
синонимов глагола "хаять" является глагол "корить".

Источник:
   1. ЛОАВ.
   2. http://dic.academic.ru/dic.nsf/enc2p/372394/

Автор:
Виктор Бойков (Раменское)

Вопрос 5:
(pic: 20150138.jpg)
   На картине Александра Бейдемана Эскулап негодующе машет рукой на
профессоров медицины, суетящихся вокруг пациента. Назовите фамилию
человека, которого Бейдеман изобразил на картине справа, в образе
печального старца.

Ответ:
Ганеман.

Комментарий:
Полотно Бейдемана, написанное в 1857 году, носит название "Гомеопатия,
взирающая на ужасы аллопатии". Немецкий ученый Христиан Фридрих Самуэль
Ганеман - основоположник гомеопатии (вида альтернативной медицины,
предполагающей использование сильно разведенных препаратов). Аллопатия -
термин, предложенный сторонниками гомеопатии и других альтернативных
направлений медицины для обозначения фармакотерапии и других методов
общепринятой медицины.

Источник:
   1. http://omeopathy.livejournal.com/19743.html
   2. http://ru.wikipedia.org/wiki/Гомеопатия
   3. http://ru.wikipedia.org/wiki/Аллопатия

Автор:
Евгений Кравченко (Москва)

Вопрос 6:
[Ведущему: максимально четко прочитать словосочетание "ФЕДЕРАЛЬНЫЙ КУБОК
ЯРМАРОК".]
   В мае 2013 года на сайте Lenta.ru [лента точка ру] вышла статья под
заголовком "ФЕДЕРАЛЬНЫЙ КУБОК ЯРМАРОК". Ответьте двумя словами, что мы
заменили на "ФЕДЕРАЛЬНЫЙ КУБОК ЯРМАРОК".

Ответ:
Бундеслига чемпионов.

Комментарий:
Статья была посвящена финалу футбольной Лиги чемпионов, в котором
встречались мюнхенская "Бавария" и дортмундская "Боруссия". Оба этих
клуба представляют высшую лигу Чемпионата Германии по футболу, которая
официально именуется Бундеслигой. Корень "бундес-" переводится с
немецкого языка как "федеральный". Кубок ярмарок - название футбольного
турнира - предшественника Кубка УЕФА и Лиги Европы.

Источник:
   1. http://lenta.ru/articles/2013/05/02/league/
   2. http://ru.wikipedia.org/wiki/Кубок_ярмарок

Автор:
Виктор Бойков (Раменское)

Вопрос 7:
Внимание, в вопросе есть замена.
   Статья новостного агентства "РИА Новости" о событиях на Корейском
полуострове была озаглавлена "КНДР ОТЖАЛА на себя санкции". Герой
произведения 1965 года рассказывает, как ОТЖАЛ, и упоминает два
числительных. Какие?

Ответ:
10000, 500.

Зачет:
Десять тысяч, пятьсот; десять тыщ, пятьсот.

Комментарий:
Заголовок новостного материала о взрыве в рамках северокорейских ядерных
испытаний звучал так: "КНДР РВАНУЛА на себя санкции". Герой написанной в
1965 году Владимиром Высоцким "Песни про конькобежца на короткие
дистанции..." признавался: "... Я на десять тыщ РВАНУЛ, как на пятьсот,
- и спекся". Жим, как и рывок, - одно из упражнений тяжелой атлетики.

Источник:
   1. http://ria.ru/world/20130212/922452250.html
   2. http://www.kulichki.com/vv/pesni/desyat-tysyach-i-vsego.html
   3. http://ru.wikipedia.org/wiki/Жим_над_головой

Автор:
Виктор Бойков (Раменское)

Вопрос 8:
В англоязычных странах так изначально называли изображения на
геральдических щитах и церковные хоругви. В современной жизни мы чаще
всего сталкиваемся с НИМИ в Интернете или на спортивных мероприятиях, да
и просто на улицах. Назовите ИХ.

Ответ:
Баннеры.

Источник:
   1. http://en.wikipedia.org/wiki/Banner
   2. http://ru.wikipedia.org/wiki/Баннер

Автор:
Евгений Кравченко (Москва)

Вопрос 9:
Журнал "Time" [тайм] отмечает, что некая шиитская традиция сыграла
ключевую роль в событиях 1978 года в Иране, когда волны протестов,
захлестнувшие страну, переросли в исламскую революцию. Какое число
"Time" [тайм] упоминает в связи с этой традицией?

Ответ:
40.

Комментарий:
По шиитской традиции, поминальные службы о погибшем тоже идут 40 дней.
Соответственно, волнения в Иране проходили с явным интервалом в 40 дней,
ширясь и ширясь за счет новых смертей. И вопрос девятый, что говорит еще
об одной традиционной поминальной дате.

Источник:
   1. http://www.time.com/time/world/article/0,8599,1906049,00.html
   2. http://ru.wikipedia.org/wiki/Исламская_революция_в_Иране

Автор:
Евгений Кравченко (Москва)

Вопрос 10:
Эмблема болгарской партии, представляющей интересы граждан - сторонников
европейского развития страны, представляет собой ряд звезд, опоясывающих
слово "ЩИТ". Что мы заменили словом "ЩИТ"?

Ответ:
ГЕРБ.

Комментарий:
На эмблеме партии "Граждане за европейское развитие Болгарии" звезды,
символизирующие Евросоюз, опоясывают аббревиатуру названия партии,
составляющую слово "ГЕРБ". В геральдике гербы часто имеют форму щита.

Источник:
http://www.gerb.bg/

Автор:
Сергей Филин (Красноармейск)

Вопрос 11:
В сатирической песне, посвященной коррупции в системе МВД, автор сетует,
что с "оборотнями в погонах" нельзя СДЕЛАТЬ ЭТО, т.к. взяточники ДЕЛАЮТ
ТО. Какие три слова мы заменили словами "СДЕЛАТЬ ЭТО", а какие три -
словами "ДЕЛАТЬ ТО"?

Ответ:
Поймать за руку, брать на лапу.

Зачет:
Ловить за руку, взять на лапу.

Комментарий:
"Поймать за руку" - устойчивое выражение, означающее "уличить кого-либо
или поймать с поличным". "Брать на лапу" - также устойчивое выражение,
означающее "получить взятку". Действительно, сложно поймать за руку
того, кто берет на лапу. Упоминание в вопросе "оборотней" намекает на
периодическое превращение рук в лапы и наоборот.

Источник:
   1. http://music.yandex.ru/#!/track/4230086/album/479578
   2. http://ru.wikipedia.org/wiki/Оборотень
   3. http://phraseology.academic.ru/6218/
   4. http://phrase_dictionary.academic.ru/65/

Автор:
Виктор Бойков (Раменское)

Вопрос 12:
Рассуждая о роли Твиттера и Фейсбука в волнениях на территории
североафриканских государств, Евгений Козловский переосмысливает
известные пушкинские строки. Напишите название стихотворения, из
которого они взяты.

Ответ:
"Утопленник".

Комментарий:
Twitter и Facebook - современные социальные сети. Говоря о погибших в
народных волнениях на территории Магриба, автор вспоминает фразу "Тятя,
тятя, наши сети притащили мертвеца" из стихотворения А.С. Пушкина
"Утопленник" (1825).

Источник:
   1. http://ria.ru/authors/20110301/340574046.html
   2. http://ru.wikisource.org/wiki/Утопленник_(Пушкин)

Автор:
Евгений Кравченко (Москва)

Вопрос 13:
Согласно современной шутке, "настоящая женщина должна родить сына,
воспитать мужа и построить МОНСТРА". Какое слово в предыдущем
предложении мы заменили другим?

Ответ:
Свекровь.

Комментарий:
Замена является отсылкой к русскоязычному варианту названия известной
голливудской комедии "Если свекровь - монстр".

Источник:
   1. http://www.inpearls.ru/comments/329985
   2. http://ru.wikipedia.org/wiki/Если_свекровь_%E2%80%94_монстр

Автор:
Виктор Бойков (Раменское)

Вопрос 14:
(pic: 20150139.jpg)
   Блиц.
   Перед вами шуточные валентинки американского художника Бена Клинга.
   1. Какие четыре буквы мы закрыли от вас на первой картинке?
   2. Какое слово мы закрыли от вас на второй картинке?
   3. Какое слово испанского происхождения мы закрыли от вас на третьей
картинке?

Ответ:
   1. Майн.
   2. Председатель.
   3. Эмбарго.

Зачет:
   1. Mein.
   2. Chairman.
   3. Embargo.

Комментарий:
Первая валентинка обыгрывает название известной книги Адольфа Гитлера
"Mein Kampf" ("Моя борьба"). Вторая посвящена первому председателю КНР и
ЦК Коммунистической партии Китая Мао Цзэдуну, также часто именуемому
"председателем Мао". Третья посвящена экс-лидеру Кубы Фиделю Кастро, чья
родина с 1961 года находится под торговым эмбарго со стороны США.

Источник:
   1. http://www.benkling.com/valentines
   2. http://ru.wikipedia.org/wiki/Моя_борьба
   3. http://ru.wikipedia.org/wiki/Мао_Цзэдун
   4. http://ru.wikipedia.org/wiki/Эмбарго

Автор:
Виктор Бойков (Раменское)

Вопрос 15:
В известном романе ОНИ "распустились в тумане, как кристаллики соли в
воде". А по мнению Линды Маккартни, если бы ИМИ оборудовали некоторые
сооружения, то весь мир отказался бы от мяса. Назовите ИХ двумя словами,
начинающимися на одну и ту же букву.

Ответ:
Стеклянные стены.

Комментарий:
В романе-антиутопии Евгения Замятина "Мы" элементом постоянного контроля
жизни граждан со стороны тоталитарного государства являются стеклянные
стены жилых домов. Линда Маккартни предлагала оборудовать стеклянными
стенами бойни.

Источник:
   1. http://az.lib.ru/z/zamjatin_e_i/text_0050.shtml
   2. http://ru.wikipedia.org/wiki/Маккартни,_Линда

Автор:
Евгений Кравченко (Москва)

Вопрос 16:
В материале на сайте sports.ru [спортс точка ру] журналист пожелал нашим
прыгунам-синхронистам чисто выполнить свою программу. Пожелания были
снабжены ремаркой: "Чтобы ПЕРВЫЙ ПРОПУСК зал, важно не ВТОРОЙ ПРОПУСК
бассейн". Заполните оба пропуска однокоренными словами.

Ответ:
Зарукоплескал, расплескать.

Зачет:
Любые подходящие по смыслу формы данных слов.

Комментарий:
Качество входа в воду в спортивных прыжках оценивается, в частности, по
отсутствую брызг.

Источник:
http://www.sports.ru/others/london2012/142333658.html

Автор:
Евгений Кравченко (Москва)

Вопрос 17:
Недалеко от бывшего здания фабрики по производству шляп в
Сан-Жуан-да-Мадейра расположен памятник "Черные ногти", поставленный в
честь работниц фабрики. Традиционный местный головной убор "карапУца"
имеет особенную форму, напоминающую об одном литературном персонаже.
Назовите этого персонажа.

Ответ:
Железный дровосек.

Комментарий:
У работниц фабрики из-за особенностей производства ногти с годами
приобретали черную окраску. Постепенно все жители Сан-Жуана получили в
Португалии прозвище "черные ногти". КарапУца внешне напоминает воронку -
считается, что ее удлиненный хвост позволяет стягивать головной убор, не
запачкав его грязными руками.

Источник:
   1. http://pt.wikipedia.org/wiki/S%C3%A3o_Jo%C3%A3o_da_Madeira
   2. http://www.moda-dic.ru/html/k/karapuca.html
   3. ЛОАВ, материалы экскурсионных рассказов.
   4. http://ru.wikipedia.org/wiki/Железный_Дровосек

Автор:
Евгений Кравченко (Москва)

Вопрос 18:
МоасИр БарбОза - выдающийся голкипер футбольного клуба "Васко да Гама" и
сборной Бразилии середины XX века. Однако именно его обвиняют в
поражении от уругвайцев на домашнем чемпионате мира. Статья о его судьбе
называется так же, как музыкальная композиция 1991 года. Воспроизведите
это название.

Ответ:
"[The] Unforgiven".

Зачет:
"Непрощенный".

Комментарий:
В Бразилии такое не прощают. Всю последующую жизнь Барбоза ощущал на
себе последствия злополучного второго гола в финале ЧМ-1950. "The
Unforgiven" - песня американской музыкальной группы "Metallica" с
вышедшего в 1991 году альбома "The Black Album".

Источник:
   1. http://en.wikipedia.org/wiki/The_Last_Save_of_Moacyr_Barbosa
   2. http://ru.wikipedia.org/wiki/Барбоза,_Моасир
   3. http://ru.wikipedia.org/wiki/The_Unforgiven

Автор:
Евгений Кравченко (Москва)

Вопрос 19:
(pic: 20150140.jpg)
   Перед вами иллюстрация к статье доктора физико-математических наук
Сергея Сухинина, демонстрирующая понятие волнового фронта. Иллюстрация
опровергает тезис, выдвинутый в известном анекдоте. На ней мы скрыли от
вас некий предмет. Какой?

Ответ:
Кирпич.

Комментарий:
Анекдот, соответственно, такой:
   Создали чукчи научный институт. Просят привезти им железнодорожный
состав кирпича. Прислали. Через некоторое время просят привезти еще.
Опять прислали. Потом еще. Заинтересовались в Москве: "Что же они там
такое строят?". Выслали комиссию проверить строительство. Приезжают.
Видят - стоят на берегу моря несколько человек и бросают в воду кирпичи.
"Что это вы такое делаете?" - спрашивают их. А чукчи и отвечают: "А вы
посмотрите, кирпичи-то квадратные, а круги от них по воде круглые!".
   Из рисунка видно, что волны, конечно же, не круглые.

Источник:
http://elementy.ru/lib/430289?context=1532471

Автор:
Евгений Кравченко (Москва)

Вопрос 20:
Статья газеты "Московский комсомолец" о компании "Де Бирс",
присматривающейся к некоторым российским месторождениям, носит название
"У Де Бирс [ПРОПУСК]. На чужие деньги". Заполните пропуск устойчивым
словосочетанием.

Ответ:
Глаз - алмаз.

Комментарий:
"Де Бирс" - мировой монополист алмазной промышленности.

Источник:
http://socarchive.narod.ru/digest/pr/archive/0313/20.html

Автор:
Виктор Бойков (Раменское)

Вопрос 21:
   <раздатка>
   Белые овечки, белые овечки на голубом холме,
   Когда ветер утихнет, вы смирно стоите,
   Когда ветер подует, уходите прочь.
   Белые овечки, белые овечки, куда же уходите вы?
   </раздатка>
   [Ведущему: сделать ярко выраженную паузу между словами "загаданное" и
"в песне".]
   Мы не спрашиваем, что загадано в розданном вам стихотворении
английской поэтессы Кристины Россетти. Ответьте, с чем сравнивалось
загаданное в песне на слова советского поэта и переводчика Леонида
ЯхнинА.

Ответ:
С [белыми] корабликами. Незачет: С [белогривыми] лошадками.

Комментарий:
Совпадение или нет, но песня "Белые кораблики" содержит следующие слова:
"Белые кораблики, белые кораблики // По небу плывут. // Белые кораблики,
белые кораблики // Дождики везут". Последние слова песни: "Белые
кораблики, белые кораблики - // Это облака". Слова "овца" и "корабль"
по-английски звучат почти одинаково.

Источник:
   1. http://www.poets.org/poetsorg/poem/clouds
   2. http://www.a-pesni.org/baby/belkorab.php

Автор:
Антон Тилипман (Москва)

Вопрос 22:
Добавив в лермонтовскую строчку притяжательное и указательное
местоимения, этот бард превратил ямб в амфибрахий. В результате
получилась песня, ставшая сквозной темой классической советской комедии.
Назовите фамилию барда.

Ответ:
Ким.

Комментарий:
При написании сквозной музыкальной темы фильма Марка Захарова
"Двенадцать стульев" (1975) бард Юлий Ким вставил в строку "Белеет парус
одинокий" из стихотворения М.Ю. Лермонтова местоимения "мой" и "такой".

Источник:
http://ru.wikipedia.org/wiki/12_стульев_(фильм,_1976)

Автор:
Евгений Кравченко (Москва)

Вопрос 23:
В этот день 2002 года в США было объявлено о самом крупном за всю
историю страны банкротстве. В тот же день восемь лет спустя в Аргентине
вступил в силу закон об однополых браках. В точности через год произошли
кровавые теракты в Норвегии. День приближения чего отмечается 22 июля?

Ответ:
[Число] Пи.

Зачет:
[Число] &pi;.

Комментарий:
Одной из дат, связанных с числом Пи, является 22 июля, называемое "Днем
приближения числа Пи" (англ. Pi Approximation Day). Дело в том, что в
европейском формате дат этот день записывается как 22/7, а значение
данной дроби является приближенным значением числа Пи с точностью до
двух знаков после запятой. А наш тур приближается к логическому
завершению.

Источник:
   1. http://ru.wikipedia.org/wiki/22_июля
   2. http://ru.wikipedia.org/wiki/Пи_(число)

Автор:
Антон Тилипман (Москва)

Вопрос 24:
Согласно шутке, представители некоторой профессии могут устроить КРАСНУЮ
ЖАРУ, если их не поздравить с этим праздником. Поэтому с одним из
последних таких праздников этих людей поздравляли особенно тепло. Что мы
заменили словами "КРАСНАЯ ЖАРА"?

Ответ:
Конец света.

Комментарий:
Широко известный благодаря масс-медиа потенциальный "конец света",
ожидавшийся 21 декабря 2012 года, по случайности совпал с днем
энергетика. "Красная жара" (1988) и "Конец света" (1999) - фильмы с
участием известного американского актера Арнольда Шварценеггера. На этом
наш тур завершается.

Источник:
   1. http://ru.wikipedia.org/wiki/День_энергетика
   2. http://ru.wikipedia.org/wiki/Конец_света_(2012)
   3. http://ru.wikipedia.org/wiki/Шварценеггер,_Арнольд

Автор:
Евгений Кравченко (Москва)

Тур:
5 тур. "Канатчикова дача" (Москва)

Вопрос 1:
Внимание, в вопросе есть замены.
   а где тут руль промолвил кедр
   деревня хмыкнула заря
   еще спроси а где тут вожжи
   еще ПРОПУСК скажи
   Догадавшись, какие слова мы заменили, заполните пропуск.

Ответ:
Поехали.

Комментарий:
Во время первого полета в космос позывным Гагарина был Кедр, Королёва -
Заря-1.

Источник:
   1. http://www.perashki.ru/piro/29313/
   2. http://www.cosmoworld.ru/spaceencyclopedia/gagarin/index.shtml?doc10.html

Вопрос 2:
В конце XIX века ОН совершил дерзкое убийство, всколыхнувшее Англию и
вызвавшее небывалый переполох в определенных кругах. Все хорошо знали
виновника, но его невозможно было наказать. Существует легенда, что сама
королева Виктория была настолько возмущена его поступком, что вынудила
вышеупомянутого джентльмена добровольно искупить свою вину. От вас же
требуется всего лишь назвать его имя - ведь это так просто!

Ответ:
Артур.

Комментарий:
Речь, конечно же, идет об Артуре Конан Дойле и об убийстве Шерлока
Холмса в схватке с профессором Мориарти.

Источник:
http://ru.wikipedia.org/wiki/Шерлок_Холмс

Вопрос 3:
В середине 40-х годов прошлого века в Америке были запатентованы
необычные шахматы с фигурами в виде танков, самолетов и прочего. В этих
шахматах пешка, доходя до последней линии, становилась АЛЬФОЙ. Назовите
АЛЬФУ двумя словами.

Ответ:
Атомная бомба.

Источник:
Е. Гижицкий. С шахматами через века и страны. - Warszawa: Sport I
Turystyka, 1970. - С. 75.

Вопрос 4:
В средневековой Японии гейша не имела права вступать в брак. Поэтому
вместо церемонии бракосочетания была принята другая процедура, во время
которой влюбленные обменивались ИКСАМИ вместо колец. Назовите комедию
1995 года, в которой ИКС появился в пентхаусе.

Ответ:
"Четыре комнаты".

Комментарий:
ИКС - отрубленный палец.

Источник:
   1. А.Н. Мещеряков. Книга японских символов. Книга японских
обыкновений. - М.: Наталис, 2004. - С. 510.
   2. http://ru.wikipedia.org/wiki/Четыре_комнаты

Вопрос 5:
Согласно одной кхмерской легенде, раньше ОНИ были огромного роста, даже
больше грифов, у них были огромные пасти, отличались они необыкновенной
прожорливостью и питались исключительно людьми. Чтобы не дрожать всю
жизнь в ожидании неминуемой смерти, люди бросали жребий, кому быть
съеденным ИМИ в следующую ночь. Назовите ИХ.

Ответ:
Комары.

Источник:
http://www.bibliotekar.ru/azia/71.htm

Вопрос 6:
"На берегу гладкого озера выросли кривые вербы", - говорят калмыки про
АЛЬФЫ. А одна группа "родственников" всерьез уверена, что с помощью АЛЬФ
можно летать. Назовите этих "родственников".

Ответ:
"Братья Гримм".

Комментарий:
АЛЬФЫ - ресницы.

Источник:
   1. http://ru.wikiquote.org/wiki/Калмыцкие_загадки
   2. http://megalyrics.ru/lyric/bratia-grimm/riesnitsy.htm

Вопрос 7:
Этот топоним любят заимствовать для названия чего-нибудь грандиозного.
Например, так называется один из популярных сервисов для автоматического
перевода. Чаще всего это название встречается в единственном числе,
однако интернет-словарь редких и забытых слов приводит определение для
множественного числа данного слова: ОНИ - это "извилины, кривые или
ломаные линии, вычурный узор". Почти наверняка вы хорошо ИХ помните
благодаря советской кинокомедии 1964 года. А где ОНИ были в этом фильме?

Ответ:
На голове.

Комментарий:
ОНИ - это вавилоны, а вы наверняка помните фразу из кинофильма "Добро
пожаловать, или Посторонним вход воспрещен!": "А Митрофанова вавилоны на
голове крутит!". Ну а программа, как вы наверняка знаете, носит название
"Babylon".

Источник:
   1. http://www.babylon.com/
   2. http://www.zabytye-slova.ru/vavilony/
   3. http://frazochka.ru/authors/6703.html

Вопрос 8:
(pic: 20150141.jpg)
   Поисково-прицельная система "Новелла-П-38", устанавливаемая при
модернизации самолетов Ил-38, получила среди авиаторов прозвище "АЛЬФА".
В статье Википедии, посвященной АЛЬФЕ, упоминаются республики Поволжья и
Крым. Назовите АЛЬФУ.

Ответ:
Тюбетейка.

Источник:
   1. http://www.sdelanounas.ru/blogs/54229/
   2. http://ru.wikipedia.org/wiki/Тюбетейка

Вопрос 9:
Во время президентства Джорджа Буша-старшего его советником по
национальной безопасности был генерал-лейтенант Брент Скоукрофт, который
отличался тем, что часто ДЕЛАЛ ЭТО. Буш даже учредил специальную премию
в его честь и сам лично выбирал победителей. В России, если верить
известному выражению, в среде "младших коллег" Скоукрофта как раз в
обычае ДЕЛАТЬ ЭТО, так как их работе это никак не мешает. Что мы
заменили на "ДЕЛАТЬ ЭТО"?

Ответ:
Спать на совещаниях.

Зачет:
Спать на работе.

Комментарий:
Премией Скоукрофта награждались те, кто уснул во время встречи с
президентом, причем тот оценивал кандидатов по трем показателям -
продолжительности и глубине сна, а также храпу. Известная русская
поговорка - "Солдат спит - служба идет".

Источник:
http://ru.wikipedia.org/wiki/Скоукрофт,_Брент

Вопрос 10:
   <раздатка>
   В 2009 году 67-летний Стивен Кинг был удостоен одной из двух высших
наград США для гражданских лиц - президентской медали свободы, которой
награждаются люди, "внесшие существенный вклад в безопасность и защиту
национальных интересов США, в поддержание мира во всём мире, а также в
общественную и культурную жизнь США и мира.
   </раздатка>
   Какие две буквы мы опустили в раздаче?

Ответ:
Хо.

Комментарий:
Стивену Кингу на данный момент всё еще 67 лет, а медаль получил Стивен
Хокинг.

Источник:
   1. http://ru.wikipedia.org/wiki/Хокинг,_Стивен_Уильям
   2. http://ru.wikipedia.org/wiki/Кинг,_Стивен

Вопрос 11:
Иван Дмитриевич Папанин в своем дневнике "Жизнь на льдине" писал: "От
лагеря до лебедки, на расстоянии одного километра, мы протянули веревку,
чтобы, в случае сильной пурги, можно было двигаться, держась за нее и не
рискуя заблудиться". Ответьте двумя словами, как назвали папанинцы "это
новое веревочное сооружение".

Ответ:
Троллейбусная линия.

Комментарий:
Номер вопроса напоминает троллейбусные штанги.

Источник:
И.Д. Папанин. Жизнь на льдине. Дневник. - 2-е издание. - М.:
Художественная литература, 1940. - С. 230.

Вопрос 12:
Бернард Шоу как-то сказал: "Я бы променял все картины с изображением
Христа на одну АЛЬФУ". По мнению физика Джона Джексона, такой АЛЬФОЙ
является ОНА. Назовите ЕЕ абсолютно точно.

Ответ:
Туринская плащаница.

Комментарий:
АЛЬФА - это фотография. А Джон Джексон возглавляет центр по изучению
Туринской плащаницы.

Источник:
   1. http://www.photoquotations.com/a/614/George+Bernard+Shaw
   2. http://www.youtube.com/watch?v=1c3rJKt8wjA

Вопрос 13:
В 1855 году в газетах писали об одном из экспонатов Парижской выставки:
ОН был впервые "показан публично, и при этом стало ясно, что бОльшую
часть того, что о нем говорилось, следует приписать фантазии и
легковерности широкой публики. Вместо ожидаемых гор ЕГО там оказалось
всего 12 маленьких брусочков, общим весом около килограмма, что,
конечно, немного для открытия, которое казалось способным перевернуть
мир". Назовите ЕГО.

Ответ:
Алюминий.

Комментарий:
Алюминий - 13-й элемент периодической системы, на что, надеемся,
намекает номер вопроса. И хотя алюминий впервые был получен в 1825 году,
только в 1854 году Анри Сент-Клер Девиль изобрел первый способ его
промышленного производства.

Источник:
   1. http://www.ng.ru/koncep/2008-02-28/9_element.html
   2. http://ru.wikipedia.org/wiki/Алюминий

Вопрос 14:
Фанаты общественного транспорта называют ИМ автобус ЛиАЗ-677. В 1993
году на аукционе Sotheby's ОН был продан за 68.500 долларов Ричарду
Гэрриотту. Назовите ЕГО сложносоставным словом.

Ответ:
Луноход.

Комментарий:
В 1993 году на аукционе Sotheby's, посвященном космической тематике,
"Луноход-2" был продан за 68.500 долларов Ричарду Гэрриотту,
американскому коллекционеру космических артефактов и сыну астронавта
NASA.

Источник:
   1. http://lurkmore.to/ЛиАЗ-677
   2. http://ru.wikipedia.org/wiki/Луноход-2

Вопрос 15:
Существует распространенное мнение, что спортсмены не отличаются высоким
интеллектом. Например, есть такой анекдот про известного итальянского
футболиста Франческо Тотти. В результате пожара в доме Тотти сгорела вся
его библиотека. Сам футболист так прокомментировал данный инцидент:
"Конечно же, мне очень жаль обе книжки! Тем более, одну из них я даже не
успел полностью прочитать...". В комментарии Тотти мы заменили одно
слово. Напишите его в первоначальном варианте.

Ответ:
Раскрасить.

Комментарий:
Это были даже не книжки, а детские раскраски.

Источник:
http://francescototti.narod.ru/jokebook.htm

Вопрос 16:
На соревнованиях "Бегущий город. Москва. 2011" участникам выдавалась
аудиозагадка - по "минусовке" требовалось угадать "Песенку об Арбате"
Булата Окуджавы. У некоторых участников это вызывало затруднения, и
судьи давали им совет из двух слов, совпадающий со слоганом социальной
рекламы. Воспроизведите этот слоган.

Ответ:
"Позвоните родителям".

Источник:
   1. Личный опыт автора вопроса.
   2. http://www.youtube.com/watch?v=SBhuqZnzcwA

Вопрос 17:
Когда-то X являлось для американцев символом надежды и защиты, но
некоторое время назад X стало вызывать и прямо противоположные
ассоциации. Чему же равно X?

Ответ:
911.

Источник:
   1. http://ru.wikipedia.org/wiki/Номера_телефонов_экстренных_служб
   2. http://ru.wikipedia.org/wiki/Террористические_акты_11_сентября_2001_года

Вопрос 18:
Согласно британскому ученому Ричарду Докинзу, ИКСЫ во многом ведут себя
подобно генам, хотя и не являются материальными объектами. Несмотря на
довольно долгий срок существования этого термина, наибольшее
распространение он получил лишь в последнее десятилетие, в чем явная
заслуга Интернета. Так что сегодня можно с уверенностью заявить, что
число ИКСОВ превышает 9000. Назовите ИКС коротко и ясно.

Ответ:
Мем.

Источник:
   1. http://ru.wikipedia.org/wiki/Эгоистичный_ген
   2. http://lurkmore.to/Более_9000

Вопрос 19:
Внимание, в вопросе есть замены.
   Рок-группа "Фактор страха" появилась в 1987 году и за свою историю
выпустила 13 альбомов. В январе 2009 года появилась шутка, что все
последние новости делятся на две темы: фактор страха и фактор без
страха. Какие два слова мы заменили в этих предложениях?

Ответ:
Сектор, Газа.

Зачет:
Сектор, газа.

Комментарий:
В январе 2009 года имели место газовый конфликт между Россией и Украиной
и операция Вооруженных Сил Израиля в Секторе Газа.

Источник:
   1. http://ru.wikipedia.org/wiki/Сектор_Газа_(группа)
   2. http://ru.wikipedia.org/wiki/Фактор_Страха_(группа)
   3. http://ru.wikipedia.org/wiki/Январь_2009_года
   4. http://www.anekdot.ru/id/379690/

Вопрос 20:
По словам Нины Ватолиной, "ОН - первый насмешник, первый агитатор,
первый сказочник, первый свидетель жизни народной. ОН - демократ по
рождению. ОН - для всех". К его помощи прибегали Малевич, Маяковский,
Бурлюк. Назовите ЕГО одним словом.

Ответ:
Лубок.

Комментарий:
В августе 1914 года авангардисты К. Малевич, А. Лентулов, В.В.
Маяковский, Д.Д. Бурлюк создали группу "Сегодняшний лубок", возродившую
старинные традиции батального лубка XIX века. Эта группа выпустила,
используя традицию лубочного примитива, серию из 22 листов на военные
сюжеты.

Источник:
   1. Н.Н. Ватолина. Пейзажи Москвы. - М.: Советский художник, 1983. -
С. 37.
   2. http://blog.trud.ru/users/rodich2007/post88950294/

Вопрос 21:
Однажды, гуляя поздним вечером по пустынным улицам города Туниса, автор
вопроса услышал звуки, доносящиеся из закрытого магазина. Как оказалось,
эти звуки были призваны отпугнуть воров. Ответьте абсолютно точно, на
кого же надеялся владелец магазина.

Ответ:
На Аллаха.

Комментарий:
Как известно, большинство населения Туниса исповедуют ислам. Владелец
магазина оставил включенным магнитофон с записью чтения Корана, надеясь,
что вор, если таковой будет, окажется человеком богобоязненным и,
услышав слова священной книги, задумается над своим поступком. А слова
"на кого же надеялся" - наводка на пословицу "На Бога надейся, а сам не
плошай".

Источник:
Личный опыт автора вопроса.

Вопрос 22:
В начале 80-х годов XX века автор вопроса занимался фехтованием на
саблях. Оружие, защитные маски и куртки предоставлялись в секции, а
перчатки приходилось покупать самим. Лучше всего для защиты руки
подходили мотоциклетные краги, которые стоили недешево и были дефицитом.
В итоге все ими обзавелись, но те, кому меньше повезло, пользовались
ТАКИМИ крагами. Некоторые машинки компании "VooV", выпускающиеся по
известной франшизе тоже могут быть ТАКИМИ. Какими ТАКИМИ?

Ответ:
Вывернутыми наизнанку.

Зачет:
По смыслу.

Комментарий:
Перчатка нужна одна, а продаются они парами. Покупали одну пару на
двоих, и одному из двух правшей приходилось выворачивать левую перчатку
наизнанку, чтобы можно было надеть ее на правую руку. Компания "VooV"
выпускает машинки-трансформеры, которые при трансформации выворачиваются
шиворот-навыворот.

Источник:
   1. ЛОАВ.
   2. http://milamama.ru/category/voov_2_in_1

Вопрос 23:
Первое упоминание об этой организации встречается в именном указе Петра
I от 1724 года, где к знакомому нам словосочетанию было добавлено "и
курьезных художеств" - ведь в то время помимо прочих премудростей ОНА
отвечала за организацию массовых празднеств и фейерверков. Назовите ЕЕ
двумя словами.

Ответ:
Академия наук.

Источник:
   1. И. Словцова. История петербургских районов: Энциклопедия. - М.:
Астрель; СПб.: Астрель-СПб, 2012. - С. 69.
   2. http://ru.wikipedia.org/wiki/Санкт-Петербургская_академия_наук

Вопрос 24:
У некоторых из вас АЛЬФА может ассоциироваться с ударным музыкальным
инструментом импортного производства, у других - с однолетним растением
семейства тыквенных, обогащенным хлоридом натрия, а у третьих - с полым
кондитерским изделием на основе масла какао. Примечательно, что в
зависимости от вашей ассоциации можно сделать некоторые выводы о том,
где проходило ваше детство. А какие два слова, написанные через дефис,
мы заменили на АЛЬФУ?

Ответ:
Жадина-говядина.

Зачет:
Ябеда-корябеда.

Комментарий:
В разных городах дети по-разному продолжают эту дразнилку. В Москве,
например, это был "турецкий барабан", в Питере - "пустая шоколадина", в
некоторых других городах - "соленый огурец".

Источник:
http://forum.lingvo.ru/actualthread.aspx?tid=117253

Тур:
6 тур. "По знакомству" (Москва)

Вопрос 1:
(pic: 20150142.jpg)
   Первый вопрос команды "По знакомству"!
   В этом вопросе словом "ИКС" заменено три других слова.
   Перед вами первый ИКС. На одном коллаже нынешний ИКС выложен
кокаином. Каким слоганом из трех слов подписан этот коллаж?

Ответ:
"Оставайтесь на первом".

Комментарий:
Кокаин на жаргоне называют первым, так же как амфетамин называют феном.
ИКС - логотип Первого канала.

Источник:
   1. http://ru.wikipedia.org/wiki/Первый_канал_(Россия)#.D0.9B.D0.BE.D0.B3.D0.BE.D1.82.D0.B8.D0.BF.D1.8B
   2. http://demotivators.to/p/393071/ostavajtes-na-pervom.htm

Автор:
Амаль Имангулов (Самара - Москва)

Вопрос 2:
Жительница города Химки пострадала в результате пожара, вызванного тем,
что она пыталась сжечь фотографии и вещи своего бывшего молодого
человека. По выражению сообщества Лентач, девушку убила АЛЬФА.
Распространение АЛЬФЫ заметно увеличилось с тех пор, как мексиканских
пчел и колибри научились заменять трудом человека. Что мы заменили на
АЛЬФУ?

Ответ:
Ваниль.

Источник:
   1. https://vk.com/oldlentach?w=wall-29534144_1501839
   2. http://www.vokrugsveta.ru/vs/article/7546/

Автор:
Вера Звягинцева (Москва)

Вопрос 3:
Вопреки достаточно распространенному мнению, ОН не является
изобретателем МАНСАРДЫ - он лишь способствовал ее распространению,
руководствуясь идеями гуманизма и равенства. Согласно афоризму
Станислава Ежи Леца, автор весь день не мог вспомнить слово "МАНСАРДА" -
голова сопротивлялась. Догадайтесь, что мы заменили на слово "МАНСАРДА",
и назовите человека, которого мы заменили на слово "ОН".

Ответ:
Гильотен.

Комментарий:
Мансарда тоже названа в честь конкретного человека.

Источник:
   1. http://www.vokrugsveta.ru/chronograph/2601/
   2. http://lib.ru/ANEKDOTY/lec.txt_with-big-pictures.html

Автор:
Вера Звягинцева (Москва)

Вопрос 4:
Согласно одной из распространенных версий, когда канцлер британского
казначейства Уильям Гладстон спросил у Майкла Фарадея, какой толк в
открытии электромагнитной индукции, ученый ответил, что когда-нибудь с
его открытием смогут СДЕЛАТЬ ЭТО. ЭТО СДЕЛАЛИ с солнцем на Балеарских
островах и с гражданским браком в Китае. Что мы заменили на "СДЕЛАТЬ
ЭТО"?

Ответ:
Обложить налогом.

Зачет:
По смыслу.

Источник:
   1. У. Гратцер. Эврики и эйфории. Об ученых и их открытиях. - М.:
КоЛибри, Азбука-Аттикус, 2011.
   2. http://m.forbes.ru/article.php?id=234249

Автор:
Вера Звягинцева (Москва)

Вопрос 5:
Ученый Джон Рид назвал "невоспетым героем" стереохимии и автором многих
открытий ТАКОГО ЕГО - ему ТАКОЙ ОН нечаянно помог изменить ход
эксперимента и совершить прорыв в его исследовании. Маловероятно, что
Майкл Фарадей был ТАКИМ, однако ИМ он был с 1812 по 1815 год. Какие два
слова, начинающиеся на одну и ту же букву, мы заменили на "ТАКОЙ ОН"?

Ответ:
Ленивый лаборант.

Источник:
   1. У. Гратцер. Эврики и эйфории. Об ученых и их открытиях. - М.:
КоЛибри, Азбука-Аттикус, 2011.
   2. http://ru.wikipedia.org/wiki/Фарадей,_Майкл

Автор:
Вера Звягинцева (Москва)

Вопрос 6:
В вопросе словом "ИКС" заменено выражение из двух слов.
   "ИКС" - ироничный заголовок статьи, рассказывающей о том, что
Владимир Путин принял решение повысить зарплаты чиновникам и
представителям силовых структур. Согласно уставу, регулирующему
отношения между военными, перед тем как произнести фразу "ИКС",
военнослужащий должен обратиться к начальнику "товарищ" и далее назвать
его полное воинское звание. Что мы заменили на ИКС?

Ответ:
Разрешите доложить.

Источник:
   1. http://www.gazeta.ru/politics/2013/10/01_a_5677533.shtml
   2. http://www.consultant.ru/document/cons_doc_LAW_165097/?frame=1

Автор:
Вера Звягинцева (Москва)

Вопрос 7:
   <раздатка>
   Нельзя отвращаться. Врубать заднего. Мол, этот хороший, я ему помогу,
а этот нет - пусть подыхает. Если ты помогаешь тем, кто тебе нравится,
ты просто расчесываешь его. Когда твой компас начинает крутиться как
сумасшедший, ты не обращаешь на него внимания. Потому что ты делаешь это
не для себя. В этом всё дело. Понимаешь?
   </раздатка>
   Перед вами отрывок из романа Виктора Пелевина "Бэтман Аполло". Какую
букву мы заменили в приведенной цитате?

Ответ:
Э.

Комментарий:
Расчесываешь эго.

Источник:
http://www.zhlob.info/?p=736

Автор:
Вера Звягинцева (Москва)

Вопрос 8:
Дэнни Де Вито утверждает, что ИКС - как атомная бомба: он есть у всех,
значит, есть и у него. Согласно одному афоризму, хороший ИКС - всегда
стОящий. Какое слово мы заменили на ИКС?

Ответ:
Адвокат.

Источник:
   1. http://esquire.ru/wil/danny-devito
   2. http://www.aphorism.ru/comments/jth838dsai.html

Автор:
Вера Звягинцева (Москва)

Вопрос 9:
Героиня Алексея Иванова при встрече с бывшим любовником снова начинает
выяснять отношения. Автор шутит, что этим она как бы хочет СДЕЛАТЬ ЭТО.
ЭТО нужно СДЕЛАТЬ, чтобы звуки читались раздельно. Ответьте точно, какие
пять слов мы заменили словами "СДЕЛАТЬ ЭТО".

Ответ:
Поставить вторую точку над i.

Комментарий:
Например, во французском словосочетании "na&iuml;ve remark" &iuml; [и с
двумя точками] ставится, чтобы показать, что "a" и "i" читаются
раздельно.

Источник:
   1. А. Иванов. Блуда и МУДО.
   2. http://ru.wikipedia.org/wiki/%C3%8F_(латиница)

Автор:
Амаль Имангулов (Самара - Москва)

Вопрос 10:
В одной шутке известный уроженец Эр-Рияда просит оценить свою внешность.
Ответьте как можно точнее, какую оценку он получает.

Ответ:
9/11.

Зачет:
9 из 11.

Комментарий:
Часто оценки маркируют дробью: 10/10 [десять слэш десять] и т.д. Город,
в котором на свет появился Усама бен Ладен, мог натолкнуть на необычную
шкалу измерения.

Источник:
Тредшот автора вопроса с сайта 4chan.

Автор:
Амаль Имангулов (Самара - Москва)

Вопрос 11:
Говоря об очевидных проблемах в расчете одного из численных методов,
Александр Злотник заметил, что ОНИ лежат на поверхности. Назовите ИХ.

Ответ:
Подводные камни.

Источник:
ЛОАВ.

Автор:
Амаль Имангулов (Самара - Москва)

Вопрос 12:
Герои называющейся двукоренным словом песни группы "Птицу Емъ" приводят
загипнотизированных жертв в свою секту, ДЕЛАЯ ЭТО. "ДЕЛО ВОЛШЕБНОГО
ЭТОГО" - книга мастера сексуальной коммуникации Екатерины Федоровой.
Какие три слова мы заменили словами "ДЕЛАТЬ ЭТО"?

Ответ:
Играть на флейте.

Комментарий:
Песня называется "Крысоловы". "Игра на волшебной флейте" - это, кхм-кхм,
частый синоним всяких там пошлостей.

Источник:
   1. Птицу Емъ - Крысоловы.
   2. http://www.fedorova-e.ru/about/my_book

Автор:
Амаль Имангулов (Самара - Москва)

Вопрос 13:
Дуплет.
   1. Эдуард I вынужден был запретить ИКС. ИКС имеет схожее
происхождение с английским глаголом to hurt [ту хёрт]. Назовите ИКС.
   2. Кристофер Бакли в книге "Флоренс Аравийская" описывает
французско-американский конфликт. В том числе, он пишет, как в США
поджигались Пежо, а ОНИ разрывались напополам. Назовите ИХ одним словом.

Ответ:
   1. Бугурт.
   2. Багет.

Комментарий:
   1. Рыцарское сражение.
   2. Антифранцузские настроения.
   Оба ответа на дуплет являются в Интернете синонимами
распространенного понятия "butthurt".

Источник:
   1. http://ru.wikipedia.org/wiki/Бугурт
   2. К. Бакли. Флоренс Аравийская.

Автор:
Амаль Имангулов (Самара - Москва)

Вопрос 14:
   <раздатка>
   - А я вс-сё думал, кто же это так жаждет со мной пообщатьс-ся, -
ПРОПУСК1 ему прямо в ухо знакомый холодный голос. Зельевар вздрагивает.
Он запоздало вспоминает, что недавно, в связи с ремонтом западного крыла
Малфой-мэнора (затеянного после одного не слишком удачного эксперимента
Волдеморта), Лорд переселился на второй этаж восточного крыла
   Автор рассказа, отрывок из которого перед вами, ПРОПУСК2 Волдеморта и
Северуса Снэйпа.
   </раздатка>
   Восстановите пропуски, содержащие по одному слову.

Ответ:
Шипит, шиппит.

Зачет:
Шипит, пэйрит.

Комментарий:
Шиппинг, или пэйринг, - настойчивое соединение персонажей вселенной
вместе. Ну а антагонисты мира Гарри Поттера только и делали, что
общались со змеями и всячески шипели.

Источник:
http://www.ficbook.net/readfic/160147

Автор:
Амаль Имангулов (Самара - Москва)

Вопрос 15:
Согласно Википедии, в эпоху Возрождения АЛЬФА ассоциировалась с
Сатурном. АЛЬФА ПЕРВОГО, как ни странно, не показана в АЛЬФЕ ВТОРОГО. Мы
не спрашиваем, какое слово греческого происхождения мы заменили на
АЛЬФУ. Назовите ПЕРВОГО и ВТОРОГО.

Ответ:
Дюрер, Триер.

Комментарий:
АЛЬФА - меланхолия.

Источник:
   1. http://ru.wikipedia.org/wiki/Меланхолия
   2. Общие знания.

Автор:
Анна Кузнецова (Липецк - Москва)

Вопрос 16:
Героя фильма "Lego Movie" перебивают на середине мотивационной речи.
Ответьте точно, какое английское ругательство ему кричат в ответ на его
следующую реплику.

Ответ:
Butt.

Комментарий:
На русский язык эту шутку перевели как - "Нет, тут должно было быть
"но"" - "Сам ты дно!". Первая половина мотивационной речи часто
посвящена тому, что всё плохо, но - не надо отчаиваться, нужно
продолжать бороться.

Источник:
Фильм "Lego Movie", английский оригинал и русский перевод.

Автор:
Амаль Имангулов (Самара - Москва)

Вопрос 17:
Одним из главных научных достижений этого английского ученого стали его
открытия в области атомной теории строения вещества. Однако его имя
стало известно во многом благодаря его неудачным занятиям ботаникой: он
не мог разобраться в классификации цветущих растений, долгое время
объясняя свою неспособность запутанностью названий. Назовите термин,
который его прославил.

Ответ:
Дальтонизм.

Источник:
У. Гратцер. Эврики и эйфории. Об ученых и их открытиях. - М.: КоЛибри,
Азбука-Аттикус, 2011. - С. 134.

Автор:
Вера Звягинцева (Москва)

Вопрос 18:
Герой песни группы "Птицу Емъ", находясь ПРОПУСК, каждой клеточкой тела
почувствовал музыку и проболтался милиционерам о преступлении своих
друзей. Героиня Джеффри Евгенидиса, находясь ПРОПУСК, решила написать
гневное и бессмысленное письмо в газету. В каком заведении она
находилась?

Ответ:
Парикмахерская.

Комментарий:
В этом вопросе обыгрываются жаргонные названия наркотиков: как кокаин
называют первым, так и амфетамин называют феном. Пропущено "под колпаком
фена" (под феном). В первой части вопроса речь идет об амфетамине,
пробивающем на болтливость, танцы и бессмысленного содержания письма.

Источник:
   1. Птицу Емъ - Нумеро Уно.
   2. Джеффри Евгенидис. Девственницы-самоубийцы.

Автор:
Амаль Имангулов (Самара - Москва)

Вопрос 19:
(pic: 20150143.jpg)
   Дуплет.
   1. Назовите фильм, который зашифрован на первом постере из серии
"Классика кино".
   2. Назовите фильм, который зашифрован на втором постере из серии
"Классика кино".

Ответ:
   1. "В джазе только девушки".
   2. "Последнее танго в Париже".

Источник:
   1. http://www.poster.pl/poster/homework_pol_zartem/pl
   2. http://www.poster.pl/poster/homework_ostatnie_tango/pl

Автор:
Ольга Старчикова (Тамбов - Москва)

Вопрос 20:
Писательница Мариам Петросян дает неопрятному герою такое описание: Он
был равнодушен к еде и неряшлив в одежде, носил траур и подолгу не менял
носков. Какие два слова мы пропустили в предыдущем предложении?

Ответ:
Под ногтями.

Источник:
Мариам Петросян. Дом, в котором... - М.: Live Book, 2014. - С. 106.

Автор:
Наталия Павлова (Москва)

Вопрос 21:
   <раздатка>
   P&oacute;jd&#378;&#380;e, ki&#324; t&#281; chmurno&#347;&#263; w
g&#322;&#261;b flaszy.
   </раздатка>
   Фразу, розданную вам, можно перевести как "Давай, брось свою тоску на
дно бутылки". Назовите то, чем эта фраза является, двумя словами на одну
и ту же букву.

Ответ:
Польская панграмма.

Источник:
http://pl.wikipedia.org/wiki/Pangram

Автор:
???

Вопрос 22:
Прослушайте стихотворение Александра Матюшкина-Герке:
   Вспыхает небо, разбужая ветер,
   Проснувший гомон птичьих голосов;
   Проклинывая всё на белом свете,
   Я вновь бежу в нетоптанность лесов.
   Шуршат зверушки, выбегнув навстречу,
   Приветливыми лапками маша,
   Я среди тут пробуду целый вечер,
   Бессмертные творения пиша.
   Но, выползя на миг из тины зыбкой,
   Болотная зелёновая тварь
   Совает мне с заботливой улыбкой
   Большой...
   Закончите двумя словами.

Ответ:
"... орфографический словарь".

Источник:
http://pikabu.ru/story/kosmatyiy_oblak_667250

Автор:
Наталия Павлова (Москва)

Вопрос 23:
Серия постеров художника Тан Яу Хунга посвящена известным цитатам. На
одном из постеров изображены два яблочных огрызка, которые обрисовывают
портрет автора цитаты. Напишите эту цитату.

Ответ:
Stay hungry, stay foolish.

Зачет:
В русском переводе.

Источник:
   1. http://www.tangyauhoong.com/portfolio/quote-illustration/
   2. http://ok-english.ru/stay-hungry-stay-foolish-9-vyiskazyivaniy-stiva-dzhobsa/

Автор:
Наталия Павлова (Москва)

Вопрос 24:
В ответе на этот вопрос могут упоминаться: просторечное упоминание
интуиции, карликовая лошадь, слово, к которому восходит слово "ассасин".
Назовите этот вопрос.

Ответ:
Что курил автор вопроса?

Комментарий:
Интуиция - это "чуйка", сленговое название сорта "чуйская долина",
карликовая лошадь - "пони", сленговое название взрощенной на водном
парнике гидропоники, ну а ассасин, или хашишин, произошло от слова
"гашиш".

Источник:
ЛОАВ.

Автор:
Амаль Имангулов (Самара - Москва)

Тур:
7 тур. "Сэр Пухов" (Серпухов)

Вопрос 1:
Жан Эффель курил так много, что, по воспоминаниям друзей, "в день ему
требовалась всего одна ОНА...". Назовите двумя словами на одну и ту же
букву место, где ОНА была в цене.

Ответ:
Планета Плюк.

Зачет:
Планета Пацак.

Комментарий:
ОНА - спичка. После того как от спички Эффель прикуривал первую
сигарету, остальные он прикуривал от предыдущей сигареты.

Источник:
   1. О. Сердобольский. Автографы в антракте. - СПб.: Нотабене, 2001. -
С. 55.
   2. http://ru.wikipedia.org/wiki/Кин-дза-дза!

Автор:
Олег Холодов, Евгений Кононенко

Вопрос 2:
Сотрудница секретной лаборатории на вопрос Шерлока Холмса, чем она здесь
занимается, отвечает: "Смешиваю гены и скрещиваю ИХ". При помощи одного
из НИХ легко определить, находитесь ли вы в зоне поражения во время
ядерного взрыва. Назовите его абсолютно точно.

Ответ:
Большой палец.

Комментарий:
Есть простой способ определить, находится ли человек в зоне поражения во
время ядерного взрыва. Для этого нужно вытянуть руку с оттопыренным
большим пальцем в сторону "грибка". Если он выше пальца - плохо.

Источник:
   1. "Шерлок", s01e02.
   2. http://copypast.ru/2014/08/28/interesnye_fakty_o_bolshom_palce.html

Автор:
Олег Холодов

Вопрос 3:
Дуплет.
   1. По версии писателя Алексея Вязовского, ОН появился в конце XII
века, во время противостояния кланов Минамото и Тайра, как бы
символизируя победу клана Минамото. Назовите ЕГО.
   2. Артемий Лебедев в своем твите от 10 августа 2014 года сообщил, что
в Японии после первой брачной ночи принято выставлять ЕГО на всеобщее
обозрение. Назовите ЕГО.

Ответ:
   1. Японский флаг.
   2. Японский флаг.

Комментарий:
   1. Минамото выступали под белыми знаменами, а Тайра - под красными.

Источник:
   1. http://samlib.ru/i/isaew_a_w/mikado.shtml
   2. http://ru.wikipedia.org/wiki/Флаг_Японии
   3. https://plus.google.com/+temalebedev/posts/EahhiFS3vbt

Автор:
Олег Холодов

Вопрос 4:
В рассказе Брайана Олдисса "Слюнное дерево" герои запасаются мукой и
идут сражаться с пришельцем, обладающим тем же свойством, какое приобрел
герой произведения 1897 года. Мы не просим назвать этого героя. Назовите
его врожденную особенность.

Ответ:
Альбинизм.

Зачет:
Альбинос.

Комментарий:
Пришелец - невидимый, мука облепит его, и это позволит эффективнее с ним
сражаться. Произведение - "Человек-невидимка". Кстати, рассказ "Слюнное
дерево" написан к столетию со дня рождения Герберта Уэллса. Для полной
невидимости человек должен был быть альбиносом, каковым Гриффин и
являлся.

Источник:
   1. http://ru.wikipedia.org/wiki/Человек-невидимка
   2. Брайан Олдисс. Слюнное дерево.
http://www.flibusta.net/b/309248/read

Автор:
Ася Самойлова, Надежда Богданова

Вопрос 5:
Недалеко от антарктической станции "Амундсен - Скотт" учеными установлен
столб, к которому участники экспедиций прибивают указатели, на которых
написаны расстояния и направления до различных географических точек
Земли. Есть таблички с указателями направления на Вашингтон, Нью-Йорк,
Лондон, Москву и т.д. Все они выполнены в оригинальной манере. Назовите
словом голландского происхождения то, в виде чего сделан указатель на
Северный полюс.

Ответ:
Флюгер.

Комментарий:
Столб находится точно на Южном полюсе. К нему прикреплен флюгер, на нем
написано "Nord" (север), и он всегда указывает на север, так как ветер
всегда указывает на север, куда бы он ни дул. Только вот каждый год
из-за дрейфа ледников приходится столб переносить.

Источник:
"Дискавери". Антарктическая станция "Амундсен - Скотт".

Автор:
Святослав Холодов

Вопрос 6:
Французский юморист Ги Бедо накануне своего юбилея пошутил:
"Единственное утешение в том, что скоро я задую шестидесятую свечку, -
это то, что в шестидесятилетнем возрасте есть ОН". Назовите ЕГО
односложным словом.

Ответ:
Sex.

Комментарий:
Во французском слове "sexag&eacute;naire" - шестидесятилетний.

Источник:
Мирей Гильяно. Француженки подтяжек не делают.

Автор:
Виктория Холодова

Вопрос 7:
Интернет наводнили разные советы для поклонниц одного популярного
сериала. Среди советов есть и такой: "Дождитесь полнолуния и ровно в
полночь съешьте селедочный хвост, запив стаканом молока. Для закрепления
результата примите горячую ванну". Какого эффекта мечтают достигнуть
поклонницы, используя эти советы?

Ответ:
Стать русалкой.

Зачет:
По смыслу.

Комментарий:
Сериал - "H2O: Просто добавь воды" про девочек-подростков, которые
однажды попадают в древнюю пещеру и превращаются в русалок. В
поисковиках на сегодня очень популярен запрос "как стать русалкой".

Источник:
   1. http://www.sovetprost.ru/kak-legko-i-bystro-stat-nastoyashhej-rusalkoj.html
   2. http://www.ymni4ka.ru/kak-stat-rusalkoj-s-siloj/

Автор:
Надежда Богданова

Вопрос 8:
   <раздатка>
   Я был так взволнован, что едва мог усидеть на месте от мыслей в
голове. Такое приятное чувство возбуждения испытывает только ПРОПУСК.
ПРОПУСК в начале долгого пути.
   </раздатка>
   Перед вами цитата из фильма "Побег из Шоушенка" - естественно, в
русском переводе. Заполните пропуск двумя словами.

Ответ:
Свободный человек.

Комментарий:
Это говорил герой Моргана Фримена. Фримен = свободный человек.

Источник:
Фильм "Побег из Шоушенка" (1994).

Автор:
Надежда Богданова

Вопрос 9:
Во времена Второй мировой войны А. Бабкиным и В. Сосновским был
предложен препарат из НЕЕ для восполнения кровопотери раненых. В 2010
году шотландская компания представила мировую новинку - ЕЕ в бутылке.
Среди главных покупателей своего товара компания видит рестораны. ОНА
станет незаменимым продуктом для тех поваров, кто уделяет особое
внимание вкусовым качествам блюд. Назовите ЕЕ.

Ответ:
Морская вода.

Источник:
   1. http://www.trinity-metody.com.ua/read/id=107/
   2. http://www.seryogina.ru/sea-water-aquamara/

Автор:
Надежда Богданова, Олег Холодов

Вопрос 10:
По одной из легенд, пять ЕЕ лепестков напоминают о пяти чувствах,
дарованных человеку, и о пяти ранах Христа. В Англии есть грустное
поверье: якобы ОНИ появляются в больших количествах на полях сражений,
где погибло много солдат. Назовите ЕЕ.

Ответ:
Незабудка.

Комментарий:
Синие цветы - словно письма, посланные погибшими с того света: "Не
забывайте нас!".

Источник:
   1. http://c-amazonka.ru/post111681395/
   2. http://www.liveinternet.ru/users/3090603/post124901208/

Автор:
Олег Холодов

Вопрос 11:
Автор вопроса Татьяна Соломенникова хоть и не является моделью, но
считает, что чем-то похожа на Лесли. К тому же ее прозвище похоже на
прозвище Лесли, которое та получила почти полвека назад и под которым
стала известна всему миру. Мы не спрашиваем у вас прозвище автора
вопроса. Назовите прозвище Лесли.

Ответ:
Твигги.

Зачет:
Тростинка, Соломинка.

Комментарий:
Была тонкой, как тростинка. Прозвище автора вопроса пошло от ее фамилии.

Источник:
http://ru.wikipedia.org/wiki/Твигги

Автор:
Татьяна Соломенникова

Вопрос 12:
Внимание, в вопросе есть замены.
   В романе Олега Бубелы "Герой" полководец жалеет, что "единственный ОН
быстро перешел в разряд ТАКИХ". ТАКИЕ ОНИ - один из предметов, который
изучают в "Школе Монстров". Какой термин мы заменили на "ТАКОЙ ОН"?

Ответ:
Мертвый язык.

Комментарий:
У героя в результате пыток умер язык (пленный). В "Школе Монстров", так
как они все вымирающие, один из основных предметов - "мертвые языки",
чтобы монстры могли общаться друг с другом.

Источник:
   1. Олег Бубела. Герой. http://www.flibusta.net/b/231586/read
   2. http://www.mhfans.ru/interestingly/23-chto-takoe-mertvye-yazyki.html

Автор:
Олег Холодов

Вопрос 13:
Эбису - один из семи богов удачи в синтоизме. Обычно изображается в
высокой шляпе с удочкой в руках и большой рыбой. Назовите предельно
точно общественные места, в которых часто встречаются изображения Эбису.

Ответ:
Рестораны, где подают рыбу фугу.

Зачет:
По смыслу.

Комментарий:
Да сопутствует вам удача при поедании рыбы фугу!

Источник:
http://ru.wikipedia.org/wiki/Эбису

Автор:
Олег Холодов

Вопрос 14:
Хозяин одного мини-отеля использует понятную систему информирования с
помощью цветных флагов. Например, поднятый красный флаг сигнализирует о
запрете купания, зеленый - о штиле на море, желтый - "волны, будьте
осторожны". В каких случаях хозяин отеля поднимет белый флаг?

Ответ:
При наличии в отеле свободных номеров.

Зачет:
По смыслу.

Комментарий:
Номера в отеле сдаются.

Источник:
ЛОАВ.

Автор:
Олег Холодов

Вопрос 15:
Википедия утверждает, что прототип ЭТОГО появился в 1733 году. Среди
различных видов ЭТОГО встречаются: рюкзак, книжка, чемодан. Согласно
афоризму, будущее нации зависит от количества ЭТОГО. Назовите ЭТО.

Ответ:
Детская коляска.

Источник:
   1. http://ru.wikipedia.org/wiki/Детская_коляска
   2. http://www.pervenez.ru/kolyaska.html
   3. http://www.novate.ru/blogs/110613/23190/
   4. http://citaty.info/quote/man/211744

Автор:
Олег Холодов, Ася Самойлова

Вопрос 16:
"Это даже не рисунок мальчишки, а узор не окончившего ПТУ резчика". Так
о "НЕЙ" отозвался Борис Гребенщиков. В 2010 году о правах на НЕЕ заявила
некая Анхелес Дуран. Впрочем, как ехидно замечают журналисты, вступить в
права ей будет, мягко сказать, затруднительно. Назовите ЕЕ четырьмя
словами.

Ответ:
Звезда по имени Солнце.

Комментарий:
Так Гребенщиков высказался о песни Цоя "Звезда по имени Солнце". Анхелес
Дуран провозгласила себя хозяйкой Солнца.

Источник:
   1. http://volna.afisha.ru/archive/onesong_zvezda/
   2. http://izvestia.ru/news/481810

Автор:
Татьяна Соломенникова

Вопрос 17:
Внимание, в вопросе есть замена.
   При правильном подборе коромысла всегда учитывается вес и рост
владельца. На самом деле, если верить производителям, существует много
разновидностей коромысел. В частности, коромысло для кормления грудью,
коромысло для новорождённых. Недавно американский архитектор Пол Кветон
совместил коромысло с домиком для кошки. Его изобретение стало хитом
среди зажиточных жителей США и Европы. Какое сложное слово, пишущееся
через дефис, мы заменили на "коромысло"?

Ответ:
Кресло-качалка.

Комментарий:
Обоснование замены: по-английски кресло-качалку часто называют "rocker",
на русский это слово также переводится как "коромысло".

Источник:
   1. http://www.kreslo-kachalka.ua/blog/?p=31
   2. http://www.drunov.ru/news/kreslo_kachalka_dlya_dvoikh/
   3. http://www.malutka.net/kreslo-kachalka-dlya-novorozhdennogo
   4. http://www.mebelniykrug.ru/kak-sdelat-kreslo-kachalku-svoimi-rukami.html
   5. http://translate.academic.ru/кресло-качалка/ru/en/1

Автор:
Алексей Паклин

Вопрос 18:
Дуплет.
   1. В 2011 году во время хорошо вам известного мероприятия, по данным
агентства "РИА Новости", один человек ответил на 90 вопросов за 4 часа
33 минуты. Кто этот человек?
   2. Как ни удивительно, но провести почти 19 дней без сна и установить
рекорд Роберту Макдональду помогло ОНО. Назовите ЕГО.

Ответ:
   1. [Владимир] Путин.
   2. Кресло-качалка.

Комментарий:
   1. Речь идет о ежегодной передаче "Прямая линия" с Владимиром
Путиным.
   2. С 14 марта по 2 апреля добровольно провел без сна 453 часа 40
минут, борясь с ним в кресле-качалке.

Источник:
   1. http://ria.ru/infografika/20130424/934270082.html
   2. http://www.para-psychology.ru/perception7.htm

Автор:
Олег Холодов, Надежда Богданова

Вопрос 19:
Этот эпоним имеет много значений, большинство из которых на данный
момент устарело. Например, Готье де Куанси в одном из стихотворений
использовал это слово как метафору для характеристики абсолютно
никчёмного человека. В средние века этим словом называли сочинения по
искусству счета. В XVIII веке в одном из германских словарей это слово
объяснялось как понятие о четырех арифметических операциях. Назовите это
слово.

Ответ:
Алгоритм.

Комментарий:
Слово "алгоритм" происходит от имени хорезмского ученого Абу Абдуллах
Мухаммед ибн Муса аль-Хорезми (алгоритм - аль-Хорезми). В первой
половине XII века книга аль-Хорезми в латинском переводе проникла в
Европу. Переводчик, имя которого до нас не дошло, дал ей название
"Algoritmi de numero Indorum" ("Алгоритмы о счете индийском"). Таким
образом, мы видим, что латинизированное имя среднеазиатского ученого
было вынесено в заглавие книги, и сегодня считается, что слово
"алгоритм" попало в европейские языки именно благодаря этому сочинению.

Источник:
http://ru.wikipedia.org/wiki/Алгоритм

Автор:
Святослав Холодов

Вопрос 20:
По одной версии, дружеское прозвище композитора повлияло на выбор имени
героини комедии - мол, именно так композитор закрепил в массовом
сознании свое авторство знаменитой песни. Позже прозвище перешло по
наследству. Назовите этого композитора.

Ответ:
Исаак Дунаевский.

Комментарий:
Прозвище Дунаевского, "переданное" и его сыну Максиму, также
композитору, - "Дуня". По сюжету фильма "Волга-Волга", музыку к которому
написал Исаак Дунаевский, разыскивается автор песни, о которой известно
лишь то, что "ее написала Дуня".

Источник:
   1. Документальный фильм "Максим Дунаевский. Жизнь по завещанию".
   2. http://ru.wikipedia.org/wiki/Волга-Волга

Автор:
Евгений Кононенко

Вопрос 21:
В сериале "Две легенды" бизнесмен сравнивает ИХ с мужской бижутерией. ИХ
качество и количество указывает на статус владельца. Алан Пинкертон
пошутил, что для защиты от экономического кризиса нужно приобретать ИХ.
Назовите ИХ сложносоставным словом, которое является названием "самого
простого фильма Акиры Куросавы".

Ответ:
Телохранители.

Комментарий:
Алан Пинкертон в 1850 году в Чикаго организовал Национальное детективное
агентство Пинкертона. На сегодняшний день Pinkerton Government Services
является одним из крупнейших охранных агентств в мире. "Телохранитель" -
кинофильм режиссера Акиры Куросавы, снятый в жанре дзидайгэки. Остальные
работы японца - философские притчи.

Источник:
   1. Сериал "Две легенды", s01e01.
   2. "Алан Пинкертон - история успеха".
   3. http://kino-leone.narod.ru/tel.htm

Автор:
Олег Холодов

Вопрос 22:
Во времена работы в издательстве "Алконост" Александр Блок за привычку
одевать строгие черные костюмы или смокинги для фотографирования получил
прозвище ШУМАН. Это прозвище настолько хорошо передавало сущность поэта,
что Владимир Орлов свою биографическую книгу назвал "ШУМАН. Жизнь
Александра Блока". Какое слово мы заменили в этом вопросе?

Ответ:
Гамаюн.

Комментарий:
У птицы Гамаюн черная окраска. Александру Блоку шел девятнадцатый год,
когда он написал стихотворение "Гамаюн, птица вещая". По одной из
версий, слово "Гамаюн" произошло от славянского слова "гам" - шум.

Источник:
   1. http://blok.ouc.ru/andrei-turkov.html
   2. http://az.lib.ru/b/blok_a_a/text_0430.shtml

Автор:
Олег Холодов

Вопрос 23:
По одной из версий, ЕЕ завезли в Россию немцы, которые таким способом
пытались прикрыть свои попытки соблазнять русских барышень. Есть даже
версия, что ЕЕ название возникло от традиционного немецкого приглашения
к интимной близости. Назовите ЕЕ и не перепутайте.

Ответ:
Фига.

Комментарий:
Fick-fick machen - так звучало традиционное немецкое приглашение к
интимной близости. Фиговый листок - переносное лицемерное прикрытие
заведомо бесстыдных действий, нечестных поступков; существует много
различных версий, в которых утверждается что первая одежда была сделана
не из фиговых листьев, а листьев каких-то других растений. Чаще всего
упоминается яблоко.

Источник:
http://copypast.ru/2014/08/28/interesnye_fakty_o_bolshom_palce.html

Автор:
Олег Холодов

Вопрос 24:
Внимание, в вопросе есть замена.
   АВТОГОЛ Тотти стал причиной недавнего раздора между президентами
"Ромы" и "Лацио". В России в 2014 году надзирающий орган рекомендовал
обходиться без АВТОГОЛОВ. Какое слово мы заменили на "АВТОГОЛ"?

Ответ:
Селфи.

Комментарий:
Русский вариант слова "селфи" - себяшка, самострел. Франческо Тотти
забрал телефон у тренера вратарей "Ромы" после своего второго гола в
ворота "Лацио" и сфотографировал себя на фоне беснующейся от восторга
фанатской трибуны, что стало причиной перебранки президентов клубов.
Селфи является главной причиной распространения вшей, заявило курское
управление Роспотребнадзора в октябре 2014 года.

Источник:
   1. http://football.sport-express.ru/reviews/52631/
   2. http://ru.wikipedia.org/wiki/Селфи

Автор:
Олег Холодов

Тур:
8 тур. "Водопой кротов" (Тверь)

Вопрос 1:
Героиня Маркеса в молодости втайне ДЕЛАЛА ЭТО вместе со своей двоюродной
сестрой, причем они ДЕЛАЛИ ЭТО шиворот-навыворот, как солдаты во время
войны по ночам. Какой глагол мы заменили на "ДЕЛАТЬ ЭТО"?

Ответ:
Курить.

Комментарий:
Солдаты курили сигареты огоньком внутрь, чтобы не выдать свое
местоположение ночью. В дальнейшем уважаемые знатоки убедятся, что и
авторы пакета курили что попало и тоже шиворот-навыворот.

Источник:
Г. Гарсиа Маркес. Любовь во время чумы.
http://www.flibusta.net/b/96270/read

Автор:
Станислав Попов (Тверь)

Вопрос 2:
В магазине жена автора вопроса повертела в руках блузку и посетовала,
что производители экономят на декоративных элементах, а в результате
получается ОНА. Назовите ЕЕ двумя словами, начинающимися на одну и ту же
букву.

Ответ:
Перевязь Портоса.

Комментарий:
Блестки, пайетки и прочий декор располагался только на лицевой стороне.

Источник:
Личный опыт жены автора вопроса.

Автор:
Станислав Попов (Тверь)

Вопрос 3:
Одна американская компания выпустила разновидность ЭТОГО под названием
"Impossible", отличающуюся неровными краями и наличием пяти
дополнительных деталей. Назовите ЭТО одним заимствованным словом.

Ответ:
Паззл.

Комментарий:
Паззл стал намного сложнее, так как его нельзя начать собирать с более
простой "рамки".

Источник:
http://www.ugames.com

Автор:
Станислав Попов (Тверь)

Вопрос 4:
(pic: 20150144.jpg)
   Перед вами фотографии Ангелы Меркель, использованные в качестве
рекламы одного товара. Лучшие образцы этого товара выдерживают более 200
килограммов на квадратный сантиметр. Назовите этот товар.

Ответ:
Суперклей.

Зачет:
Клей.

Источник:
   1. http://9gag.com/gag/26728
   2. http://ru.wikipedia.org/wiki/Цианоакрилат

Автор:
Станислав Попов (Тверь)

Вопрос 5:
В Риме после официального принятия христианства были уничтожены все
статуи языческих императоров, кроме памятника Марку Аврелию, потому что
его ПРОПУСК. Дмитрий Дибров рассказывал, как однажды в Киеве его ПРОПУСК
Хабенского. Заполните пропуск тремя словами.

Ответ:
Приняли за Константина.

Источник:
   1. http://en.wikipedia.org/wiki/Equestrian_Statue_of_Marcus_Aurelius
   2. http://www.ddibrov.com/press/dmitriy-dibrov-kogda-ya-v-efire-u-menya-v-glazah-smertelnaya-skuka/

Автор:
Станислав Попов (Тверь)

Вопрос 6:
Врач-ортодонт, оперирующий в крупном городе, сделал логотип своей
клиники в виде силуэта известного здания, расположенного там же. В каком
городе это произошло?

Ответ:
Сидней.

Комментарий:
(pic: 20150145.jpg)
   Ортодонты, в основном, занимаются выправлением прикуса. Оперный театр
Сиднея в профиль выглядит как ОЧЕНЬ кривой ряд зубов. Не зря в вопросе
стоит слово "оперирующий".

Источник:
http://www.findingnemo2.co.uk/p-sherman-42-wallaby-way-sydney-apparently-real-orthodontic-clinic/

Автор:
Станислав Попов (Тверь)

Вопрос 7:
Роман Тонино Бенаквисты "Малавита" написан, в основном, на французском,
но содержит и английские реплики. К примеру, в разговоре об одной опере
один из героев замечает: "Если она достаточно хороша для тебя, она и для
меня достаточно хороша". Назовите заглавного героя этой оперы.

Ответ:
Борис Годунов.

Комментарий:
Игра слов "Godunov = Good Enough".

Источник:
Тонино Бенаквиста. Малавита. http://www.flibusta.net/b/316583/read

Автор:
Станислав Попов (Тверь)

Вопрос 8:
Есть легенда, что поэт Ли Бо утонул в реке, когда ночью в лодке спьяну
попытался СДЕЛАТЬ ЭТО. На некоторых алхимических гравюрах ЭТО ДЕЛАЕТ
Солнце. Какие два слова мы заменили на "ДЕЛАТЬ ЭТО"?

Ответ:
Обнять Луну.

Зачет:
Поймать Луну, поцеловать Луну и т.п.

Источник:
http://en.wikipedia.org/wiki/Li_Bai

Автор:
Станислав Попов (Тверь)

Вопрос 9:
Ученый и философ Дуглас Хофштадтер сформулировал такой шуточный закон:
"Что бы вы ни делали, это всегда займет больше времени, чем
планировалось, даже если учесть ЕГО". Назовите ЕГО двумя словами.

Ответ:
Закон Хофштадтера.

Источник:
http://en.wikipedia.org/wiki/Hofstadter's_law

Автор:
Станислав Попов (Тверь)

Вопрос 10:
После мятежа на "Баунти" на поиски предателей был послан корабль. Экипаж
напал на след мятежников, задержал их и поместил в крохотную камеру на
судне. Однако на пути назад корабль разбился о риф и начал тонуть,
поэтому арестантов пришлось выпустить, но ничего катастрофического в
результате не случилось. А какое имя носил этот корабль?

Ответ:
"Пандора".

Комментарий:
А камеру на корабле так и называли - "ящик Пандоры".

Источник:
http://en.wikipedia.org/wiki/HMS_Pandora_(1779)

Автор:
Станислав Попов (Тверь)

Вопрос 11:
На сайте kinonews.ru приводится следующая справка: "Алехандро Аменабар
родился 31 марта 1973 года, был режиссером фильмов: "Море внутри"
(2004), "ПРОПУСК" (2001), "Дипломная работа" (1996) и ПРОПУСК".
Восстановите любой из пропусков.

Ответ:
Другие.

Источник:
http://www.kinonews.ru/person_3736/alejandro-ameni

Автор:
Станислав Попов (Тверь)

Вопрос 12:
(pic: 20150146.jpg)
   Автор вопроса составил себе такую таблицу, чтобы не запутаться. Какая
английская фамилия скрыта от вас в последней строчке?

Ответ:
Windsor.

Зачет:
Виндзор.

Комментарий:
Это памятка, как вязать различные галстучные узлы.

Источник:
   1. ЛОАВ.
   2. http://www.agreeordie.com

Автор:
Станислав Попов (Тверь)

Вопрос 13:
Объясняя один из способов завязывания галстучного узла, видеоблогер
Алекс Красны советует с самого начала оставить один конец галстука очень
длинным, так как узел очень ТАКОЙ. А что было ТАКИМ в названии фильма
2012 года?

Ответ:
Игры.

Источник:
   1. http://www.agreeordie.com/
   2. http://ru.wikipedia.org/wiki/Голодные_игры_(фильм)

Автор:
Станислав Попов (Тверь)

Вопрос 14:
Герой романа Фаулза "Коллекционер" похищает девушку и втайне ото всех
держит ее в подвале. Он даже устанавливает специальную печь, назначение
которой он описывает фразой, практически совпадающей с русской
поговоркой. Воспроизведите эту поговорку.

Ответ:
Сор из избы не выносить.

Источник:
Дж. Фаулз. Коллекционер. http://www.flibusta.net/b/338811/read

Автор:
Станислав Попов (Тверь)

Вопрос 15:
   <раздатка>
   "***** Vina Musica"
   </раздатка>
   Группа "Corvus Corax", исполняющая музыку в стиле "средневековый
рок", не чужда идеалам XX века. Один из их альбомов называется "ПРОПУСК
Vina Musica". Имя какого божества мы пропустили?

Ответ:
Venus.

Зачет:
Венера.

Комментарий:
Средневековый аналог "Sex, Drugs, Rock'n'Roll".

Источник:
http://www.corvuscorax.de

Автор:
Станислав Попов (Тверь)

Вопрос 16:
Французская певица Брижит Лен, также увлекающаяся средневековой музыкой,
выпустила альбом песен XII-XIII веков, воспевающих женщин. Название
этого альбома представляет собой короткий палиндром из двух слов.
Воспроизведите это название.

Ответ:
"Ave Eva".

Источник:
http://www.amazon.com/Ave-Eva-Brigitte-Lesne/dp/B000005W48/

Автор:
Станислав Попов (Тверь)

Вопрос 17:
В конце 1930-х годов влиятельная американская организация "Союз Дочерей
Конфедерации" заявила, что лучше пусть ЕЮ станет англичанка, чем янки.
Назовите имя и фамилию этой англичанки.

Ответ:
Вивьен Ли.

Комментарий:
Речь шла о кастинге на роль Скарлетт О'Хары - типичной южанки.

Источник:
http://diletant.ru/state_exam/?id=23033858

Автор:
Станислав Попов (Тверь)

Вопрос 18:
Известно, что Жерар Депардье сбежал от французских налогов сначала в
Бельгию, а потом в Россию. Годом раньше в Лондон эмигрировал и другой
французский актер - Кристиан Клавье. В посвященной этому заметке на
сайте lenta.ru говорится, что ОНИ-то за свою родину сражались. Назовите
ИХ имена.

Ответ:
Астерикс и Обеликс.

Источник:
В вопросе.

Автор:
Станислав Попов (Тверь)

Вопрос 19:
Как рассказывает статья в журнале "Трансаэро", венский мастер Михаэль
Тонет в середине XIX века предложил новую технику изготовления мебели, и
хотя поначалу результат многим не понравился, мастер продолжал
НАСТАИВАТЬ НА УБЕЖДЕНИЯХ. Песня "НАСТАИВАЙ НА УБЕЖДЕНИЯХ" стала основной
темой отечественного фильма. Какого?

Ответ:
"Брат-2".

Комментарий:
Мастер продолжал гнуть свою линию.

Источник:
Журнал "Трансаэро", 2013, N 8. - С. 232-234.

Автор:
Станислав Попов (Тверь)

Вопрос 20:
На афише, анонсирующей выход фильма Ларса фон Триера "Нимфоманка", были
запечатлены лица сыгравших актеров, изображающих момент оргазма. Подпись
внизу афиши состояла из двух английских слов. Воспроизведите эти слова.

Ответ:
Coming soon.

Комментарий:
Двойной смысл словосочетаний "скоро выходит" (в вопросе указано, что это
анонс) и "скоро кончит".

Источник:
http://culturmag.de/litmag/lars-von-trier-nymphmaniac-1-2/80397

Автор:
Станислав Попов (Тверь)

Вопрос 21:
(pic: 20150147.gif)
   Внимание, в вопросе есть замены.
   Город Нэшвилл в штате Теннесси, где расположена штаб-квартира
Национальной Конвенции Баптистов, иногда называют "ВЕРШИНОЙ БЕРМУДСКОГО
ТРЕУГОЛЬНИКА". На розданном вам рисунке - одна из моделей ВЕРШИНЫ
ТРЕУГОЛЬНИКА в натуральную величину в разрезе. Какие слова мы заменили
на "ВЕРШИНА БЕРМУДСКОГО ТРЕУГОЛЬНИКА"?

Ответ:
Пряжка Библейского пояса.

Комментарий:
"Библейский пояс" - собирательное название группы южных штатов США, где
сильны христианские традиции. На звание "Пряжки Библейского пояса"
претендуют и несколько других городов.

Источник:
   1. http://www.findpatent.ru/img_show/230060.html
   2. http://en.wikipedia.org/wiki/Bible_Belt

Автор:
Станислав Попов (Тверь)

Вопрос 22:
Про эту птичку в Англии есть легенда, что она вынимала шипы из ран
Христа, испачкавшись в Его крови. Согласно другой легенде, эта птичка
вовсе не благочестива, а наоборот, склонна к одному из смертных грехов,
поэтому литературный персонаж и получил свое имя. Назовите этого
персонажа, использовав дефис.

Ответ:
Робин-Бобин.

Комментарий:
Речь идет о малиновке, которая по-английски, как известно, - robin.

Источник:
Е. Коути, Н. Харса. Суеверия викторианской Англии.
http://www.flibusta.net/b/269090/read

Автор:
Станислав Попов (Тверь)

Вопрос 23:
В 1795 году из-за угрозы голода премьер-министр Великобритании Уильям
Питт ввел высокий налог на использование ЭТОГО вещества в
непродовольственных целях. С другой стороны, елизаветинские времена
давно уже прошли, и мода начала меняться сама собой. О каком веществе
идет речь?

Ответ:
Крахмал.

Комментарий:
Крахмал использовался для огромных воротничков и манжет, а также для
производства пудры. К началу XIX века эти аксессуары и сами вышли из
моды.

Источник:
С.Дж. Винсент. Анатомия моды. Манеры одеваться от эпохи Возрождения до
наших дней. - М.: Новое литературное обозрение, 2015.

Автор:
Станислав Попов (Тверь)

Вопрос 24:
Ироничный ирландец Дилан Моран задается вопросом: "Как изображение
совершенно обычной ирландской улыбки должно заставить меня СДЕЛАТЬ
ЭТО?". Какие два слова мы заменили на "СДЕЛАТЬ ЭТО"?

Ответ:
Бросить курить.

Зачет:
Не курить, близкие по смыслу ответы из двух слов.

Комментарий:
Моран комментирует изображения всяких ужасов на пачках с сигаретами, а
именно - желтые зубы с пародонтозом. Авторы пакета на этом тоже бросают
курить что попало и шиворот-навыворот и прощаются с вами.

Источник:
Dylan Moran. What It Is (2009).

Автор:
Станислав Попов (Тверь)

Тур:
9 тур. "Питер Пингвинс" (Санкт-Петербург)

Вопрос 1:
(pic: 20150148.jpg)
   Подписи под полученными вами картинками почти совпадают.
Воспроизведите подпись под нижней.

Ответ:
Ни пуха ни пера.

Комментарий:
На первой картинке изображены персонажи "Винни-Пуха" без Винни-Пуха и
персонажи "Питера Пэна" без Питера Пэна ("pen" - перо по-английски). Со
второй картинкой всё ясно. Команда "Питер Пингвинс" желает всем удачи!
   z-checkdb: Peter Pan не имеет отношения к слову "pen".

Автор:
Мария Топаж

Вопрос 2:
С 1990 года ИХ освободили от "школы", но это не значит, что о параграфах
и скобках можно было забыть. Назовите фильм 1993 года, не так давно
вдохновивший ту из НИХ, которая, тем не менее, еще в школе.

Ответ:
"Список Шиндлера".

Комментарий:
Параграфы и скобки входят в программу обязательных фигур в фигурном
катании, которую называют "школой", - она была отменена в 1990 году. На
знаменитую произвольную программу прошлого года Юлию Липницкую и ее
команду вдохновила музыка из "Списка Шиндлера".

Источник:
   1. http://ru.wikipedia.org/wiki/Обязательные_фигуры
   2. http://ru.wikipedia.org/wiki/Липницкая,_Юлия_Вячеславовна

Автор:
Мария Топаж

Вопрос 3:
В книге Евгения Лукина "Алая аура протопарторга" описывается
православно-социалистический город Лыцк. Протопарторг - третий чин
лыцкой иерархии. Первый чин имеет глава города - партиарх. Назовите
второй чин лыцкой иерархии, который имеют, например, замы партиарха.

Ответ:
Митрозамполит.

Источник:
Евгений Лукин. Алая аура протопарторга.
http://www.flibusta.net/b/132417/read

Автор:
Михаил Якубсон

Вопрос 4:
Комментатор матча Аргентина - Нигерия назвал атаку сборной Аргентины с
участием ЭзекьЕля ЛавЕсси ТАКОЙ. Если ИКС не ТАКОЙ - он белый. Мы не
спрашиваем вас, что мы заменили на "ИКС". Назовите слово, которое
заменили на "ТАКОЙ".

Ответ:
Рифмованный.

Комментарий:
В атаке принимали участие Месси и Лавесси.

Источник:
Трансляция матча Аргентина - Нигерия.

Автор:
Татьяна Шарапа

Вопрос 5:
Вступив в Киев со стороны ПечЕрска, полковник петлюровской армии
БолботУн принял решение ДЕЛАТЬ ЭТО. Вальсируя, кавалеру следует обратить
особое внимание на то, чтобы не ДЕЛАТЬ ЭТО. Что мы заменили словами
"ДЕЛАТЬ ЭТО"?

Ответ:
Наступать на Подол.

Зачет:
Наступать на подол.

Комментарий:
Подол - район Киева.

Источник:
М. Булгаков. Белая гвардия. http://www.flibusta.net/b/261423/read

Автор:
Александр Топаж

Вопрос 6:
Недавно под автостоянкой в Лестере английскими археологами было
обнаружено захоронение легендарного короля Ричарда III. Анализ останков
показал, что традиционное представление о степени физического уродства
этого монарха было сильно преувеличенным. Воспроизведите заглавие
заметки об этом на сайте "Русская планета", состоящее из трех слов.

Ответ:
Могила исправила горбатого.

Зачет:
С точностью до порядка слов.

Комментарий:
Оказалось, что горбатость Ричарда III - исключительно художественный
вымысел авторов, например, Шекспира.

Источник:
http://www.rusplt.ru/world/mogila-ispravila-gorbatogo-10192.html

Автор:
Александр Топаж

Вопрос 7:
Главный герой компьютерной игры вооружен разнообразными бластерами и по
долгу своей нелегкой и опасной службы защищает корабль. Название этой
игры можно с некоторой вольностью перевести с английского как "ИКС".
Посетитель сайта geektimes.ru [гиктаймс точка ру] с ником gwer [гвер]
считает, что наличие ИКСА поможет нам защититься от инопланетного
вторжения. Назовите ИКС двумя словами.

Ответ:
Космический мусор.

Комментарий:
"Space cop" действительно можно с некоторой вольностью перевести как
"Космический мусор".

Источник:
   1. http://www.igrystrelyalki.ru/dlya-malchikov/1045-kosmicheskiy-policeyskiy.html
   2. http://geektimes.ru/post/230199/

Автор:
Алексей Палецких

Вопрос 8:
Согласно академическому источнику, ИХ всего 11: а, б, в, ж, и, к, о, с,
у, э, я. Назовите ИХ.

Ответ:
Однобуквенные слова русского языка.

Зачет:
По смыслу.

Комментарий:
Это междометия, местоимения, предлоги, союзы и частицы.

Источник:
Словарь Ожегова.

Автор:
Михаил Дворкин

Вопрос 9:
Прослушайте перевод англоязычной шутки: "Я решил себя заморозить до ИКС
градусов по Цельсию. Жена говорит, что я умру, а я думаю, что всё будет
ИГРЕК". Запишите точно, что мы заменили на ИКС и ИГРЕК.

Ответ:
-273, 0K.

Источник:
http://memegenerator.net/instance/11179723

Автор:
Михаил Дворкин

Вопрос 10:
"Милостивый манифест" Емельяна Пугачёва начинался словами: "Жалую вас и
крестом, и АЛЬФОЮ, и волею, и землею, и угодьями, и лесами, и лугами, и
рыбными ловлями, и всем беспошлинно и безданно...". АЛЬФА явилась
важнейшим атрибутом австрийского триумфа. Что мы заменили на АЛЬФУ?

Ответ:
Борода.

Комментарий:
Примечательно, что борода в этом списке является даже большей ценностью,
чем воля, хотя со времен Петровских указов прошло достаточно времени.
Австрийский триумф - победа Кончиты Вурст на Евровидении-2014.

Источник:
   1. А.Н. Крылов. Мои воспоминания.
http://www.flibusta.net/b/179782/read
   2. http://ru.wikipedia.org/wiki/Кончита_Вурст

Автор:
Александр Топаж

Вопрос 11:
Прослушайте строфу из поэмы Давида Самойлова "Юлий КлОмпус":
   Прозрачнее, чем патер ЧЕстертона,
   Среди белья крахмально выстиранного,
   Лежал он, отрешась от женственного,
   В печальном постиженьи истинного.
   В зачитанном вам отрывке мы заменили подростка на взрослого, а
русского - на англичанина. Восстановите исходный текст и назовите имя
подростка, которого мы заменили.

Ответ:
Варфоломей.

Комментарий:
Замененные слова - "отрок Нестерова". Давид Самойлов образно сравнил
лирического героя с юным героем картины Нестерова "Видение отроку
Варфоломею".

Источник:
Д. Самойлов. Избранное. http://www.flibusta.net/b/354708/read

Автор:
Александр Топаж

Вопрос 12:
(pic: 20150149.jpg)
   На раздаточном материале вам представлен фрагмент фэнтезийного
интернет-комикса. Главный герой на следующей панели говорит, что сделал
ЭТО. "ЭТО" - авторский неологизм. Напишите этот неологизм.

Ответ:
Elfie.

Зачет:
Элфи, Эльфи, Сэльфи, а также любая подобная комбинация слов "селфи" и
"эльф". Незачет: Сэлфи.

Комментарий:
Как можно заметить, главный герой - эльф, который рисует себя.

Источник:
http://www.lfg.co/page/774/

Автор:
Алексей Палецких

Вопрос 13:
По мнению некоторых историков, Венеция была обязана процветанием своей
лагуне, которая слишком глубокая для ПЕРВОЙ и слишком мелкая для
ВТОРОГО. А кем, по мнению ТРЕТЬЕГО, являются ПЕРВАЯ и ВТОРОЙ для нашей
страны?

Ответ:
[Единственными] союзниками.

Комментарий:
Речь идет об армии и флоте. Согласно мнению историков, лагуна являлась
прекрасной естественной крепостью, защищавшей Венецию от морского или
сухопутного вторжения. Согласно известному выражению Александра III,
армия и флот - единственные союзники России.

Источник:
   1. А. Генис. Космополит. http://www.flibusta.net/b/358899/read
   2. http://www.bibliotekar.ru/encSlov/19/19.htm

Автор:
Александр Топаж

Вопрос 14:
В стихотворении Александра Городницкого 2003 года есть строки:
   Всё будет вновь: Ливонская война
   И [ПРОПУСК] и Кавказа.
   Заполните пропуск двумя словами.

Ответ:
Покорение Крыма.

Зачет:
Покоренье Крыма.

Источник:
http://magazines.russ.ru/neva/2003/3/gorod.html

Автор:
Михаил Якубсон

Вопрос 15:
Заметка на новостной ленте Google, посвященная переговорам между Италией
и Россией о смягчении торговых санкций, называлась: "Рим просит помощи у
первого...". Какое слово мы пропустили в этом названии?

Ответ:
Третий.

Комментарий:
Москва - третий Рим.

Источник:
Новостная лента Google.

Автор:
Александр Топаж

Вопрос 16:
В тексте этого вопроса заменена одна буква.
   Выдающийся итальянский певец Франко Бонисолли называл одного своего
коллегу "фламинго", намекая тем самым на отсутствие у того... Ответьте
достаточно точно: чего?

Ответ:
Нот "си" и "до" в диапазоне.

Зачет:
По упоминанию данных нот.

Комментарий:
Речь идет о Пласидо Доминго, имя которого Бонисолли по упомянутой
причине сокращает до "Пламинго".

Источник:
http://izvestia.ru/news/311888

Автор:
Максим Мельцин

Вопрос 17:
Согласно шутке, ОНА хранится в палате мер и весов как эталон рекурсии.
Назовите ЕЕ четырьмя словами.

Ответ:
Палата мер и весов.

Источник:
https://twitter.com/FiztehRadio/status/451645816990892032/

Автор:
Михаил Дворкин

Вопрос 18:
Статья на портале autonews.ru [автоньюс точка ру] рассказывает о
компаниях "Kor_Ecologic", "StreetScooter", "Maasaica" [кор эколОджик,
стритскУтер, маасАика] и других фирмах, использующих современные
инновационные технологии в автомобилестроении. Эти компании названы в
упомянутой статье тем же словом, каким принято называть, например,
Петра, Ивана или Франциска. Воспроизведите это слово.

Ответ:
Первопечатники.

Комментарий:
Эти компании первыми стали печатать детали автомобилей на 3D-принтере.

Источник:
http://www.autonews.ru/autobusiness/news/1796156/

Автор:
Александр Топаж

Вопрос 19:
Как сообщает сайт vesti.ru [вести точка ру], в 2014 году эта организация
произвела ребрендинг и теперь принимает в свои ряды также
афроамериканцев, евреев и гомосексуалов. Неизменным останется лишь
дресс-код. Назовите эту организацию.

Ответ:
Ку-клукс-клан.

Источник:
http://www.vesti.ru/doc.html?id=2115293

Автор:
Даниил Черкашин

Вопрос 20:
В слове по крайней мере три буквы "м" и две буквы "и". Какое слово мы
заменили?

Ответ:
Минимум.

Комментарий:
Слово "минимум" заменено тремя словами "по крайней мере".

Автор:
Татьяна Шарапа

Вопрос 21:
На иллюстрации к июльскому событию ОНА делает фейспалм. Назовите ЕЕ.

Ответ:
Статуя Христа-Искупителя.

Зачет:
Статуя Христа в Рио-де-Жанейро и т.п. по смыслу.

Комментарий:
Бразильцы на чемпионате мира по футболу 2014 года действительно дали
повод.

Источник:
http://www.kaztur.ru/newst/news/?id=6192

Автор:
Даниил Черкашин

Вопрос 22:
Прослушайте цитату из истории с сайта zadolba.li [задолба точка ли]:
"Неудивительно, что встретив - внезапно - марку сливок, которая выходит
в упаковке 200 граммов и 500 граммов, а не 470 или 180, я уже
обрадовалась. Не тут-то было. Ставишь пачку на кухонные весы - и да,
конечно, 200 граммов ровно. С упаковкой. Самих сливок на 15 граммов
меньше". Запишите абсолютно точно тремя словами, как была озаглавлена
эта история.

Ответ:
"И ты, брутто".

Источник:
http://zadolba.li/story/15760

Автор:
Михаил Дворкин

Вопрос 23:
История Европы, можно сказать, началась с АЛЬФА. АЛЬФА находится на
расстоянии 280 километров от Европы. Назовите АЛЬФА или АЛЬФУ.

Ответ:
Галилей.

Зачет:
Галилея.

Комментарий:
Галилей открыл спутник Юпитера - Европу. Галилея находится на расстоянии
примерно 280 километров от Кипра.

Автор:
Александр Топаж

Вопрос 24:
   <раздатка>
   Если бы ________ была у ________,
   Она бы с ________ по жизни ________,
   Но ________ была от ________ ________,
   Поскольку она проживала с ________.
   </раздатка>
   В стихотворении Татьяны Щёголевой мы пропустили восемь слов: шесть
имен собственных, глагол и наречие. Запишите пропущенный глагол и
наречие.

Ответ:
Шагала, вдали.

Комментарий:
   Если бы Гала была у Шагала,
   Она бы с Шагалом по жизни шагала,
   Но Гала была от Шагала вдали,
   Поскольку она проживала с Дали.

Источник:
http://www.orlita.org/вечер-веселой-поэзии/

Автор:
Михаил Дворкин

Тур:
10 тур. "Брейн" (Москва)

Вопрос 1:
Джалли помогала своей хозяйке зарабатывать деньги, принимая участие в
выступлениях в "чудесной" части Парижа XV века. Полли и Молли внесли
вклад в науку на рубеже XX-XXI веков. Шрек прославился после того, как
наконец-то был подстрижен в апреле 2004 года и стал талисманом Новой
Зеландии. Догадавшись, кто такие Джалли, Полли, Молли и Шрек, назовите
число, которое на сегодняшний день их объединяет.

Ответ:
2015.

Комментарий:
Джалли - козочка Эсмеральды из романа "Собор Парижской Богоматери"
Виктора Гюго. Полли и Молли - клонированные овечки, которым был введен
человеческий ген для возможного применения в медицине. Шрек -
мериносовый баран, который прятался от людей шесть лет. 2015 - год козы,
овцы, барана.

Источник:
   1. В. Гюго. Собор Парижской Богоматери.
http://www.flibusta.net/b/151369/read
   2. http://ru.wikipedia.org/wiki/Долли_(овца)
   3. http://ru.wikipedia.org/wiki/Шрек_(овца)
   4. http://ru.wikipedia.org/wiki/Китайский_гороскоп

Автор:
Дарья Коротина (Саранск - Москва)

Вопрос 2:
Внимание, в вопросе есть замены.
   Один школьник, начитавшийся мифов Древней Греции, был абсолютно
уверен, что в слове "ДОЛИНА" нужно писать букву "А" вместо "О". Он
объяснял это тем, что слово "ДОЛИНА" произошло от слова "ДАЛЬ". Назовите
слова, которые мы заменили словами "ДОЛИНА" и "ДАЛЬ", если первое из них
- глагол, а второе - имя собственное.

Ответ:
Хоронить, Харон.

Комментарий:
Школьник убежден, что глагол "ХОРОНИТЬ" произошел от имени перевозчика
душ умерших ХАРОНА, поэтому и пишет слово с ошибкой.

Источник:
Личный опыт автора вопроса.

Автор:
Дарья Коротина (Саранск - Москва)

Вопрос 3:
В одной из статей Reuters за 2012 год сообщается, что в немецком языке
появился неологизм "вульфить", образованный от фамилии Кристиана Вульфа,
являвшегося на тот момент президентом ФРГ. Глагол имеет несколько
значений. Например, уклончиво рассказывать о чем-либо, недоговаривать,
но так, чтобы говорящего нельзя было уличить в откровенной лжи. Это
значение связано с реакцией президента на обвинения в коррупции и
давлении на СМИ. Также в статье упоминается глагол "меркелить", т.е.
сомневаться, быть нерешительным. А какой неологизм используется в
значении "украсть интеллектуальную собственность"?

Ответ:
Сгуттенбергить.

Комментарий:
Неологизм появился после отставки министра обороны ФРГ Карла-Теодора
Гуттенберга из-за обвинений в плагиате при написании диссертации.

Источник:
http://lenta.ru/news/2012/01/13/verbum/

Автор:
Дарья Коротина (Саранск - Москва)

Вопрос 4:
Некоторые цветы обладают ЭТИМ свойством. Например, цветок, который
является шуточным ответом на многие вопросы. Другой цветок именно из-за
ЭТОГО свойства был любимым цветком главной героини произведения XIX
века. Назовите это произведение.

Ответ:
"Дама с камелиями".

Комментарий:
Это свойство - отсутствие запаха. Первый цветок, упомянутый в вопросе, -
гладиолус. "Потому что гладиолус" - популярный шуточный ответ на многие
вопросы. Главная героиня романа Александра Дюма-сына куртизанка
Маргарита Готье была больна чахоткой и любила камелии за то, что они не
пахнут, так как от аромата других цветов из-за болезни у нее кружилась
голова.

Источник:
   1. http://lurkmore.to/Потому_что_гладиолус
   2. А. Дюма-сын. Дама с камелиями.
http://www.flibusta.net/b/112277/read

Автор:
Дарья Коротина (Саранск - Москва)

Вопрос 5:
Представители знати и обладатели значительных денежных сумм традиционно
издревле старались перещеголять соседей демонстрацией своего богатства.
Кто-то строил вычурные дворцы, кто-то разбивал роскошные сады, а
британские аристократы XVIII века нанимали себе в услужение особых
людей. Основные требования к соискателям должности были таковы: носить
только выданную одежду, ни при каких обстоятельствах не покидать
пределов поместья, не общаться ни с кем, кроме хозяина, не стричь волос
и ногтей; пожилой возраст и знание Библии крайне приветствовались.
Заключались такие контракты, как правило, на срок от семи лет, но и
жалование выплачивалось по тем временам более чем достойное. В качестве
кого же нанимались эти люди?

Ответ:
Отшельники.

Комментарий:
Желая показать соседям и друзьям свою набожность или просто поразить
всех эксцентричностью, аристократы нанимали себе декоративных
отшельников. Эти люди жили на прилегавших к поместью землях в палатке
или пещере и... Собственно, это всё, что они делали.

Источник:
http://www.hermitary.com/lore/ornamental_hermits.html

Автор:
Максим Фомин (Москва)

Вопрос 6:
Известны случаи, когда животные используют в своих целях различные
предметы: выдра использует камни, чтобы разбивать ракушки, птица
шалашник использует ракушки и цветы, чтобы привлечь самку. Для чего
речной дельфин использует обезглавленную тушку рыбы?

Ответ:
Мастурбация.

Комментарий:
Зафиксированы случаи, когда самцы речного дельфина использовали
обезглавленные тушки рыб для мастурбации.

Источник:
   1. http://mixstuff.ru/archives/26868
   2. http://www.zoopage.ru/stat.php?idstat=68
   3. http://www.youtube.com/watch?v=ownANj8dKcQ

Автор:
Дарья Беличенко (Москва)

Вопрос 7:
Говорят, что природа отдыхает на детях гениев. Однако прапрапраправнук
одного классика американской литературы также смог добиться всемирной
известности, правда, уже в качестве популярного электронного музыканта.
А чтобы почтить именитого родственника, в качестве сценического
псевдонима музыкант взял имя антагониста самой известного произведения
писателя. Хотя, в отличие от этого кровожадного персонажа, музыкант -
известный вегетарианец. Назовите книгу, о которой идет речь.

Ответ:
"Моби Дик".

Комментарий:
Родители Ричарда Мелвилла Холла очень гордились своим именитым предком:
американским классиком Германом Мелвиллом, автором знаменитого романа
"Моби Дик". Поэтому мальчику от прапрапрапрадедушки досталось не только
второе имя, но и кличка "Моби", с которой тот не расстается и по сей
день.

Источник:
http://en.wikipedia.org/wiki/Moby

Автор:
Антон Лазарев (Москва)

Вопрос 8:
Оводов разделяют на три семейства в соответствии с тем, где на теле
животного развиваются их личинки: подкожные, желудочные и ТАКИЕ. Если
ТАКОЙ овод отложит слишком много личинок на одном животном, то оно
умрет. Причина смерти совпадает с названием романа 2001 года. Назовите
автора этого романа.

Ответ:
[Чак] Паланик.

Комментарий:
Личинки носоглоточого овода развиваются в носоглотке; когда их слишком
много, животное не может дышать и задыхается.

Источник:
   1. http://www.zooeco.com/int/int-nasek23-111.html
   2. http://dic.academic.ru/dic.nsf/dic_biology/3788/
   3. http://ru.wikipedia.org/wiki/Удушье_(роман)

Автор:
Дарья Беличенко (Москва)

Вопрос 9:
Фактопедия утверждает, что эти животные бывают шарообразными и
нитевидными. Одно время на англоязычном сайте отечественной организации
им был посвящен целый научный институт. Назовите писателя, который
поведал об особенной способности одного из представителей этих животных
к обработке драгоценных материалов.

Ответ:
Пушкин.

Комментарий:
Как переводчики РАН, так и составители Фактопедии перепутали бЕлок и
белОк. В произведении А.С. Пушкина "Сказка о царе Салтане" Белка грызла
золотые орехи с чистыми изумрудами.

Источник:
   1. http://factopedia.ru/animal/belki
   2. http://ru.wikipedia.org/wiki/Институт_белка_РАН
   3. А.С. Пушкин. Сказка о царе Салтане.

Автор:
Антон Рогожин (Саранск)

Вопрос 10:
Однажды автор наткнулся на видеоролик, посвященный фитнесу. В
комментарии к нему указывается, что, несмотря на то что ролик
англоязычный, всё ясно без перевода, потому что объясняют... Как?

Ответ:
На пальцах.

Комментарий:
Речь идет о т.н. фингер-фитнесе, т.е. о тренировке пальцев.

Источник:
https://vk.com/topic-41352307_28420063

Автор:
Антон Рогожин (Саранск)

Вопрос 11:
Герой ЭТОГО отечественного постмодернистского романа получил свое имя в
честь русского писателя XX века и российского революционера XX века. Его
имя лишь одной буквой отличается от названия города, руины которого
находятся на территории современного Ирака. Напишите название романа,
если известно, что фамилия героя образована от названия тюркоязычного
народа, составляющего более половины населения региона 16.

Ответ:
"Generation П".

Комментарий:
Герой романа Виктора Пелевина "Generation П" был назван в честь Василия
Аксёнова и Владимира Ленина. Его имя одной буквой отличается от названия
города "Вавилон". Фамилия героя "Татарский" образована от названия
народа "татары", а регион 16 - это Татарстан.

Источник:
   1. В. Пелевин. Generation П.
   2. http://ru.wikipedia.org/wiki/Вавилон
   3. http://ru.wikipedia.org/wiki/Татары

Автор:
Дарья Коротина (Саранск - Москва)

Вопрос 12:
Прослушайте шуточное стихотворение Джеймса Линдона, в котором пропущено
одно слово:
   Плотная пища жен ПРОПУСК
   Только на пользу им шла, не иначе.
   Весили жены, согласно молве,
   Каждая - как предыдущие две.
   Догадавшись, о чьих женах писал Линдон, назовите роман американского
писателя, в котором герои, для того чтобы открыть сейф в банке,
использовали то, что изобрел вышеупомянутый муж.

Ответ:
Код да Винчи.

Комментарий:
В стихотворении пропущено слово "Фибоначчи". В романе Дэна Брауна в
качестве кода доступа к сейфу герои использовали последовательность
Фибоначчи, зашифрованную в анаграмме.

Источник:
   1. http://www.tutoronline.ru/blog/chisla-fibonachchi-ishhem-sekret-mirozdanija
   2. Д. Браун. Код да Винчи. - Глава 42.

Автор:
Дарья Коротина (Саранск - Москва)

Вопрос 13:
В книге психиатра Оливера Сакса описывается случай, когда пациент,
страдавший от постоянных тиков и непроизвольных движений, нашел свое
признание, став ИМ. Один из НИХ сетовал, что его судьба - видеть перед
собой три задницы. Назовите ИХ.

Ответ:
Барабанщики.

Комментарий:
Пациент, страдавший от нервных тиков, стал превосходным джазовым
барабанщиком. К слову сказать, по излечении его импровизационные
способности практически исчезли. А слова о незавидной судьбе принадлежат
барабанщику "Битлз" Ринго Старру.

Источник:
   1. О. Сакс. Человек, который принял жену за шляпу.
http://www.flibusta.net/b/95975/read
   2. http://www.beatl.ru/beatles/citati/ringo-starr.html

Автор:
Антон Рогожин (Саранск)

Вопрос 14:
Казобон, герой романа Умберто Эко "Маятник Фуко", говорил, что каждое
утро за бритьем мы глядим в глаза двойника, который обречен пожизненно
быть ТАКИМ. В бейсболе ТАКОГО питчера называют "southpaw", что значит
"южная лапа". Назовите ТАКОГО одним словом.

Ответ:
Левша.

Комментарий:
Бейсбольные поля обычно проектируются таким образом, что отбивающий
стоит лицом на восток, чтобы полуденное или вечернее солнце не светило
ему в глаза. Это означает, что питчеры-левши бросают своей "южной"
рукой. Казобон говорил об отражении в зеркале.

Источник:
   1. У. Эко. Маятник Фуко.
   2. http://ru.wikipedia.org/wiki/Левша#.D0.9B.D0.B5.D0.B2.D1.88.D0.B0_.C2.ABSouthpaw.C2.BB

Автор:
Дарья Коротина (Саранск - Москва)

Вопрос 15:
Героиня американского сериала "Как я встретил вашу маму" Робин Щербатски
и ее подруга пели, что "два ИХ лучше, чем один". Это была милая детская
песенка, которая, однако, друзьям Робин показалась пошлой. Назовите ИХ,
если известно, что ИХ существует два вида: обыкновенный (или речной) и
канадский.

Ответ:
Бобер.

Комментарий:
Девушки вели программу для детей. У каждой из них был игрушечный бобер.
Они пели песню о дружбе их бобров. Однако в американской табуированной
лексике слово "beaver" (бобер) имеет совсем иное значение - женские
половые органы. Отсюда и соответствующая реакция ее друзей.

Источник:
   1. "How I met your mother", s06e09.
   2. http://www.multitran.ru/c/m.exe?CL=1&s=beaver&l1=1
   3. http://ru.wikipedia.org/wiki/Бобры

Автор:
Дарья Коротина (Саранск - Москва)

Вопрос 16:
В ПЕРВОМ рассказывается о мире, где государство постановило, что 2+2 =
5. Во ВТОРОМ - о двух мирах, в одном из которых на небе висят две луны.
Название ВТОРОГО отличается от названия ПЕРВОГО одним символом. Этот
символ стал кодовым именем вымышленного персонажа известной серии книг и
еще более известной серии фильмов по этим книгам. Назовите этот символ.

Ответ:
Q.

Комментарий:
ПЕРВЫЙ - это роман Оруэлла "1984", ВТОРОЙ - роман Мураками "1Q84" (в
русском переводе - "Тысяча невестьсот восемьдесят четыре"). Q - кодовое
имя Джеффри Бутройта, вымышленного персонажа романов про Джеймса Бонда
британского писателя Яна Флеминга.

Источник:
   1. Дж. Оруэлл. 1984.
   2. Х. Мураками. 1Q84.
   3. http://en.wikipedia.org/wiki/Q_(James_Bond)

Автор:
Дарья Коротина (Саранск - Москва)

Вопрос 17:
ТАКОЙ ОН был куплен для могилы Железной Лошади. "ТАКОЙ ОН" - название
отечественной музыкальной группы и известного романа. Напишите это
название.

Ответ:
"Черный обелиск".

Комментарий:
Железная Лошадь - прозвище проститутки из романа Э.М. Ремарка "Черный
обелиск", для могилы которой был куплен черный обелиск. Рок-группа взяла
свое название в честь вышеупомянутого романа Ремарка.

Источник:
   1. Э.М. Ремарк. Черный обелиск. http://www.flibusta.net/b/326124/read
   2. http://ru.wikipedia.org/wiki/Чёрный_Обелиск_(группа)

Автор:
Дарья Коротина (Саранск - Москва)

Вопрос 18:
В песне Битлз "ТАКОЙ ОН" поется о небогатой девушке, у которой стены в
комнате были обшиты ТАКИМ ИМ, и не было ни одного стула, чтобы присесть
гостю. В 1987 году был написан одноименный зарубежный роман, героине
которого очень нравилась эта песня. Назовите автора романа.

Ответ:
Харуки Мураками.

Комментарий:
Песня Битлз "Норвежский лес" является любимой композицией Наоко,
возлюбленной Ватанабэ, в одноименном романе Х. Мураками.

Источник:
   1. The Beatles, "Norwegian Wood".
   2. Х. Мураками. Норвежский лес.

Автор:
Дарья Коротина (Саранск - Москва)

Вопрос 19:
В 2000 году молодой банкир с Уолл-Стрит надел на встречу со своим
коллегой прозрачное пластиковое пальто, чтобы защитить свою одежду от
ЭТОГО. В 2013 году судебный психиатр из Мериленда надел похожее пальто
на встречу со своим коллегой с той же целью. От чего они хотели защитить
одежду?

Ответ:
От крови.

Комментарий:
Банкир - Патрик Бэйтмен, главный герой фильма "Американский психопат"
(2000), психиатр - Ганнибал Лектер из сериала "Ганнибал" (2013).

Источник:
   1. Фильм "Американский психопат".
   2. Сериал "Ганнибал".

Автор:
Дарья Беличенко (Москва)

Вопрос 20:
По словам известной феминистки Сюзен Энтони, ЭТО сделало больше для
эмансипации женщин, чем всё остальное вместе взятое. В связи с
распространением ЭТОГО в 1890-х годах женщины получили свободу, которой
у них не было раньше. Кроме того, это помогло женщинам избавиться от
корсетов и другой сковывающей одежды. В одном из рассказов Чехова герой
был так поражен, увидев свою возлюбленную с ЭТИМ, что отказался от
намерения жениться на ней. Назовите ЭТО.

Ответ:
Велосипед.

Комментарий:
Велосипеды дали женщинам беспрецедентную свободу передвижения; с
распространением велосипедов связана мода на женские шаровары и одежду,
не сковывающую движения.

Источник:
   1. http://ru.wikipedia.org/wiki/Велосипед#.D0.A1.D0.BE.D1.86.D0.B8.D0.B0.D0.BB.D1.8C.D0.BD.D0.B0.D1.8F_.D1.80.D0.BE.D0.BB.D1.8C
   2. А.П. Чехов. Человек в футляре.
http://www.flibusta.net/b/82117/read

Автор:
Дарья Беличенко (Москва)

Вопрос 21:
Находящаяся в Ватиканском дворце фреска "Афинская школа", одна из самых
известных работ Рафаэля, содержит в себе немало забавных секретов. Так,
некоторым из изображенных античных философов мастер подарил лица своих
знаменитых современников Леонардо да Винчи и Микеланджело, а в качестве
здания школы Рафаэль при помощи архитектора Донато Браманте весьма точно
изобразил первый этаж знаменитого собора Святого Петра. Однако это
сходство посетители Ватикана заметили лишь спустя несколько десятилетий.
Почему?

Ответ:
Собор был построен позже, чем была создана фреска.

Зачет:
По смыслу.

Комментарий:
На момент написания фрески строительство собора еще не началось -
Браманте, которому поручили вести строительство, поделился с Рафаэлем
утвержденными планами строительства. Собор был открыт для посещения
спустя столетие.

Источник:
https://books.google.ru/books?id=MMYHuvhWBH4C&pg=PT476#v=onepage&q&f=false

Автор:
Антон Лазарев (Москва)

Вопрос 22:
В Берлине, современной столице одной из самых развитых европейских
стран, немало интересных зданий, конструкций и памятников. Однако в
первую очередь удивленные взгляды туристов притягивают, как правило,
розовые причудливо изгибающиеся трубы, которые можно увидеть практически
в любой точке города (длина всей системы составляет порядка 60 км!). Вот
такое необычное решение проблемы, неплохо знакомой, в частности, жителям
Восточного Подмосковья, нашли власти города. В чем же предназначение
этих труб?

Ответ:
Это водопроводные трубы.

Зачет:
По смыслу.

Комментарий:
Берлин, по сути, стоит на болоте (вода уже на глубине двух метров), и
нужно эту воду отводить во избежание всяких неприятностей. Но так как
копать там затруднительно, трубы провели по воздуху.

Источник:
http://www.berlinglobal.org/index.php?the-berlin-mystery-of-the-pink-pipes

Автор:
Антон Лазарев (Москва)

Вопрос 23:
В романе Алана Фостера "Призыв к оружию" инопланетяне выбирают в
качестве первого публичного контакта редакцию "Вашингтон Пост". Главному
редактору издания они заявляют, что он не единственный, с кем пришельцы
собираются контактировать. Тот сначала пугается, но вскоре
успокаивается, говоря, что на продажах его издания это не скажется. Не
бойтесь выбрать и ответьте, что, согласно старому анекдоту, отсутствует
во втором издании, с которым собрались контактировать пришельцы.

Ответ:
Правда.

Зачет:
Известия.

Комментарий:
Второе издание - "Известия". Советская газета вряд ли могла повлиять на
продажи экземпляров "Вашингтон Пост". Согласно анекдоту, в "Известиях"
нет правды, а в "Правде" - известий. Согласно другому варианту, в
"Известиях" нет известий, а в "Правде" - правды. Так что ошибиться в
выборе невозможно.

Источник:
   1. А. Фостер. Призыв к оружию. http://www.flibusta.net/b/81406/read
   2. Анекдот.

Автор:
Антон Рогожин (Саранск)

Вопрос 24:
В толковых словарях у этого слова есть несколько значений. Чаще всего
оно употребляется в разговорной речи для обозначения затяжного (иногда
бессмысленного) занятия, связанного с задержками и хлопотами. А вот для
музыкантов, играющих на струнных инструментах, это слово обычно означает
нечто более материальное - то, с чем им приходится иметь дело каждый
день. Назовите это слово.

Ответ:
Канитель.

Комментарий:
Тонкую металлическую нить, предназначенную для вышивки или, в частности,
для намотки на басовые струны инструментов, называют красивым испанским
словом "канитель"; те, кто пользовались в рукоделии подобными нитями,
наверняка поймут, почему выражение "тянуть канитель" приняло в русском
языке такое негативное значение.

Источник:
http://ru.wikipedia.org/wiki/Канитель

Автор:
Максим Фомин (Москва)

Тур:
11 тур. "Андреевские заразы" (Москва)

Инфо:
Команда благодарит за тестирование и ценные замечания Тимура Бокова,
Кирилла Железнова, Артема Митрофанова, Николая Некрылова, Евгения
Перпера, Александра Попкова, Анастасию Стебалину, Алексея Тишкина и Анну
Павлову, Олега Фею, Сергея Шорина, Вадима Штанникова, а также Игоря
Тюнькина за самое деятельное участие в работе над пакетом.

Вопрос 1:
Персонаж рассказа Чехова, неудачливый писатель, говорит о себе, что
полсотни марок он посеял на ПЕРВОЙ и еще сотню утопил во ВТОРОЙ.
Назовите ПЕРВУЮ и ВТОРУЮ, которые различаются одной буквой.

Ответ:
Нива, Нева.

Комментарий:
Здесь "Нива" и "Нева" - названия литературных журналов. Писатель
отправлял рукописи со своими творениями в различные журналы, но они
нигде не были приняты. На почтовые марки пришлось потратить заметную
сумму.

Источник:
А.П. Чехов. Мой юбилей. http://www.flibusta.net/b/10613/read

Автор:
Андрей Кудрявцев

Вопрос 2:
По словам Наполеона Бонапарта, высшее искусство управления состоит в
том, чтобы не дать подчиненным совершить некое действие в своей
должности. Главный герой произведений 2009 и 1922 годов совершает
противоположное действие. Назовите этого героя.

Ответ:
[Бенджамин] Баттон.

Комментарий:
Совершить некое действие - состариться. Возможно, Наполеон имел в виду,
что подчиненных следует своевременно повышать, но, по мнению Тарле,
получалось иначе - у него на службе долго не жили. Обратный порядок дат
в вопросе призван служить подсказкой - Баттон тоже молодеет.

Источник:
   1. Е.В. Тарле. Наполеон
http://www.museum.ru/1812/library/tarle/napoleon_win.txt
   2. http://ru.wikiquote.org/wiki/Наполеон_I_Бонапарт
   3. http://ru.wikipedia.org/wiki/Загадочная_история_Бенджамина_Баттона_(фильм)
   4. http://ru.wikipedia.org/wiki/Загадочная_история_Бенджамина_Баттона_(рассказ)

Автор:
Андрей Кудрявцев, Екатерина Лобкова

Вопрос 3:
Заметка на сайте zadolba.li рассказывает о том, что социопатия - уже не
только удел отдельных людей, но и повод для общения. Неудивительно, что
в заголовке из социопатии пропала ОНА. Напишите ЕЕ.

Ответ:
Я.

Комментарий:
Заметка называется "У нас тут социопати".

Источник:
http://zadolba.li/story/16196

Автор:
Андрей Волыхов, Игорь Тюнькин

Вопрос 4:
Учитель автора вопроса почти весь урок ДЕРЖАЛ ФОРМУ, так ничего и не
заподозрив. Какие три слова мы заменили в вопросе?

Ответ:
Сидел на кнопке.

Источник:
ЛОАВ.

Автор:
Андрей Кудрявцев

Вопрос 5:
В меню столовой МГУ автор вопроса обнаружил дарницкий хлеб, пшеничный
хлеб и ТАКОЙ хлеб. Чаще можно услышать, что ТАКИМ бывает другой продукт.
Какое слово мы заменили словом "ТАКОЙ"?

Ответ:
Бесплатный.

Комментарий:
А вот бесплатного сыра в столовой не оказалось.

Источник:
ЛОАВ.

Автор:
Андрей Волыхов

Вопрос 6:
Автор вопроса предположил, что в столовой МГУ для продуктов применяется
ОНА. Назовите ЕЕ двумя словами.

Ответ:
Китайская классификация.

Зачет:
Классификация Борхеса, борхесовская классификация.

Комментарий:
Там же были обнаружены, например, "разовый чай" и "зеленый чай". В одном
эссе Борхеса приведена анекдотическая классификация животных, взятая
якобы из китайской энциклопедии.

Источник:
   1. ЛОАВ.
   2. http://ru.wikipedia.org/wiki/Классификация_животных_(Борхес)

Автор:
Андрей Волыхов

Вопрос 7:
Хозяйка из рассказа Чехова не успевает приготовить праздничный обед и
потому ДЕЛАЕТ ЭТО. Многим из нас необходимость регулярно ДЕЛАТЬ ЭТО
приносила удовольствие. Что именно делать?

Ответ:
Переводить часы назад.

Зачет:
По смыслу с обязательным указанием на перевод назад.

Комментарий:
Различные персонажи, каждый по своей причине, хотят приблизить момент
наступления Нового года. Хозяйка же не успевает приготовить праздничный
обед к полуночи и переводит назад минутную стрелку. Когда часы
переводятся назад, можно спать дольше.

Источник:
А.П. Чехов. Мошенники поневоле. http://www.flibusta.net/b/10615/read

Автор:
Андрей Кудрявцев

Вопрос 8:
Зал славы ТРЕТЬЕГО создан в 1983 году, но первые члены в нем появились
лишь в 1986 году. Автор вопроса предположил, что в первые три года
оргкомитет был слишком увлечен ПЕРВЫМ и ВТОРЫМИ. Назовите ТРЕТИЙ.

Ответ:
Рок-н-ролл.

Комментарий:
И слово "члены" в вопросе не случайно. Видимо, в первые три года фонд
уделял больше внимания сексу и наркотикам, чем поиску прибежища для
музея.

Источник:
   1. http://en.wikipedia.org/wiki/Rock_and_Roll_Hall_of_Fame
   2. Фантазии автора вопроса.

Автор:
Павел Казначеев

Вопрос 9:
Государство Силенд - "морская земля" - расположено недалеко от побережья
Великобритании на платформе, с которой планировали транслировать
независимое радио. Представители какого ресурса в 2007 году пытались
купить Силенд?

Ответ:
[The] Pirate Bay.

Комментарий:
Ресурс "The Pirate Bay", видимо, захотел создать собственную пиратскую
бухту в независимом государстве, чтобы разместить серверы.

Источник:
http://en.wikipedia.org/wiki/Principality_of_Sealand

Автор:
Андрей Кудрявцев, Игорь Тюнькин

Вопрос 10:
Недоброжелатели положили коньки одного фигуриста на НЕЕ, чтобы тот
провалился. В 1837 году был заложен памятник на месте другой ЕЕ, которую
возглавлял... Кто?

Ответ:
[Николай Николаевич] Раевский.

Комментарий:
Разогретые на батарее коньки глубоко ушли в лед, хотя совсем под лед
фигурист не провалился. По случаю двадцатипятилетия победы над
Наполеоном на Бородинском поле провели торжества, а на Курганной высоте
установили первый памятник.

Источник:
   1. И. Роднина. Слеза чемпионки. http://www.flibusta.net/b/351532/read
   2. http://www.borodinoru.ru/Pamyatniki/pamyat1812/pam52.htm

Автор:
Екатерина Лобкова

Вопрос 11:
Форму чего имеют точки над "ё" в логотипе фирмы "Лимузёнок"?

Ответ:
Обручальные кольца.

Комментарий:
Фирма предоставляет лимузины для торжественных событий.

Источник:
Субботние наблюдения автора вопроса.

Автор:
Екатерина Лобкова

Вопрос 12:
Прослушайте рекламный слоган книги: "Больше, чем [два слова пропущено]
страсти". Заполните пропуск.

Ответ:
"... пятьдесят оттенков...".

Источник:
Виденная автором вопроса в вагоне метро реклама какой-то книги.

Автор:
Андрей Волыхов

Вопрос 13:
Прослушайте цитату из песни Михаила Щербакова:
   Вчера я видел сеть узкоколейных линий,
   по каким ту глину поезда везли...
   Каким словом заканчивается следующая строка песни?

Ответ:
Алюминий.

Комментарий:
   ... туда, где из нее, по слухам, алюминий
   извлекают нынче. Веришь ли?
   Вот миномёт. Не веришь - застрели. :-)
   Основной источник алюминия - боксит - является глиноподобным
минералом. Да и в любой глине основа - алюмосиликаты.

Источник:
М. Щербаков. Инициалы.
http://www.blackalpinist.com/scherbakov/htmtexts/1996/inicialy.html

Автор:
Андрей Волыхов

Вопрос 14:
   <раздатка>
   Википедия утверждает, что слухи о монархизме Гитлера могли быть
вызваны британской пропагандистской песней, написанной в 1939 году.
   </раздатка>
   В одном из слов в раздаточном материале мы заменили одну букву.
Напишите исходное слово.

Ответ:
Монорхизме.

Комментарий:
Монорхизм - отсутствие одного яичка. Песня называлась "Hitler Has Only
Got One Ball".

Источник:
   1. http://en.wikipedia.org/wiki/Adolf_Hitler's_possible_monorchism
   2. http://en.wikipedia.org/wiki/Hitler_Has_Only_Got_One_Ball

Автор:
Андрей Волыхов, Екатерина Лобкова

Вопрос 15:
[Ведущему: указать, что перед пропуском и после него есть запятые.]
   Заполните двумя словами пропуск в цитате из песни Виталия
Калашникова:
   Потерпим немного в своем ожиданье итога,
   Листва облетает, и смысл по углам растасован,
   И воздух подернут тончайшей, [ПРОПУСК],
   И люди, как в бомбоубежище, прячутся в слово.

Ответ:
"... воздушной тревогой...".

Комментарий:
По-видимому, здесь "воздушная тревога" - не устойчивое словосочетание.

Источник:
В. Калашников. Геннадию. http://www.bards.ru/archives/part.php?id=40298

Автор:
Андрей Волыхов

Вопрос 16:
(pic: 20150150.jpg)
   Какое прилагательное дизайнер студии Артемия Лебедева использовал при
описании этого головного убора?

Ответ:
Каракулевая.

Комментарий:
Из каракулей.

Источник:
http://www.artlebedev.ru/everything/brain/2014/09/18/

Автор:
Павел Казначеев

Вопрос 17:
Героиня произведения Фаулза называет своего разговорившегося мужа
кресельным социалистом и говорит, что он мог бы переманить всех в
радиусе достижения. В одном из слов предыдущего предложения мы заменили
одну букву. Напишите этот неологизм в изначальном виде.

Ответ:
Перемаоить.

Комментарий:
Мао, известный в англоязычных источниках как "chairman Mao", тоже в
каком-то смысле кресельный социалист.

Источник:
Дж. Фаулз. Туча. http://www.flibusta.net/b/349571/read

Автор:
Павел Казначеев

Вопрос 18:
В своей автобиографии Ирина Роднина вспоминает, что во время первого
заграничного дебюта вместе с коллегами по сборной уже после первой
тренировки побежала в магазин за НИМ. Роднина называет ЕГО одним словом,
а вы назовите одним или, так и быть, двумя.

Ответ:
[Нижнее] белье.

Комментарий:
Девушкам было стыдно демонстрировать в общей раздевалке советское белье.
"Своя автобиография" и "первый дебют" - плеоназмы, как и "нижнее белье".

Источник:
   1. И. Роднина. Слеза чемпионки. http://www.flibusta.net/b/351532/read
   2. http://poetique.academic.ru/407/

Автор:
Екатерина Лобкова, Игорь Тюнькин

Вопрос 19:
Авторы заметки на сайте housefan.ru [хаусфан точка ру] пишут, что
некоторые врачи ДЕЛАЮТ ЭТО, когда в сериале "Доктор Хаус" пользуются
дефибриллятором. Делают что?

Ответ:
Хватаются за сердце.

Комментарий:
В реальности дефибриллятор не может "запустить" остановившееся сердце.
Его задача - остановить фибрилляцию, т.е. нескоординированное сокращение
отдельных групп мышечных волокон сердечной мышцы, и дать возможность
естественному водителю ритма вернуть себе контроль над сокращениями.

Источник:
   1. http://www.housefan.ru/news-7mistakes.html
   2. http://ru.wikipedia.org/wiki/Фибрилляция_сердца
   3. http://ru.wikipedia.org/wiki/Дефибриллятор
   4. Консультация Андрея Азова.

Автор:
Андрей Кудрявцев

Вопрос 20:
Узнав, что по ее фразе был написан вопрос ЧГК, Мария Логачёва упомянула
двузначное число. Какое?

Ответ:
34.

Комментарий:
Аналог "правила 34" - и про это есть вопрос! Кстати, удивительно, что
вопросов про "правило 34" в Базе не нашлось.

Источник:
Переписка в Скайпе Марии Логачёвой и Виктории Штратниковой,
подсмотренная автором вопроса.

Автор:
Андрей Волыхов

Вопрос 21:
Размышляя о гендерных различиях в сходных профессиях, автор вопроса
уподобил ИХ верхним ножкам X-хромосомы. Назовите ИХ двумя словами.

Ответ:
Троллейбусные штанги.

Зачет:
Штанги троллейбуса. Зачет с негодованием: Рога троллейбуса и прочие
подобные ответы.

Комментарий:
Женщины - водители троллейбусов встречаются намного чаще, чем женщины -
водители автобусов.

Источник:
   1. Палец автора вопроса.
   2. http://www.the-village.ru/village/city/asking-question/131167-avtobus-trolleybus-zhenschiny

Автор:
Андрей Волыхов

Вопрос 22:
В одном романе цитируется надпись на плакате: "Приветствуем участников
DCLXV [шестьсот шестьдесят пятого] традиционного съезда феминисток,
работающих в сферах геронтологии, косметологии, ботаники и межличностных
отношений". Слово, которым в романе назван этот конгресс, вероятно,
образовано от названия дня недели. Напишите это слово.

Ответ:
ШАбаш.

Комментарий:
На съезд слетаются ведьмы. Согласно этимологическому словарю Фасмера,
слова "шАбаш" и "шабАш" связаны с шаббатом через немецкий и польский
языки.

Источник:
   1. С. Лукьяненко. Шестой Дозор.
   2. http://dic.academic.ru/dic.nsf/vasmer/51196/

Автор:
Екатерина Лобкова

Вопрос 23:
(pic: 20150151.jpg)
   Перед вами вариант покроя юбки, названный в ЕЕ честь. В порту какого
города в 1913 году установили памятник ЕЙ?

Ответ:
Копенгаген.

Комментарий:
Покрой "mermaid" получил свое название из-за сходства с фигурой Ариэль.
А в Копенгагене стоит памятник андерсеновской Русалочке.

Источник:
   1. https://www.google.ru/search?q=mermaid+skirt&tbm=isch
   2. http://ru.wikipedia.org/wiki/Русалочка_(статуя)

Автор:
Екатерина Лобкова

Вопрос 24:
   <раздатка>
   Т'Пау
   Т'Пол
   Т'Принг
   </раздатка>
   Как известно, раса вулканцев из вселенной "Звездного пути" высоко
ценит логику, интеллект и контроль над эмоциями. Перед вами несколько
вулканских имен. Какое слово мы пропустили в тексте этого вопроса?

Ответ:
Женских.

Зачет:
Женщин, героинь.

Комментарий:
Для пользователя Рунета забавно, что имена вулканок нередко начинаются с
буквосочетания "тп". Но, конечно, это случайное совпадение, поскольку из
экранного канона известны еще как минимум вулканки СаАвик и ВалЕрис.
Живите долго и процветайте!

Источник:
Вселенная Звездного Пути (сериалы "Star Trek: The Original Series" и
"Enterprise", полнометражные фильмы "Звездный путь: Гнев Хана",
"Звездный путь: В поисках Спока" и "Звездный путь: Неоткрытая страна").

Автор:
Екатерина Лобкова

Тур:
12 тур. "Любовь" (Калуга)

Вопрос 1:
Средневековая легенда рассказывает о Франческе и Паоло, которые были
застигнуты и убиты мужем Франчески в самом начале любовных отношений.
Произведение, по одной из версий навеянное этим сюжетом, было впервые
показано в 1889 году. Назовите автора этого произведения.

Ответ:
[Огюст] Роден.

Комментарий:
Существует мнение, что на скульптуре Родена "Поцелуй" пара не касается
друг друга губами. И это намекает, что они были убиты, не совершив
греха.

Источник:
http://ru.wikipedia.org/wiki/Поцелуй_(Роден)

Автор:
???

Вопрос 2:
В одном из эпизодов сериала "Симпсоны" у Мардж в результате проклятия
цыганки появилась КРАСНАЯ ШАПОЧКА. Какие слова мы заменили на "КРАСНАЯ
ШАПОЧКА"?

Ответ:
Синяя борода.

Комментарий:
Мардж Симпсон, как известно, - обладательница огромной копны синих
волос. "Синяя Борода", как и "Красная Шапочка", - название сказки Шарля
Перро.

Автор:
Даниил Марченко (Калуга)

Вопрос 3:
В вопросе есть замены.
   По шутке автора вопроса, румынская народная мудрость звучит так:
"Если ты не нашел в своем кармане ЕГО АЛЬФУ - значит, ты живешь не в
Румынии". На портале psifactum.ru [псифактум точка ру] утверждается, что
в своей деятельности ОНИ стараются дотронуться до АЛЬФЫ, чтобы
воздействовать на тактильный канал восприятия. Какие слова мы заменили
на "ОНИ" и "АЛЬФА"?

Ответ:
Цыгане, рука.

Источник:
   1. http://irrichidubili.livejournal.com/14914.html
   2. http://psifactum.ru/manipulation/521-manipulation.html

Автор:
???

Вопрос 4:
В сериале "Сыны анархии" крутой сюжет перемежается с тягучей семейной
драмой. В обзоре на этот сериал отмечено, что от городка Чарминг, где
происходит действие, совсем недалеко до другого населенного пункта.
Назовите его.

Ответ:
Санта-Барбара.

Источник:
Эфир канала "2x2", 02.11.2014 г., 23:35.

Автор:
???

Вопрос 5:
Как ни странно, ИКС не упоминается в диете для набора мышечной массы. В
1912 году начал издаваться журнал, посвященный силовым видам спорта, под
названием "ИКС". Какое слово мы заменили на ИКС?

Ответ:
Геркулес.

Источник:
http://sportswiki.ru/

Автор:
???

Вопрос 6:
На Урале, чтобы облагородить цвет дымчатого кварца, крупные кристаллы
ЖАРИЛИ НА МАСЛЕ. Рассказывают, что Державина в младенчестве ЖАРИЛИ НА
МАСЛЕ, чтобы излечить от тяжелой болезни. Какие три слова мы заменили на
слова "ЖАРИЛИ НА МАСЛЕ"?

Ответ:
Запекали в тесте.

Зачет:
Запекали в печи.

Комментарий:
Чтобы не повредить, кристаллы обволакивали тестом и томили в печи. В
старину младенцев тоже оборачивали тестом и сажали в печь для лечения.

Источник:
   1. http://www.imperis.ru/art/katalog-kamnej-samocvetov/340/
   2. http://www.vorojea.info/vega/novikov/novikovartikles/vediratmir.htm

Автор:
Денис Карабанов (Калуга)

Вопрос 7:
Остроумный Шеридан Саймов выпустил необычную книгу под названием "О чем
думают все мужчины, помимо секса". В статье Википедии, рассказывающей об
этой книге, упоминаются также уроженец Киева и уроженец Лос-Анджелеса.
Назовите их фамилии.

Ответ:
Малевич, Кейдж.

Комментарий:
Книга состояла из пустых страниц, что, по мнению автора, должно отражать
его "шокирующий вывод" о том, что, помимо секса, мужчины больше не
думают ни о чем. Книга с пустыми страницами так же концептуальна, как
"Черный квадрат" Малевича и музыкальная пьеса Кейджа "4 минуты 33
секунды".

Источник:
http://ru.wikipedia.org/wiki/Книга_Ничто

Автор:
???

Вопрос 8:
В романе Стивена Кинга "11/22/63" упоминаются такие события, как
покушение на Кеннеди, убийство Мартина Лютера Кинга в Чикаго и концерт
мира воссоединившихся "Битлз". В одном из эпизодов главный герой слышит,
как за окном ПРОИЗОШЛО ЭТО. Ответьте, что произошло, двумя словами,
начинающимися на одну и ту же букву.

Ответ:
Грянул гром.

Комментарий:
"И грянул гром" - знаменитый рассказ Рэя Брэдбери, в котором
иллюстрируется так называемый эффект бабочки. В романе "11/22/63"
вернувшийся в прошлое главный герой предотвратил убийство Джона Кеннеди,
и это повлекло за собой глобальные изменения истории.

Автор:
Денис Карабанов (Калуга)

Вопрос 9:
В России приобретают популярность экологически чистые источники энергии.
По словам Веры Писаревой, с электричеством в России "ПРОИСХОДИТ ЭТО". А
с чем ПРОИСХОДИТ ЭТО в начале стихотворения 1858 года?

Ответ:
С травкой.

Комментарий:
Электричество "зеленеет".

Источник:
   1. http://www.ogoniok.com/4898/43/
   2. http://www.a-pesni.org/popular20/last-ples.htm

Автор:
Алексей Пономарев (Калуга)

Вопрос 10:
   Он был монтером Ваней, но... в духе парижан,
   Себе присвоил званье: "электротехник Жан...".
   Этим эпиграфом Максим Кравчинский предваряет свою книгу, посвященную
истории жанра. Назовите этот жанр, используя заимствованное слово.

Ответ:
[Русский] шансон.

Комментарий:
Термин "русский шансон" в начале 1990-х стал своего рода эвфемизмом
"блатной песни". Текст Маяковского вполне подходит для песни в стиле
"русский шансон".

Источник:
М. Кравчинский. История русского шансона.
http://www.flibusta.net/b/367967/read

Автор:
???

Вопрос 11:
Кандидат исторических наук Артем Ковалев участвовал в конференции,
посвященной Калужской губернии. Свое сообщение об этом в Живом Журнале
он самоиронично озаглавил "Йа [я] ПРОПУСК". Восстановите пропуск,
добавив одну букву к интернет-мему.

Ответ:
Краеведко.

Комментарий:
Историки к краеведению относятся снисходительно.

Источник:
http://a-v-kovalev.livejournal.com/20293.html

Автор:
Алексей Пономарев (Калуга), по идее Павла Столярова (Калуга)

Вопрос 12:
На предприятии, где работает автор вопроса, в период 90-х годов время от
времени заказывали рулон красной ткани, а в деревообрабатывающем цехе
изготавливали предмет, не имеющий прямого отношения к продукции завода.
Назовите этот предмет.

Ответ:
Гроб.

Источник:
ЛОАВ.

Автор:
???

Вопрос 13:
Друг автора вопроса, получив на день рождения современное устройство,
пошутил, что слишком гетеросексуален для того, чтобы пользоваться ИМ.
Существует гипотеза, что слово "ОН" произошло от имени Анри-Луи
Жаке-Дроза. Назовите ЕГО.

Ответ:
Андроид.

Источник:
   1. ЛОАВ.
   2. http://ru.wikipedia.org/wiki/Андроид

Автор:
???

Вопрос 14:
На одном из этапов создания "ИХ" звукооператор Ари Пулкиннен принес ящик
пива, и все нахрюкались. Назовите ИХ двумя словами, начинающимися на
соседние буквы алфавита.

Ответ:
"Angry Birds".

Источник:
http://www.rbcdaily.ru/autonews/562949980422921

Автор:
???

Вопрос 15:
В Колумбии были найдены древние статуи с раздутыми щеками, которые
подтверждают, что еще инки знали свойства АЛЬФЫ. В каком виде спорта
АЛЬФА была отменена в 2009 году?

Ответ:
Дзюдо.

Комментарий:
АЛЬФА - кока. В первом случае - растение, листья которого жуют. Во
втором случае - оценка в дзюдо.

Источник:
   1. http://incamusic.narod.ru/latinorama/coca/coca.htm
   2. http://ru.wikipedia.org/wiki/Дзюдо

Автор:
Алексей Пономарев (Калуга)

Вопрос 16:
Автор вопроса назвал битвы коллег по работе из-за очереди к
микроволновке ИМИ. "ОНИ" - название произведения 2012 года. Назовите ИХ
двумя словами.

Ответ:
Голодные игры.

Источник:
   1. ЛОАВ.
   2. http://ru.wikipedia.org/wiki/Голодные_игры_(фильм)

Автор:
???

Вопрос 17:
[Ведущему: после оглашения ответа обязательно прочитать комментарий!]
   Для выполнения удара топ-спин справа в настольном теннисе следует
отвести правую руку с ракеткой назад, повернуть плечи, а ноги согнуть в
коленях. Рассказывая об этом, один тренер упомянул уроженца города
ЭлевфЕры. Назовите ЕГО.

Ответ:
МИрон.

Комментарий:
Личный опыт автора вопроса показывает, что поза при замахе ударом
топ-спин напоминает позу "Дискобола" в скульптуре МИрона.

Источник:
   1. http://ru.wikipedia.org/wiki/Топ-спин
   2. http://ru.wikipedia.org/wiki/Мирон_из_Елевфер

Автор:
Денис Карабанов (Калуга)

Вопрос 18:
В настольном теннисе перед ответственной игрой резиновую накладку
ракетки отделяют от деревянной основы и заново наклеивают специальным
клеем, что увеличивает скорость полета мяча, но сокращает срок службы
накладки. Автор вопроса сравнил такую накладку с НЕЙ. "ОНА" - название
литературного произведения. Назовите ЕЕ.

Ответ:
Шагреневая кожа.

Комментарий:
В романе исполнение желаний приводило к сжиманию шагреневой кожи и
сокращению жизни ее обладателя.

Источник:
   1. ЛОАВ.
   2. http://ru.wikipedia.org/wiki/Шагреневая_кожа

Автор:
Денис Карабанов (Калуга)

Вопрос 19:
"Осенью 1847 года в Риме Герцен участвует в народных шествиях,
манифестациях, посещает революционные клубы...". Прослушанный вами
отрывок из биографии написавшего "Кто виноват?" можно назвать четырьмя
словами. Воспроизведите эти четыре слова, которые вы сегодня уже
слышали.

Ответ:
Личный опыт автора вопроса.

Источник:
http://www.c-cafe.ru/days/bio/6/036.php

Автор:
Денис Карабанов (Калуга)

Вопрос 20:
Пациент психоаналитика Шернера видел сон, где два ряда белокурых
мальчиков с нежным цветом лица готовились к борьбе. Далее во сне
произошло действие, которое обычно осуществляют в кабинете... Какого
врача?

Ответ:
Стоматолога.

Зачет:
Дантиста; зубного врача.

Комментарий:
Действие - вытаскивание зуба.

Источник:
http://www.magister.msk.ru/library/philos/freud/freud206.htm

Автор:
Денис Карабанов (Калуга)

Вопрос 21:
Когда в 1968 году среди прочих подразделений в Чехословакию вошла 7-я
воздушно-десантная дивизия, местные жители подумали, что мировая
общественность пришла им на помощь. Причиной ошибки были ОНИ. Кстати,
"ОНИ" - название музыкального коллектива. Назовите ИХ.

Ответ:
Голубые береты.

Комментарий:
Тогда советские десантники только перешли с малиновых на голубые береты,
и местные жители спутали их с войсками ООН.

Источник:
http://ru.wikipedia.org/wiki/Голубой_берет

Автор:
Алексей Пономарев (Калуга)

Вопрос 22:
В историческом словаре галлицизмов Епишкина НЕКИМ СЛОВОМ определяют
человека, применяющего кулачную расправу. Еще одно значение -
исследователь творчества поэта. Назовите этого поэта.

Ответ:
Данте [Алигьери].

Комментарий:
Это слово - "дантист", т.е. дающий в зубы.

Источник:
http://gallicismes.academic.ru/12285/

Автор:
???

Вопрос 23:
Согласно шутке команды КВН "Саботаж", среди НИХ 1% имеют юридическое
образование, 27% - нетрадиционной сексуальной ориентации. Мы не просим
назвать ИХ. Назовите двумя словами продукт, который упоминается в этой
шутке в связи с остальной частью.

Ответ:
Хозяйственное мыло.

Комментарий:
ОНИ - футбольные судьи. Согласно шутке: остальные 72% составляют кусок
хозяйственного мыла. 72% - это тот процент содержания жирных кислот,
который не должен быть превышен по ГОСТу при изготовлении хозяйственного
мыла.

Источник:
   1. http://www.ukrgame.net/forum/topic/5126-shutki-o-futbole-iz-kvn/
   2. http://ru.wikipedia.org/wiki/Хозяйственное_мыло

Автор:
???

Вопрос 24:
Причинами, по которым в России XIX века развивались ТАКИЕ промыслы, были
неурожаи, а также недостаточная обеспеченность крестьян землей. По
запросу "ТАКОЕ место" Википедия перенаправляет на статью, озаглавленную
словом французского происхождения. Что это за слово?

Ответ:
Туалет.

Зачет:
Сортир.

Комментарий:
ТАКОЙ - отхожий. Крестьяне уходили на промыслы в другие земли.

Источник:
   1. http://ru.wikipedia.org/wiki/Отхожий_промысел
   2. http://ru.wikipedia.org/wiki/Отхожее_место

Автор:
???

Тур:
13 тур. "Львята отца Симбы" (Северодвинск - Санкт-Петербург - Коломна)

Вопрос 1:
(pic: 20150152.jpg)
   Уважаемые игроки, у вас есть минута, чтобы раскрутить этот вопрос!
   Перед вами четыре не совсем обычных ИХ. Мы не просим перечислить ИХ
все. Назовите абсолютно точно то, что чаще всего обозначается этими
четырьмя.

Ответ:
[Игра] "Что? Где? Когда?".

Комментарий:
(pic: 20150153.jpg)
   ОНИ - латинские буквы. Изображены буквы C, H, G, K. Авторы рисунка
раскрутили буквы относительно вертикальной оси пересечения с первым
пикселом.

Источник:
http://melotar.blogspot.ru/2011/05/3d.html

Автор:
Елена Ганькова, Анна Дуплищева (Северодвинск)

Вопрос 2:
Внимание, в вопросе ИКС является заменой.
   Вам наверняка известен ресторан быстрого питания, в названии которого
ИКС - это сокращение от фамилии владельца, Гершеля Пинкуса Йорахама. А
кто слышал про другой - лучший в городе ресторан, в названии которого
ИКС соседствует с фамилией владельца?! Несмотря на стихийные различия,
оба ресторатора не без изъяна: они частенько бывают неприветливы и
раздражительны по отношению к окружающим. Какое слово мы заменили на
ИКС?

Ответ:
Красти.

Комментарий:
Гершель Пинкус Йорахам Крастовски - клоун Красти из мультсериала
"Симпсоны" - является владельцем "Красти Бургера". В мультсериале "Губка
Боб Квадратные Штаны" ресторан мистера Юджина Крабса носит название
"Красти Крабс". Слово "Красти" (Krusty) созвучно английскому "crusty" -
резкий, неприветливый, раздражительный (по отношению к человеку). "А кто
слышал" и "не без изъяна" - отсылки к песенке в заставке мультсериала
"Губка Боб квадратные штаны".

Источник:
   1. http://ru.wikipedia.org/wiki/Клоун_Красти
   2. http://ru.wikipedia.org/wiki/Мир_мультсериала_%C2%ABГубка_Боб_Квадратные_Штаны%C2%BB#.D0.9A.D1.80.D0.B0.D1.81.D1.82.D0.B8_.D0.9A.D1.80.D0.B0.D0.B1.D1.81

Автор:
Елена Ганькова, Анна Дуплищева (Северодвинск)

Вопрос 3:
В одном из сюжетов "Даешь, молодежь!" Ржавый стал главой местной банды
хулиганов и активно подражает оскароносному персонажу. Один из хулиганов
вспоминает, как новый предводитель когда-то сходил в туалет под дверь
ненавистному участковому. В ответ на это Ржавый цитирует известное
выражение, заменяя одно слово на "теплым". Воспроизведите первое слово
этой фразы.

Ответ:
Месть.

Комментарий:
Стал строить из себя крестного отца. Диалог звучал примерно так:
   - Я сделаю ему предложение, от которого он не сможет отказаться!
   - Что, опять под дверь наделаешь?
   - Месть - это блюдо, которое подается теплым!
   Марлон Брандо и Роберт Де Ниро, сыгравшие старшего дона Корлеоне в
первом и втором фильмах кинотрилогии о семействе Корлеоне, -
единственная пара актеров, удостоенных премии "Оскар" за исполнение роли
одного и того же персонажа.

Источник:
   1. "Даешь, молодежь!".
   2. http://ru.wikipedia.org/wiki/Вито_Корлеоне

Автор:
Елена Ганькова (Северодвинск)

Вопрос 4:
[Ведущему: обязательно прочитать "КАРИТАКУРА" и "ИЗОРБАЖЕНО" именно с
таким порядком букв!]
   Вопрос задает Льюис Кэрролл.
   На одной англоязычной КАРИТАКУРЕ изображен рай для людей, страдающих
неким расстройством. Какое животное ИЗОРБАЖЕНО на этой карикатуре?

Ответ:
Собака.

Зачет:
Dog.

Комментарий:
Одним из проявлений дислексии является перестановка букв при чтении.
Люди, страдающие от этого недуга, обычную для рая надпись GOD могут
прочитать как DOG, т.е. собака, поэтому на карикатуре вместо Бога
изображена собака. Слова "КАРИТАКУРА" и "ИЗОРБАЖЕНО" произнесены с
характерной перестановкой букв. Одним из известных людей, страдавших
дислексией, был Льюис Кэрролл.

Источник:
   1. http://jasonkatzenstein.tumblr.com/post/105812203060/artistsontumblr-cartoons-katzentoons
   2. http://www.dyslexialearning.com/perspective.htm

Автор:
Владимир Киц (Северодвинск - Санкт-Петербург), Константин Осичев
(Северодвинск)

Вопрос 5:
В вопросе есть замены.
   Заглавный герой советского мультфильма, убеждая хозяина в своей
полезности, пел: "Конечно, я не ПЕРВАЯ, но я и не ВТОРОЙ". С этим не
поспоришь, ведь он действительно не ВТОРОЙ. Назовите ПЕРВУЮ и ВТОРОГО.

Ответ:
Мельница, осел.

Комментарий:
В сказке "Кот в сапогах" первый (старший) сын получил мельницу, второй -
осла. А Кот в сапогах, доставшийся младшему сыну, будущему маркизу де
Карабасу, - не осел во всех значениях этого слова.

Источник:
Мультфильм "Кот в сапогах" (1968).

Автор:
Елена Ганькова (Северодвинск)

Вопрос 6:
В 2003 году на сайте www.kostyor.ru проходил конкурс, на который
школьники присылали свои стихотворные "вредные советы". Лучшие из них
были мастерски выполнены по всем требованиям жанра - в стилистике
основоположника Григория Остера. Автор вопроса назвал эту подборку,
немного изменив известное словосочетание, которое не утратило значимости
и привлекательности и в наши дни. Напишите получившееся в итоге
название.

Ответ:
Сделано по Г. Остеру.

Зачет:
С незначительными отклонениями.

Комментарий:
Например:
   Если ты не скушал кашу
   И не выпил свой компот,
   Ну а мама почему-то
   Говорит, что нужно есть,
   Объясни ей, что ты хочешь
   В Книгу Гиннесса попасть,
   Под портретом будет подпись:
   "Самый тощий человек".
   (Слава Левитин, Екатеринбург)
   Словосочетание "Сделано по ГОСТу" привлекает покупателей и в наше
время, поэтому часто используется в рекламных целях.

Источник:
   1. http://www.kostyor.ru/4-03/oster4-03.php
   2. http://ru.wikipedia.org/wiki/Государственный_стандарт

Автор:
Анна Дуплищева, Елена Ганькова (Северодвинск)

Вопрос 7:
Начинающий литератор Григорий Остер предпочитал ЕЕ. Уже пожилого
художника Илью Репина ошибочная она, наоборот, раздражала. У того, кому
она обязана своим появлением, ЕЕ не было. У некоторых из вас ОНА есть.
Назовите ЕЕ абсолютно точно.

Ответ:
[Буква] Ё в фамилии.

Зачет:
[Буква] Ё в псевдониме.

Комментарий:
Ранним литературным псевдонимом Остера был "Остёр"; после реформы
орфографии 1918 года Репина, которому было за семьдесят, бесило, что ему
иногда писали письма с его фамилией через Ё; ну а в фамилии "Карамзин" и
его литературных псевдонимах Ё нет и не было.

Источник:
   1. http://ru.wikipedia.org/wiki/Остер,_Григорий_Бенционович
   2. http://www.people.su/92873_3
   3. http://nesusvet.narod.ru/txt/yo

Автор:
Елена Ганькова (Северодвинск)

Вопрос 8:
Комментируя известную беседу между двумя историческими личностями,
Валерия Новодворская сравнивала ее с более поздними реалиями: "А
представьте, что Константин Симонов говорит даже не Сталину, а Брежневу,
что он мог бы вступить в РОА, в армию Власова!". О разговоре кого с кем
изначально шла речь?

Ответ:
А.С. Пушкина и Николая I.

Зачет:
В любом порядке.

Комментарий:
"... Сочувствие к идеалам заставили бы его и впрямь выйти на Сенатскую,
будь он в Петербурге 14 декабря. Здесь он Николаю I сказал правду. И
Николай съел этот прикол и не наложил взыскания. ... К счастью, его в
Петербурге не было (заяц по дороге помешал: спасибо ему, косому!)".

Источник:
   1. В.И. Новодворская. Поэты и цари. - М.: АСТ, 2010. - С. 6-10.
   2. http://www.vikent.ru/enc/4211/

Автор:
Елена Ганькова (Северодвинск)

Вопрос 9:
(pic: 20150154.jpg)
   Согласно шиитской традиции, первым имамом был зять ИКСА ИГРЕК. С 23
июля 2007 года бывший игрок NFL Кёртис Конуэй - зять ИКСА ИГРЕКА.
Назовите ИКСА и ИГРЕКА в правильном порядке.

Ответ:
Мухаммед, Али.

Комментарий:
Первым имамом был Али - зять пророка Мухаммеда. 23 июля 2007 года зятем
Мухаммеда Али стал Кёртис Конуэй, за которого Лейла Али, дочь боксера,
вышла замуж. Ранее, 27 августа 2000 года, Али вышла замуж за Джонни
Макклейна, который стал ее менеджером. В конце 2005 года они развелись.
Форма вопроса напоминает бабочку, что содержит намек на знаменитое
выражение Мухаммеда Али - "Порхай как бабочка, жаль как пчела".

Источник:
   1. http://ru.wikipedia.org/wiki/Имам
   2. http://ru.wikipedia.org/wiki/Али,_Лейла
   3. http://ru.wikipedia.org/wiki/Али,_Мохаммед

Автор:
Елена Ганькова (Северодвинск)

Вопрос 10:
Внимание, в вопросе есть замена.
   Миниатюра Михаила Жванецкого: "Молодая врачиха встречалась с одним
врачом довольно долго, потом заявила: "Надоело мне это КОЧЕВОЕ жилье.
Перехожу на ОСЕДЛОЕ"". Какие два слова латинского происхождения мы
заменили на "кочевое" и "оседлое"?

Ответ:
Амбулаторное, стационар.

Зачет:
Амбулаторное, стационарное.

Источник:
М.М. Жванецкий. Любовь (Коротко). - М.: Эксмо, 2012. - С. 74.

Автор:
Дмитрий Гевель (Северодвинск)

Вопрос 11:
На одной карикатуре изображены спортивные судьи на стадионе. Надпись
внизу картинки гласит: "Даже высокие технологии иногда бесполезны".
Действительно, даже фотофиниш не позволяет озадаченным судьям найти
ответ на вопрос... Назовите соревнующихся "спортсменов" в любом порядке.

Ответ:
Курица, яйцо.

Зачет:
В любом порядке.

Комментарий:
Судьи ищут ответ на вопрос: что было раньше - курица или яйцо?

Источник:
http://comicsia.ru/collections/ru_comicstrip/i28175.html

Автор:
Анна Дуплищева, Елена Ганькова (Северодвинск)

Вопрос 12:
Фотограф Марк Никсон опубликовал снимки плюшевых медведей, которые
служат своим хозяевам десятилетиями (и со временем стали представлять
собой жалкое зрелище). Заголовок подборки в русском издании журнала
"Esquire" [эсквайр] представляет собой усеченное на одну букву
устойчивое выражение, примененное Белинским к творчеству Гоголя.
Воспроизведите получившийся заголовок.

Ответ:
"Мех сквозь слезы".

Комментарий:
"Смех сквозь слезы" - Белинский о Гоголе.

Источник:
   1. http://esquire.ru/photo/muchloved
   2. http://www.lingvomania.info/krylatye-slova/99-2

Автор:
Дмитрий Гевель (Северодвинск)

Вопрос 13:
Одна из статей журнала "Вокруг света" посвящалась заимствованиям в
русском языке из других языков, и некоторые примеры таких заимствований
были проиллюстрированы. Рисунок, сопровождающий слово из турецкого языка
со значением "он не знает", изображал известного человека. Назовите имя
и фамилию этого человека.

Ответ:
Юрий Никулин.

Комментарий:
Слово "балбес". Никулин был нарисован в образе своего знаменитого
персонажа - Балбеса.

Источник:
"Вокруг света", 2012, N 1. - С. 116.

Автор:
Елена Ганькова (Северодвинск)

Вопрос 14:
[Ведущему: кавычки не озвучивать, четко прочитать "икс".]
   Уважаемые знатоки! При ответе не будьте многословны.
   Пару месяцев назад автор вопроса нашел в Базе Вопросов "Что? Где?
Когда?" вопрос 2009 года и, используя его на тренировке, убрал из
комментария "ИКС-". Что мы заменили в этом вопросе?

Ответ:
Э.

Комментарий:
Вопрос 2015 года. В.В. Путин в 2009 году был ЭКС-президентом, автор
вопроса убрал "ЭКС-", а не "ИКС-". С 7 мая 2012 года Путин - президент
РФ. Пожелание "не быть многословными" также намекает на него: "Буду
краток".

Источник:
http://db.chgk.info/question/sport09.2/10

Автор:
Елена Ганькова (Северодвинск)

Вопрос 15:
На одной из праздничных листовок Либерально-демократической партии
России изображено, как ее лидер - Владимир Жириновский - рисует некий
символ. На этой же листовке аббревиатура партии расшифрована как
побуждающая к действию строчка из песни 1994 года. Назовите автора этой
строчки.

Ответ:
[Валерий] Сюткин.

Комментарий:
На листовке ко Дню всех влюбленных Жириновский пальцем изображает
сердечко, аббревиатура ЛДПР расшифрована как "Любите, девушки, простых
романтиков!" - строчкой из песни группы "Браво", вошедшей в альбом
"Дорога в облака" (1994).

Источник:
   1. http://ibigdan.livejournal.com/14602475.html
   2. http://www.pesnigitara.com/statpokaz.php?i=bravo__lyubite_devus

Автор:
Елена Ганькова (Северодвинск)

Вопрос 16:
По словам героя Сергея Лукьяненко, ОНИ - странное оружие, "единственное,
которое применялось не для защиты, а только для убийства. Пусть даже и
узаконенного...". Толстой воспользовался ИМИ как минимум 11 раз.
Назовите ИХ.

Ответ:
Дуэльные пистолеты.

Зачет:
Пистолеты для дуэли и т.п.

Комментарий:
Имеется в виду знаменитый дуэлянт Федор Толстой "Американец".
Неизвестно, сколько раз в своей жизни Толстой дрался на дуэлях, однако,
по некоторым данным, он убил на них в общей сложности одиннадцать
человек.

Источник:
   1. С. Лукьяненко. Способность спустить курок. / С. Лукьяненко. "Л" -
значит люди. - М.: Изд-во "Москва", 2006. - С. 209.
   2. http://ru.wikipedia.org/wiki/Толстой,_Фёдор_Иванович

Автор:
Елена Ганькова (Северодвинск)

Вопрос 17:
(pic: 20150155.jpg)
   Перед вами изображение болида "Формулы-1" и приблизительная
информация об одном из параметров. Слева от чисел, изображенных на
картинке, мы убрали один и тот же знак, а справа - букву. Мы не
спрашиваем, что это за буква. Скажите, какое слово она обозначает.

Ответ:
Миллион.

Зачет:
Миллионы, million, millions.

Комментарий:
(pic: 20150156.jpg)
   Это стоимость годового размещения рекламы на том или ином месте
болида "Формулы-1" в миллионах долларов. С изображения мы убрали знак
доллара и букву "m".

Источник:
http://ibigdan.livejournal.com/15837893.html

Автор:
Анна Дуплищева, Елена Ганькова (Северодвинск)

Вопрос 18:
Блиц.
   Дизайнер Александр Уткин предложил необычное оформление собрания
сочинений Н.В. Гоголя.
   1. На обложке первого тома собрания сочинений звезды на ночном небе
образуют профиль автора, а рядом изображены ОНИ. Назовите ИХ обоих.
   2. На обложке второго тома собрания сочинений на дощатом полу
нарисован профиль автора, а рядом изображены горящие свечи и ОН.
Назовите ЕГО.
   3. На обложке третьего тома собрания сочинений схематически нарисован
профиль автора, и особо обозначен ОН Гоголя. Назовите ЕГО.

Ответ:
   1. Кузнец Вакула, черт.
   2. Хома Брут.
   3. Нос.

Зачет:
   1. Вакула, черт; кузнец, черт.

Комментарий:
(pic: 20150157.jpg)

Источник:
https://vk.com/novlit?w=wall-26488304_5832

Автор:
Елена Ганькова, Анна Дуплищева (Северодвинск)

Вопрос 19:
Внимание, в вопросе есть замены.
   Когда пожилая ПУШКИНА очень пренебрежительно высказалась о
ПУШКАРЁВОЙ, поэт Вольпин посмеялся и заметил: "Вы просто ревнуете,
потому что ПУШК-, да молодая, да хорошенькая!". По словам ПУШКИНОЙ,
стихи ПУШКАРЁВОЙ "пахли хорошим кофе". Какие фамилии мы заменили на
ПУШКИНУ и ПУШКАРЁВУ?

Ответ:
Ахматова, Ахмадуллина.

Комментарий:
"Вы просто ревнуете, потому что Ахма-, да молодая, да хорошенькая!".

Источник:
Т. Катаева. Анти-Ахматова. - М.: ЕвроИНФО, 2007. - С. 396-397.

Автор:
Елена Ганькова (Северодвинск)

Вопрос 20:
Сергей Лукьяненко в "Ночном дозоре" пишет: "Наверное, лучшие в мире
дипломаты - восточные. ДЕЛАЮТ ЭТО, но это вовсе не значит, что они
воздерживаются...". Согласно одному правилу, ДЕЛАТЬ ЭТО должны участники
известной игры. А что предписывает другое правило этой игры?

Ответ:
Черное и белое не брать.

Зачет:
Черное и белое не покупать/выбирать/называть и т.п.

Комментарий:
"Наверное, лучшие в мире дипломаты - восточные. "Да" и "нет" не говорят,
но это вовсе не значит, что они воздерживаются...". В детской игре нужно
"да" и "нет" не говорить, черное и белое не брать.

Источник:
   1. С. Лукьяненко. Ночной дозор. - М.: АСТ, 2006. - С. 235.
   2. http://www.malchishki-i-devchonki.ru/chernoe%20i%20beloe.html

Автор:
Елена Ганькова (Северодвинск)

Вопрос 21:
Вулкан Михара - одна из главных достопримечательностей японского острова
Идзуосима. За последние 80 лет здесь совершили самоубийство тысячи
людей, в основном - из-за неразделенной любви. Одна из компаний даже
умудрилась заработать на самоубийцах и жаждущих необычного зрелища
зеваках. Мы не спрашиваем вас, что это за компания. Ответьте, что
запретили власти Японии для снижения эпидемии суицида.

Ответ:
Продавать билеты в один конец.

Зачет:
По смыслу.

Комментарий:
На остров можно попасть, в основном, по воде. Пароходная компания делала
деньги на самоубийцах, желающих попасть на него. Совсем запретить
продавать туда билеты власти не могли, так как там постоянно живут люди.

Источник:
   1. http://www.factroom.ru/facts/43230
   2. http://ru.wikipedia.org/wiki/Идзуосима

Автор:
Владимир Киц (Северодвинск - Санкт-Петербург)

Вопрос 22:
В 70-х годах XX века один из экспонатов зоологического музея МГУ покинул
его почти на месяц, а всё для того, чтобы принять участие в съемках
кинокомедии. Почти всю картину чучело парнокопытного буквально бок о бок
"проработало" рядом с персонажем в исполнении... Кого?

Ответ:
[Эльдара] Рязанова.

Комментарий:
Бегемот, на котором спал Рязанов в фильме "Гараж".

Источник:
"Москва студенческая пешком", телеканал "Культура", 13.05.2013 г.

Автор:
Елена Ганькова (Северодвинск)

Вопрос 23:
Внимание, вопрос с заменой.
   "Человеческая комедия" поэта-сатирика Олега Молоткова - это
произведение АЛЬФА. Вам наверняка известно другое стихотворение,
которое, как и данный вопрос, тоже АЛЬФА. АЛЬФА - это два слова. Какие
именно?

Ответ:
Без глаголов.

Комментарий:
Стихотворение Молоткова начинается со строк:
   Мама, сказка, каша, кошка,
   книжка, яркая обложка,
   Буратино, Карабас,
   ранец, школа, первый класс...
   и заканчивается:
   ....юбилей, банкет, награда,
   речи, памятник, ограда.
   Вот такая "Человеческая комедия" без единого глагола. В тексте
данного вопроса нет ни одного глагола. А вам известно стихотворение
"Шепот, робкое дыханье..." А. Фета.

Источник:
   1. http://www.proza.ru/2010/10/22/922/
   2. http://ru.wikisource.org/wiki/Шёпот,_робкое_дыханье_(Фет)

Автор:
Анна Дуплищева, Елена Ганькова (Северодвинск)

Вопрос 24:
После этого вопроса мы отдохнем.
   Цитата из произведения В.А. Гиляровского: "... Я завернулся в облака
и море, сунул под голову крышку гроба... и уснул сном счастливого
человека...". А где именно устроился на ночлег повествователь?

Ответ:
В театре.

Зачет:
В уборной театра; по смыслу.

Комментарий:
Будучи в Рязани, ночевал в театре и завернулся в декорации. "Мы
отдохнем" - намек на пьесу А.П. Чехова "Дядя Ваня", т.е. на постановку в
театре, а не только на то, что вы сможете пойти поспать, как и герой
вопроса.

Источник:
   1. В.А. Гиляровский. Люди театра. / В.А. Гиляровский. Трущобные люди.
Москва и москвичи. - Кишинев, 1988. - С. 261.
   2. http://www.bibliotekar.ru/encSlov/13/167.htm

Автор:
Елена Ганькова (Северодвинск)

Тур:
14 тур. "Благоест" (Коломна)

Вопрос 1:
Блиц.
   1. Телепрограмма Никиты Белоголовцева, посвященная баскетболу,
называется ОНО. Назовите ЕГО двумя словами.
   2. На карикатуре, посвященной событию начала XXI века, изображены
ОНИ. Назовите ИХ двумя словами на разные буквы.
   3. Георг II правил Грецией в 1922-1924 годах. Затем в стране была
установлена республика. В 1935 году произошло ОНО, и Георг II вновь стал
править Грецией. Назовите ЕГО двумя словами.

Ответ:
   1. Братство кольца.
   2. Две башни.
   3. Возвращение короля.

Автор:
Кирилл Третьяков (Коломна)

Вопрос 2:
Тетраблиц. Четыре вопроса по пятнадцать секунд.
   1. Прослушайте стихотворение.
   На тихих берегах ПРОПУСК
   Церквей, венчанные крестами,
   Сияют ветхие главЫ
   Над монастырскими стенами.
   Заполните пропуск.
   2. Прослушайте стихотворение.
   Август выдался прохладным...
   Что же жарко мне?
   Стало пусто в ПРОПУСК
   Утром. На заре.
   Заполните пропуск.
   3. Прослушайте стихотворение.
   Я в восторге от ПРОПУСК города.
   Но кепчонку не сдерну с виска.
   У советских собственная гордость:
   На буржуев смотрим свысока.
   Заполните пропуск.
   4. Прослушайте стихотворение.
   Путешественник по всей Вселенной,
   Наш профессор говорил не раз,
   Как, похож на уголек нетленный,
   Умирал и возрождался ПРОПУСК.
   Заполните пропуск.

Ответ:
   1. Москва.
   2. Нагасаки.
   3. Нью-Йорка.
   4. Марс.

Комментарий:
Ответы отсылают к стихотворению Игоря Северянина.

Автор:
Кирилл Третьяков (Коломна)

Вопрос 3:
Внимание, в вопросе слово "АЛЬФА" заменяет другое слово.
   Американский антрополог Карлтон Стивенс Кун в своем труде "Расы
Европы" пишет: "В конце бронзового века на период два или три столетия
завеса АЛЬФЫ падает над расовой историей Европы". Какое слово мы
заменили на АЛЬФУ?

Ответ:
Кремация.

Комментарий:
Кремация трупов привела к отсутствию материалов для исследователей. Она,
подобно занавесу, скрыла длительный период развития рас в Европе.

Источник:
В вопросе.

Автор:
Иван Морозов (Коломна)

Вопрос 4:
Внимание, в вопросе слово "ОН" является заменой.
   Польский писатель Анджей Сапковский в статье "ВаренИк, или Нет золота
в Серых Горах" пишет: "Увы, повторяю, в случае фэнтези отсутствует
знание канона. И метода. Нет МЕТОДА. Остался ОН...". Имя какого
христианского святого мы заменили местоимением "ОН"?

Ответ:
Кирилл.

Комментарий:
Кирилл и Мефодий (по-польски - Cyryl i Metody [цырыл и методы]).
Своеобразная игра слов. Замена звука "Ф" на "Т" вполне известна
(например, Фома и Том). А если нет Мефодия, то остается Кирилл!

Источник:
В вопросе.

Автор:
Иван Морозов (Коломна)

Вопрос 5:
Как-то раз весной автор вопроса, просматривая интернет-странички, с
удивлением обнаружил, что некоторые особо заядлые пользователи
Интернета, увидев ИХ, первым делом бросаются просматривать таблицы
смайликов. Будете в следующий раз проходить мимо - обратите на это
внимание. Мы не просим вас назвать ИХ. Напишите наречие, которое в тот
день автор вопроса отправил в наибольшем количестве писем.

Ответ:
Воистину.

Комментарий:
Некоторые заядлые интернет-пользователи буквы "ХВ" воспринимают как один
из смайликов, таких как XD, т.е. смех с зажмуренными глазами, или B-),
т.е. надевший очки в роговой оправе. На сайте
http://www.diary.ru/~jafnvel/p156394694.htm так прямо и сказано: "ХВ - и
это не смайлик". :-) Кстати, Пасха дословно означает "прохождение мимо".
Тоже намек на правильный ответ.

Автор:
Иван Морозов (Коломна)

Вопрос 6:
В одной из статей "Новой газеты" рассказывалось, что в преддверии
выборов многие неугодные действующей власти статьи в ЖЖ практически
мгновенно получали огромный отрицательный рейтинг, и наоборот -
хвалебные статьи в адрес правящей партии за считанные минуты вырывались
в лидеры. Высказывалось предположение, что это делается искусственно.
Статья называлась: "На полях "Живого журнала" - высокая урожайность ЕЕ".
Чего?

Ответ:
БОТвы.

Зачет:
Ботвы.

Источник:
http://www.novayagazeta.ru/society/6070.html

Автор:
Иван Морозов (Коломна)

Вопрос 7:
Дуплет.
   Внимание, словосочетания "ПЕРВЫЙ ПРОПУСК" и "ПОСЛЕДНИЙ ПРОПУСК"
являются заменами.
   1. В математике взаимно-однозначные отображения на торе,
заключающиеся в растяжении по параллели и сжатии по меридиану, получили
название "Кот ПЕРВОГО ПРОПУСКА". Напишите фамилию, которую мы заменили
словами "ПЕРВЫЙ ПРОПУСК".
   2. В первом фильме, где снимался ПОСЛЕДНИЙ ПРОПУСК, все реплики его
героя пришлось дублировать другому актеру, а во втором фильме ему и
вовсе досталась роль глухонемого киллера. Напишите фамилию, которую мы
заменили словами "ПОСЛЕДНИЙ ПРОПУСК".

Ответ:
   1. Арнольд.
   2. Шварценеггер.

Комментарий:
Владимир Игоревич Арнольд - известнейший советский и российский
математик. Замены "первый пропуск" и "последний пропуск" - намек на
английские "first name" и "last name", т.е. имя и фамилия. Таким
образом, фамилия первого должна была оказаться именем второго.

Автор:
Иван Морозов (Коломна)

Вопрос 8:
В 2013 году в России вышла автобиографическая книга Арнольда
Шварцнеггера под названием "[Два слова пропущено]: моя необыкновенно
правдивая история". Пропущенные слова являются российским вариантом
названия известного произведения по книге американского
писателя-фантаста. Назовите этого писателя.

Ответ:
ФИлип [КИндред] Дик.

Комментарий:
Название автобиографии Шварценеггера в русском издании: "Вспомнить всё:
моя необыкновенно правдивая история". "Вспомнить всё" - это русское
название фильма по мотивам рассказа Дика. А сам рассказ назывался "We
Can Remember It for You Wholesale". Это переводили как "Мы вам всё
припомним", "В глубине памяти" или "Из глубин памяти", но никак не
"Вспомнить всё".

Автор:
Иван Морозов (Коломна)

Вопрос 9:
В повести "Республика ШКИД" для предотвращения распространения ЭТОГО
герои вынужденно меняют свою внешность, из-за чего им становится сложно
общаться с некоторыми из своих друзей. В наши дни Роспотребнадзор
предупреждает о возможности распространения ЭТОГО среди подростков,
которые любят делать групповые селфи. Назовите ЭТО.

Ответ:
Педикулез.

Зачет:
Вшивость.

Комментарий:
В повести всех парней побрили налысо, и им стало стыдно ходить к
девушкам на свидания.

Источник:
   1. Г. Белых, Л. Пантелеев. Республика ШКИД.
   2. http://ru.wikipedia.org/wiki/Селфи

Автор:
Изета Насырова (Коломна)

Вопрос 10:
Сэр Генри Мерривейл - герой детективных романов Джона Диксона Карра -
обладает тонким живым умом и склонностью к решению криминальных
головоломок. К его помощи прибегают исключительно в безнадежных случаях.
Коллеги по Уайтхоллу за глаза называют сэра Генри именем одного
литературного героя, говоря, что он очень похож на этого персонажа -
также обладает большими "дедуктивными способностями... только слишком
ленив, чтобы использовать их на практике", что он "хранит в памяти
информацию, как в картотеке", а его главным достоинством является
недюжинный ум. Напишите имя, которым называют сэра Генри.

Ответ:
Майкрофт.

Зачет:
Майкрофт Холмс.

Комментарий:
Шерлок не был столь ленив, а напротив - с удовольствием брался за любое
"дело".

Источник:
Джон Диксон Карр. Убийства в Плейг-Корте.
http://www.flibusta.net/b/116430/read

Автор:
Валерия Комаровская (Коломна)

Вопрос 11:
Главный герой одного фильма стал ИКСОМ в буквальном смысле этого слова.
В России людей, увлекающихся специфичным, по мнению автора вопроса,
хобби, называют ИКСАМИ. В Финляндии - "выдрами" и "нерпами", в Северной
Америке - "белыми медведями". Назовите ИКСА.

Ответ:
Морж.

Комментарий:
Главный герой фильма "Бивень" Уоллес Бритон попал в "лапы" маньяка, и
тот путем нескольких операций "сшил" из него моржа. Людей, занимающихся
зимним плаванием, называют моржами.

Источник:
   1. Фильм "Бивень".
   2. http://ru.wikipedia.org/wiki/Зимнее_плавание

Автор:
Кристина Шадрина (Коломна)

Вопрос 12:
Вопрос для тех, кто "в теме".
   Как ни парадоксально, но публикация сайта wikiHow подробно дает
рекомендации как стать ИМ. В частности, к таким рекомендациям относятся:
   - заведите огород и станьте вегетарианцем;
   - используйте слова, значения которых никто не знает, например,
"фантасмагория", "экспроприация", "ревербератор", "экзальтированность";
   - когда вас о чем-то спрашивают, не отвечайте напрямую, вместо этого
ответьте расплывчато, задайте встречный вопрос или просто проявите
сарказм и другое.
   Назовите мультипликационного героя, который, согласно шутке, является
первым советским ИМ.

Ответ:
Шарик.

Зачет:
Шарик из Простоквашино.

Комментарий:
Шарик вместо полезных в хозяйстве вещей купил себе кеды и фоторужье.
Хипстеры - первоначально представители особой субкультуры,
сформировавшейся в среде поклонников джазовой музыки, понятие образовано
от жаргонного "to be hip" ("быть в теме"). Главная задача современного
хипстера - достижение полной независимости взглядов и действий.

Источник:
   1. http://ru.wikihow.com/быть-хипстером
   2. http://www.anekdot.ru/id/502287/
   3. http://insurgent.ru/hipster

Автор:
Александр Романкин (Коломна)

Вопрос 13:
Модель походного вентилятора фирмы "Экспедиция" своим названием как бы
говорит покупателю, что этот вентилятор просто делает свою работу. Это
название созвучно слогану компании, основанной в 1964 году. Назовите эту
компанию.

Ответ:
"Nike".

Комментарий:
Слоган - "Just Do It" ("Просто сделай это").

Источник:
   1. http://www.e-xpedition.ru/products/ventiljatory-just-duet/
   2. http://en.wikipedia.org/wiki/Just_Do_It

Автор:
Кирилл Третьяков (Коломна)

Вопрос 14:
Статья на сайте politonline.ru [полит онлайн точка ру], посвященная
событиям весны 2014 года на Украине, называется "Обескровленная и
расколотая". Какие две буквы мы заменили в предыдущем предложении?

Ответ:
ЫМ.

Комментарий:
"ОбескрЫМленная и расколотая".

Источник:
http://www.politonline.ru/comments/16482.html

Автор:
Кирилл Третьяков (Коломна)

Вопрос 15:
Город Урюпинск в фольклоре служит символом глубинки. В прессе и
литературе Урюпинск зачастую парадоксально называют ЕЮ. Назовите ЕЕ
тремя словами, начинающимися на соседние буквы.

Ответ:
Столица российской провинции.

Источник:
   1. http://www.5-tv.ru/news/62882/
   2. http://www.rg.ru/2014/11/11/reg-ufo/urypinsk.html

Автор:
Кирилл Третьяков (Коломна)

Вопрос 16:
Внимание, слово "АЛЬФА" заменяет несколько других слов.
   Ричард Касл - герой детективного сериала - писатель, помогающий вести
следствие полицейским убойного отдела. Он всегда полон различных
фантастических идей и о причинах убийств, и о самих жертвах, и о
способах умерщвления. Так, при расследовании смерти финансиста с
Уолл-Стрит Касл предполагает, что его убила АЛЬФА, ведь на теле нет
оборонительных ран. Назовите того, кто в XVIII веке впервые ввел понятие
АЛЬФЫ.

Ответ:
Адам Смит.

Комментарий:
АЛЬФА - невидимая рука рынка.

Источник:
Сериал "Касл", s06e19, 04:54.

Автор:
Валерия Комаровская (Коломна)

Вопрос 17:
(pic: 20150158.jpg)
   На розданной вам картинке изображен Монумент Славы в виде рабочего с
крыльями. Жители Самары в шутку именуют его "Непьющий человек", а на
вопрос "Почему непьющий?" отвечают именно так. Как именно?

Ответ:
[У него] руки заняты.

Источник:
http://www.9999-777.ru/pamyatniki-samary.html

Автор:
Иван Морозов (Коломна)

Вопрос 18:
(pic: 20150159.jpg)
   На одном из сайтов автор вопроса увидел данное фото. Остроумным
комментарием к этой фотографии явилась цитата заглавного героя
произведения 1969 года. Напишите эту цитату.

Ответ:
Это неправильные пчелы!

Источник:
   1. http://ibigdan.livejournal.com/10706110.html
   2. http://ru.wikipedia.org/wiki/Винни-Пух_(мультфильм)

Автор:
Иван Морозов (Коломна)

Вопрос 19:
Блинов и Жарков - участники ЧГК-движения, которые должны быть знакомы
всем присутствующим. Блинов прославился благодаря Саше Барону Коэну в
качестве человека с диктаторскими замашками, не привыкшего ограничивать
свои желания. Жарков был известен своим стремлением докапываться до
истины, даже если это представляло опасность для его жизни. Автор
вопроса знаком с Блиновым лично, а с Жарковым заочно и, пользуясь
случаем, передает им привет. Какие фамилии в вопросе заменены на
"Блинов" и "Жарков"?

Ответ:
Аладин и Холодов.

Источник:
   1. http://ru.wikipedia.org/wiki/Диктатор_(фильм)
   2. http://ru.wikipedia.org/wiki/Холодов,_Дмитрий_Юрьевич

Автор:
Александр Берелехис (Уфа)

Вопрос 20:
(pic: 20150160.jpg)
   Перед вами ИКСЫ. ИКС является символом ОМЕГИ. "ОМЕГА" - название
русского романа второй половины XIX века. Назовите подмосковный город,
название которого происходит от ОМЕГИ и на гербе которого изображен ИКС.

Ответ:
Воскресенск.

Комментарий:
ИКС - Феникс: слева - актер Хоакин Феникс, а справа - созвездие Феникс;
ОМЕГА - возрождение, воскрешение; у Льва Толстого есть роман
"Воскресение".

Источник:
   1. http://www.filmz.ru/photos/181107/
   2. http://ru.wikipedia.org/wiki/Феникс_(созвездие)
   3. http://www.symbolarium.ru/index.php/Феникс,_птица
   4. http://ru.wikipedia.org/wiki/Воскресение_(роман)
   5. http://ru.wikipedia.org/wiki/Герб_Воскресенска

Автор:
Кирилл Третьяков (Коломна)

Вопрос 21:
Коса, зулу, сесото, тсвана, свази, венда, тсонга - это официальные ОНИ.
Назовите ИХ двумя словами, начинающимися на соседние буквы.

Ответ:
Языки ЮАР.

Источник:
http://ru.wikipedia.org/wiki/Языки_Южно-Африканской_Республики

Автор:
Кирилл Третьяков (Коломна)

Вопрос 22:
По правилам футбола гол не засчитывается, если мяч был направлен в
ворота первым касанием при пробитии штрафного удара, свободного
штрафного удара, удара от ворот, углового удара или непосредственно
вбросом аута. Согласно футбольной шутке прошлого десятилетия, защитник
сборной России по футболу Юрий Ковтун установил новый рекорд, забив уже
в первом тайме два гола. Достижение прославленного защитника могло быть
еще более выдающимся, если бы сразу после второго гола он не был
травмирован Александром Филимоновым. Какие четыре буквы мы неоднократно
пропустили в этом вопросе?

Ответ:
авто.

Комментарий:
Все слова "автогол" в этом вопросе были заменены просто на "гол".
Согласно шутке, любитель забивать в свои ворота Юрий Ковтун после
второго автогола был благоразумно травмирован своим же вратарем.

Источник:
http://lurkmore.to/Российский_футбол

Автор:
Александр Романкин (Коломна)

Вопрос 23:
Статья в "Нью-Йорк Таймс" от 26 марта 2014 года носит название "Crime
and Punishment". Какую букву мы пропустили в этом вопросе?

Ответ:
A.

Комментарий:
"Crimea and Punishment".

Источник:
http://www.nytimes.com/2014/03/27/opinion/blow-crimea-and-punishment.html

Автор:
Кирилл Третьяков (Коломна)

Вопрос 24:
В эфиопской письменности названия многих букв сходны с другими
семитскими системами письма, но есть и оригинальные. Например,
двенадцатую букву, название которой в других семитских алфавитах
означает "рыба", эфиопы назвали в честь другого животного, так, будто
слишком буквально поняли известное евангельское изречение. А один
афоризм гласит, что если человек очень легко меняет свои убеждения, то
доверять ему стоит так же, как этому животному. Что это за животное?

Ответ:
Змея.

Комментарий:
Христос сказал: "И когда попросит рыбы, подал бы ему змею?"
(продолжением фразы о том, что любящий отец не дал бы голодному сыну
вместо просимого хлеба камень). Это единственное изречение в Евангелии,
в котором упоминается замена рыбы на что-либо иное. Согласно афоризму,
если человек меняет свои убеждения так же легко, как змея - кожу, то
доверять ему стоит тоже как змее.

Источник:
Евангелие от Матфея, 7:10.

Автор:
Иван Морозов (Коломна)

Тур:
15 тур. "Немчиновка" (Москва)

Вопрос 1:
Изобретатель матрешки завещал похоронить себя [пропуск]. Восстановите
шесть пропущенных слов.

Ответ:
"... в гробу в гробу в гробу".

Источник:
http://ibigdan.livejournal.com/9225875.html

Автор:
Игорь Доскоч

Вопрос 2:
(pic: 20150161.jpg)
   На картинке из сообщества политических карикатур приведены "сведения"
по экстремизму - вероятно, склонности вести священные войны - среди
последователей разных религий. Какой символ мы закрыли на последнем
столбце?

Ответ:
Надкушенное яблоко.

Зачет:
Эмблема "Apple".

Комментарий:
Одна из наиболее популярных тем холиваров (священных войн) в Интернете -
это войны между макинтошниками и виндузятниками.

Источник:
http://politicartoons.livejournal.com/3287606.html

Автор:
Игорь Доскоч

Вопрос 3:
Это слово встречается в Базе Вопросов более 1000 раз. Редактор Microsoft
Word этого слова не знает и предлагает заменить его на повинность.
Воспроизведите это слово.

Ответ:
Борок.

Комментарий:
Дмитрий Борок - автор более чем 1000 вопросов ЧГК, лежащих в Базе. Word
предлагает заменить слово "Борок" на "оброк".

Источник:
   1. http://db.chgk.info/
   2. Microsoft Word.

Автор:
Мишель Матвеев

Вопрос 4:
В прошлом году автор вопроса занимался закупкой компьютерных мышей.
После получения заказа выяснилось, что ушлые или невнимательные продавцы
пропустили в описании шесть букв. Каких?

Ответ:
ОМЕХАН.

Комментарий:
Вместо ОПТИЧЕСКИХ продали ОПТомеханИЧЕСКИХ, т.е. шариковых.

Автор:
Игорь Доскоч

Вопрос 5:
   <раздатка>
   Пiнгвiни Живуть Зимою
   </раздатка>
   В раздатке мы показали лишь чуть больше 40% от полного текста. Вас же
мы попросим назвать часть тела, имя и источник из аналогичного текста на
русском языке.

Ответ:
Голова, Жак, фонарь.

Комментарий:
Это украинская мнемоника для запоминания цветов спектра: "Чому Пiнгвiни
Живуть Зимою Без Сво&#1111;х Фантазiй". Русский аналог - "Как Однажды
Жак-Звонарь Головою Сбил Фонарь".

Источник:
http://www.zapominalki.ru/interest-ukrainian-zapominalki/

Автор:
Евгений Майсюк

Вопрос 6:
Почтовый сервис Google, который располагается в домене gmail.com,
изначально был так назван по первой букве имени весьма известного
персонажа, но, вероятно, содержать этот адрес изначальным владельцам
стало лень, и они продали адрес Google. Что это был за персонаж?

Ответ:
[Кот] Гарфилд.

Комментарий:
Кот Гарфилд известен своей ленью.

Источник:
http://www.factroom.ru/facts/26623

Автор:
Надежда Герасименко

Вопрос 7:
Внимание, в вопросе есть замена.
   Прослушайте цитату из Брокгауза и Ефрона, "Каштан": "Он образует
обширные насаждения в средней и южной Франции, Испании и Италии и
разводится у нас в Крыму и Туркестане, но здорово произрастает и в
Закавказье - нижней и средней горных полосах, изредка поднимаясь до 5-6
тысяч футов высоты". Какое слово мы заменили?

Ответ:
Дико.

Комментарий:
Вместо "здорово". Во времена Брокгауза и Ефрона такое употребление слова
"здорово" выглядело несколько дико.

Источник:
http://dic.academic.ru/dic.nsf/brokgauz_efron/50717/

Автор:
Игорь Доскоч

Вопрос 8:
На специальной наклейке в упаковке с тирольским пирогом Черри-Бренди
напечатано предостережение об опасности, которой могут подвергнуться и
взрослые, и дети. Какой?

Ответ:
[Попадания] косточки.

Комментарий:
Черри - вишня, в пироге случаются и косточки, чего нет в пирогах с
другими составами.

Источник:
(pic: 20150162.jpg)

Автор:
Игорь Доскоч

Вопрос 9:
(pic: 20150163.jpg)
   На раздатке мы закрыли нечто, к чему пользователи Интернета
пририсовали видимую надпись. Назовите двумя словами то, что мы закрыли.

Ответ:
Пирамида Маслоу.

Источник:
http://weknowmemes.com/2013/08/maslows-hierarchy-of-needs-updated/

Автор:
Игорь Доскоч

Вопрос 10:
Внезапный взрыв популярности этого предмета в Америке в 1970-х связан с
тем, что много болевший к тому времени Роберт Хайнлайн уделил ему немало
внимания в своих романах, включая культовый "Чужак в чужой стране".
Назовите этот предмет.

Ответ:
Водяной матрас.

Зачет:
Водяная кровать.

Комментарий:
Хайнлайну пришлось провести много времени в постели, и он занимался
обдумыванием оптимальной конструкции такой кровати.

Источник:
http://en.wikipedia.org/wiki/Waterbed

Автор:
Надежда Герасименко

Вопрос 11:
(pic: 20150164.jpg)
   Восстановите второе слово, входящее в название изображенного жука.

Ответ:
Schwarzeneggeri.

Зачет:
С незначительными ошибками.

Комментарий:
Передние лапки насекомого напоминают накачанные бицепсы.

Источник:
http://ru.wikipedia.org/wiki/Agra_schwarzeneggeri

Автор:
Виталий Федоров

Вопрос 12:
Отрывок из мемуаров немецкого танкиста Отто Кариуса: "Повсюду активно
работали саперы. Они даже СДЕЛАЛИ ЭТО! Такая уловка иногда удавалась
позднее на Западном фронте в отношении американцев, но никак не
проходила с русскими". Конец цитаты. Маленький гном Вася тоже СДЕЛАЛ
ЭТО, и ему уловка удалась. Что мы заменили на "СДЕЛАТЬ ЭТО"?

Ответ:
Повернуть в противоположную сторону предупредительные знаки.

Зачет:
Повернуть указатели.

Источник:
   1. http://militera.lib.ru/memo/german/carius_o/05.html
   2. М. Липскеров. Маленький гном Вася.

Автор:
Игорь Доскоч

Вопрос 13:
Знакомый автора вопроса ошибочно полагал, что АЛЬФА - это отделение
компании "БЕТА мотор корпорейшн". Назовите АЛЬФУ и БЕТУ.

Ответ:
Акура, Мазда.

Автор:
Анна Рогозина

Вопрос 14:
Прослушайте шуточный стишок-клерихью:
   Мы в России считаем, что Тютчев
   Ихних Вордсвортов в тридцать раз круче.
   Но по мненью (пропуск один),
   Эдвард Лир всё же лучше (пропуск два).
   Заполните любой из пропусков.

Ответ:
"... британских ученых...", "... Кручёных".

Источник:
http://dohlik-nemruchi.livejournal.com/69264.html

Автор:
Игорь Биткин

Вопрос 15:
В романе Лукьяненко маг применяет заклинание "абсолютного ИКСА", чтобы
остановить людей, штурмующих помещение. Далее уточняется, что заклинаний
"абсолютного ИКСА" два, но второе применяется только к одушевленным
объектам. Какое слово мы заменили на ИКС?

Ответ:
Тормоз.

Автор:
Виталий Федоров

Вопрос 16:
В 1931 году в Мюнхене начался суд над неким Францем Таузендом, сумевшим
ввести в заблуждение многих влиятельных людей Германии - видимо,
благодаря отчаянному положению Веймарской республики. Последний подобный
процесс в Баварии закончился в начале XVII века, однако был назначен
следственный эксперимент, в ходе которого Таузенд должен был... Сделать
что?

Ответ:
Получить золото.

Источник:
http://www.webznayka.ru/interes-fakty/prestupleniya/77-afera-sintez-zolota-frants-tauzend

Автор:
Виталий Федоров

Вопрос 17:
Как ни странно, ОНА - часто не ОНА и может состоять из белков, жиров,
свободных жирных кислот, минеральных солей, а может и содержать аж целых
4,2% ЕЕ. Назовите ЕЕ.

Ответ:
Сера.

Комментарий:
Приведен состав ушной серы.

Автор:
Игорь Доскоч

Вопрос 18:
В 1938 году сотрудники лаборатории Кюри работали над новым радиоактивным
элементом. Его поспешно объявляли то изотопом актиния, то лантана, а то
и вовсе трансураном. Коллеги в насмешку предлагали назвать элемент
Curium. Какую систему мы пропустили?

Ответ:
IOS.

Комментарий:
CuriOSium, iOS - операционная система iPhone, iPad.

Автор:
Виталий Федоров

Вопрос 19:
Увидев сильно подержанный внедорожник Сузуки, автор вопроса подумал, что
его название слишком мало, и ЭТО надо повторить. Что именно?

Ответ:
Гранд.

Комментарий:
Внедорожник назывался "Grand-Vitara". Grand-grandmother = прабабушка.
   z-checkdb: На самом деле прабабушка по-английски - great-grandmother
(Андрей Кокуленко).

Автор:
Игорь Доскоч

Вопрос 20:
Средневековый богослов Ефрем Сирин вычислил точную дату сотворения мира.
На момент появления Адама на деревьях были плоды, следовательно, стоял
сентябрь. Далее Сирин определяет и число, исходя из того, что Бог никак
не мог сотворить ничего ТАКОГО. Какого?

Ответ:
Ущербного.

Комментарий:
Бог не мог создать ущербную луну, следовательно, на четвертый день
творения она была полной. Остальные вычисления вполне по силам
математике девятого века. В итоге вышло 1 сентября.

Автор:
Виталий Федоров

Вопрос 21:
Закончите стихотворение-порошок:
   душа так жаждет обновленья
   мозг тщится новое узнать
   и палец тянется к заветной
   ...

Ответ:
эф пять

Зачет:
F5

Источник:
https://vk.com/sandalporoshki?w=wall-31481258_36173

Автор:
Игорь Доскоч

Вопрос 22:
На Байконуре есть традиция: за полтора километра до пусковой площадки
космонавт выходит из автобуса и стоит несколько минут у заднего колеса.
Назовите космонавта, который привел первоначальную традицию, которая
идет со времен Гагарина, в существующий формат.

Ответ:
Терешкова.

Комментарий:
Ранее суеверные космонавты-мужчины повторяли поступок Гагарина и орошали
колесо. А потом полетела Терешкова.

Автор:
Александр Кузьмич

Вопрос 23:
По мнению комментатора Александра Шмурнова, добивающиеся ЭТОГО подобны
женщинам, мечтающим не иметь детей. Назовите ЭТО максимально точно.

Ответ:
0:0.

Зачет:
Нулевая ничья и т.п.

Автор:
Виталий Федоров

Вопрос 24:
Из подписи одного интернет-пользователя: "У меня три АЛЬФЫ, поэтому я
ничего не должен". Назовите любой из его долгов.

Ответ:
Родить сына / посадить дерево / построить дом.

Комментарий:
У пользователя три дочери.

Автор:
Александр Фаюстов

Тур:
16 тур. "Ксеп" (Москва)

Редактор:
Ярослав Котышов

Вопрос 1:
Разбогатевший и ставший светским человеком рэпер говорит, что теперь у
него в шкафу висят ОНИ. На одной картинке среди НИХ есть, например,
454545, E8E8E8 и 2F2F2F. Назовите ИХ тремя словами.

Ответ:
Пятьдесят оттенков серого.

Комментарий:
Шон Андерсон, который, по его словам, прежде по 50 раз в день слушал 50
Cent, очевидно, намекает на появившиеся у него костюмы. На картинке
представлены 50 оттенков серого, записанные шестнадцатеричным кодом.

Источник:
   1. http://genius.com/4349269
   2. https://twitter.com/tproger/status/560406953332330496/

Автор:
Сергей Даровских, Ярослав Котышов

Вопрос 2:
Дочь Василия и Доротеи Васильевых родилась на корабле, шедшем из
Мурманска во Владивосток. Поэтому новорождённой дали имя Марина - в
честь моря. В тексте вопроса мы немного изменили одно из слов. Напишите
это слово в исходном виде.

Ответ:
Карина.

Комментарий:
Девочка родилась в Карском море, в честь которого и получила имя.

Источник:
http://www.trud.ru/article/10-04-2013/1292062_poslednjaja_iz_cheljuskintsev_karina_vasiljeva_otmechaet_79-j_den_spasenija.html

Автор:
Евгений Пашковский

Вопрос 3:
Святой Иероним, наставляя будущую монахиню Евстохию, советует ей бежать
от искушений внешнего мира и предостерегает ее от судьбы другой женщины.
Кто эта женщина?

Ответ:
Жена Лота.

Комментарий:
Мир Иероним сравнивает с Содомом, из которого Евстохия должна бежать без
оглядки.

Источник:
http://www.ccel.org/ccel/schaff/npnf206.v.XXII.html

Автор:
Юрий Разумов

Вопрос 4:
Назовите самый древний из побратимов бразильского города Натал.

Ответ:
Вифлеем.

Комментарий:
Название бразильского города этимологически связано с Рождеством.

Источник:
http://ru.wikipedia.org/wiki/Натал_(город)

Автор:
Евгений Пашковский

Вопрос 5:
После перехода футболиста Стивена Дефура в "Андерлехт" болельщики
"пролетарского" клуба "Стандард" вывесили в его адрес угрожающий баннер
с надписью на английском языке. Эта надпись совпадает с названием марки
одежды, которую основал сын индейского вождя. Надпись содержит два
прилагательных. Напишите их.

Ответ:
Red, dead.

Зачет:
Красный, мертвый.

Комментарий:
На баннере было написано "Red or dead". Надпись отсылает к выражению
"Better red than dead" времен Холодной войны.

Источник:
http://www.bbc.com/sport/0/football/30976806

Автор:
Сергей Даровских

Вопрос 6:
Герой известной пьесы говорит, что из шести обезьян примерно половина
должна СДЕЛАТЬ ЭТО головами, а половина - хвостами. Что именно сделать?

Ответ:
Приземлиться.

Зачет:
Упасть и другие синонимы.

Комментарий:
Розенкранц и Гильденстерн в пьесе Тома Стоппарда играют в орлянку,
которая по-английски называется "heads or tails". Гильденстерн
рассуждает о теории вероятностей и говорит, что шесть обезьян, будучи
подброшенными в воздух, поровну приземлятся на хвосты и на головы. Этим
диалогом Стоппард, видимо, отсылает читателя к "Теореме о бесконечных
обезьянах", согласно которой абстрактная обезьяна рано или поздно
напечатает любой текст, например, "Гамлета" Шекспира.

Источник:
http://lib.ru/PXESY/STOPPARD/r_g_engl.txt

Автор:
Александр Марков

Вопрос 7:
Внимание, в вопросе есть замены.
   Пользователь сайта booking.com иронизирует, что в мотеле в Пущино
сложно находиться из-за ПЕРВОГО, зато там есть ВТОРАЯ. Что мы заменили
на ВТОРУЮ?

Ответ:
Дискотека.

Комментарий:
ПЕРВЫЙ - шум.

Источник:
http://www.booking.com/hotel/ru/f1-motel.ru.html#tab-reviews

Автор:
Ярослав Котышов

Вопрос 8:
Какого поэта родители до пяти лет одевали как девочку и называли Софией?

Ответ:
[Райнер Мария] Рильке.

Комментарий:
Хотя среди его имен было другое женское имя. Мать Рильке очень
переживала из-за смерти дочери в недельном возрасте.

Источник:
http://en.wikipedia.org/wiki/Rainer_Maria_Rilke

Автор:
Юрий Разумов

Вопрос 9:
Внимание, в вопросе есть замена.
   Писатель Иехиэль Динур после Второй мировой войны включил в свой
псевдоним ИКС. Другой ИКС был в первоначальном варианте названия
рассказа. Кто автор этого рассказа?

Ответ:
[Александр] Солженицын.

Комментарий:
ИКС - лагерный номер. Иехиэль Динур два года был узником Аушвица, а
потом стал писать под псевдонимом Кацетник 135633, где 135633 - его
лагерный номер. Солженицын переименовал рассказ "Щ-854" о заключенном
Иване Шухове в "Один день Ивана Денисовича".

Источник:
   1. http://en.wikipedia.org/wiki/Yehiel_De-Nur
   2. http://festival.1september.ru/articles/531138/

Автор:
Александр Марков

Вопрос 10:
Герой современного произведения считает, что в Лос-Анджелесе сложно
найти девушку, если у тебя всего одна строчка там. Ответьте, где именно,
использовав аббревиатуру.

Ответ:
[В профиле на] IMDB.

Комментарий:
Девушки в Лос-Анджелесе, по мнению героя, избалованы голливудскими
знаменитостями и не обращают внимания на начинающих актеров.

Источник:
https://itunes.apple.com/us/album/the-women-of-la/id603620511?i=603620518

Автор:
Александр Марков

Вопрос 11:
Персонаж Ивлина Во, узнав о НЕЙ, упоминает украшение, сделавшее его
одиноким царем лесов. Назовите героя, закричавшего о НЕЙ после взрыва.

Ответ:
Мальчиш-Кибальчиш.

Комментарий:
ОНА - измена. Персонаж Во говорит о появившихся у него рогах.

Источник:
   1. И. Во. Возвращение в Брайдсхед.
http://www.flibusta.net/b/337765/read
   2. А.П. Гайдар. Сказка про военную тайну, Мальчиша-Кибальчиша и его
твердое слово. http://www.flibusta.net/b/17629/read

Автор:
Евгений Пашковский

Вопрос 12:
Находящийся в Нью-Йорке магазин итальянских продуктов называется
"Итали". Воспроизведите название этого магазина так, как оно написано на
вывеске.

Ответ:
Eataly.

Источник:
http://www.eataly.com/nyc-locations-and-hours

Автор:
Иделия Айзятулова

Вопрос 13:
По одной из версий, смерть героини британского сериала была вызвана
проблемами с Исламским государством Ирака и Леванта. В честь какой
богини была названа эта героиня?

Ответ:
Изида.

Комментарий:
Речь идет о лабрадоре из сериала "Аббатство Даунтон". Левант по-арабски
al-Sham, поэтому название террористической организации сокращают как
ISIS. Кроме того, другое ее название - Исламское государство Ирака и
Сирии.

Источник:
http://www.dailymail.co.uk/news/article-2823184/From-mouth-Lord-Grantham-thinks-Downton-killing-Isis-labrador-Islamists-complete-berk.html

Автор:
Дмитрий Карякин

Вопрос 14:
Сторонник румынской антисемитской ультраправой православной организации
Мирча Элиаде писал, что она "одерживает победу по воле Господней". Чье
имя первоначально входило в название этой организации?

Ответ:
Михаила [Архангела].

Комментарий:
Румынская партия называлась "Легион Михаила Архангела", почти как
русский "Союз имени Михаила Архангела".

Источник:
http://traditio-ru.org/wiki/Легион_(движение)

Автор:
Михаил Новосёлов

Вопрос 15:
Говоря об исламском экстремизме, Фарит Салман, муфтий Ямало-Ненецкого
округа, признаётся, что ваххабиты всё чаще приходят в мечети с НИМИ.
Это, впрочем, неудивительно. Согласно одному утверждению, шесть ИХ часто
соседствуют с двенадцатью коленами Израилевыми. Назовите ИХ.

Ответ:
Обрезы.

Источник:
   1. http://expert.ru/russian_reporter/2013/43/halifat-vseya-rusi/
   2. Бородатый анекдот.

Автор:
Дмитрий Карякин

Вопрос 16:
Какой футбольный клуб, согласно шутке, назван в честь двух вещей,
которые больше всего не любят исламские террористы?

Ответ:
"Вест Хэм [Юнайтед]".

Комментарий:
Ham - ветчина. Террористы, очевидно, не любят Запад, но и к свинине вряд
ли относятся хорошо.

Источник:
https://twitter.com/paulsinha/status/526005721809186816/

Автор:
Юрий Разумов

Вопрос 17:
В статье Нурсултана Назарбаева упоминается невидимая метла и огненный
язык дракона Айдахара. Название какого города встречается в этой статье?

Ответ:
Семипалатинск.

Зачет:
Курчатов, Семипалатинск-21.

Комментарий:
Назарбаев рассказывает о ядерных испытаниях в Казахстане.

Источник:
Н. Назарбаев. Когда земля вскрикнула. // "Дилетант", 2015, N 1.

Автор:
Дмитрий Карякин

Вопрос 18:
Физик Чарльз Чой отмечает, что ИКСЫ так же сильны и опасны, как Халк.
Героиня пьесы Пола Зиндела выясняет, что ИКСЫ могут сделать цветы
красивее, а могут и погубить их. Какое слово, пишущееся через дефис, мы
заменили на ИКСЫ?

Ответ:
Гамма-лучи.

Комментарий:
Физик считает, что предполагаемая концентрация гамма-лучей убила бы
Брюса Бэннера, а не превратила в Халка. Героиня Пола Зиндела изучала
влияние гамма-излучения на маргаритки.

Источник:
   1. http://www.livescience.com/2590-gamma-rays-incredible-hulking-reality.html
   2. http://en.wikipedia.org/wiki/The_Effect_of_Gamma_Rays_on_Man-in-the-Moon_Marigolds

Автор:
Юрий Разумов

Вопрос 19:
Знакомая Ирины Насановской, находясь с подругой во Львове, решила, что к
русским там плохо относятся. Причиной этого стал ответ местного жителя
на ее вопрос. Ответьте на этот вопрос.

Ответ:
Текущее время словами или цифрами (22:40 на момент задания вопроса).

Комментарий:
На вопрос "Который час?" мужчина ответил: "Две години", т.е. "Два часа".
Девушки подумали, что он назвал их "гадинами".

Источник:
http://www.anekdot.ru/id/733008/

Автор:
Ярослав Котышов

Вопрос 20:
В Бельгии автор вопроса купил себе чехол для телефона. Мы не спрашиваем,
что на нем изображено. Что на нем написано?

Ответ:
"Ceci n'est pas une pipe".

Зачет:
"Это не трубка".

Комментарий:
Изображение на чехле - знаменитая картина Магритта.

Источник:
Чехол для телефона.

Автор:
Сергей Даровских

Вопрос 21:
Комментируя статистику выживаемости, Ада Суонсон сказала: "Если не
можешь быть ИКСОМ, будь маленьким парнем, прячущимся в углу". Какое
слово мы заменили на ИКС?

Ответ:
Король.

Комментарий:
Речь идет о выживаемости шахматных фигур к концу партии. Короли выживают
в 100% случаев, потому что их нельзя побить. На втором месте - белая
пешка с поля h2, которая выживает в 73,9% партий.

Источник:
http://knowmore.washingtonpost.com/2014/10/24/what-are-the-chances-of-survival-for-each-chess-piece/

Автор:
Ярослав Котышов

Вопрос 22:
Героиня романа Кацетника "Кукольный дом" в концлагере попадает в отряд
проституток, которые должны обслуживать немецких солдат. Назовите
англичанина, который прочитал этот роман за несколько лет до
самоубийства.

Ответ:
Иен Кёртис.

Комментарий:
Такой отряд в книге Кацетника называется "Joy Division" [джой дивИжн].
Это название в 1978 году позаимствовал для своей группы Иен Кёртис. Два
года спустя музыкант повесился.

Источник:
   1. http://curiousjew.blogspot.ru/2009/04/house-of-dolls.html
   2. http://www.theguardian.com/books/2008/may/10/popandrock.joydivision

Автор:
Александр Марков

Вопрос 23:
После появления на вулканическом острове Санторини ИКСА погонщики ослов
стали получать компенсацию за каждый проданный билет. Какое слово мы
заменили на ИКС?

Ответ:
Фуникулер.

Комментарий:
Крупнейшие поселения острова Санторини расположены на обрывистой части
острова. Подъем или спуск там был возможен только с помощью ослов. С
появлением фуникулера работа у погонщиков ослов практически пропала.

Источник:
http://www.santonet.com/holidays/cablecar.html

Автор:
Юрий Разумов

Вопрос 24:
Книга о том, насколько различны вампиры и оборотни, называется "Вампиры
[ПЕРВЫЙ ПРОПУСК], оборотни [ВТОРОЙ ПРОПУСК]". Заполните любой из
пропусков.

Ответ:
С Марса.

Зачет:
С Венеры.

Комментарий:
"Вампиры с Венеры, оборотни с Марса". Более известная книга "Мужчины с
Марса, женщины с Венеры" рассказывает о непреодолимых различиях мужчин и
женщин.

Источник:
http://www.amazon.com/Vampires-Venus-Werewolves-Mars-Comprehensive-ebook/dp/B00AC6Q2MW/

Автор:
Юрий Разумов

Тур:
17 тур. "Разведёнка с прицепом" (Москва - Серпухов)

Редактор:
Артем Колесов

Вопрос 1:
Обсуждение пакета вопросов команда вела в Google Docs. В начале
обсуждения каждый участник должен был кое-что выбрать, и первую же
реплику на эту тему можно было посчитать признанием. Что интересно,
драки, аналогичной описанной в культовом фильме 90-х, не возникло, так
как никто не захотел быть... Кем?

Ответ:
Черным.

Зачет:
Мистером Черным, Mr. Black.

Комментарий:
Для удобства обсуждения каждый член команды должен был выбрать, какого
цвета будут его комментарии. Первый же комментарий "Я голубой" повлек
шуточки на тему каминг-аута. Что интересно, черным не захотел писать
никто. К примеру, одному из редакторов было пофиг, и он стал фиолетовым.
В культовом фильме Квентина Тарантино "Бешеные псы" есть сцена, где
криминальный босс раздает "цветные" клички бандитам. А когда недовольный
своим цветом Мистер Розовый спросил, почему они сами не могут выбрать
себе цвета, босс ответил: "Я уже попробовал однажды, ничего не вышло.
Четверо парней подрались из-за того, кто будет Мистером Черным".

Источник:
   1. Обсуждение пакета вопросов.
   2. Фильм "Бешеные псы", реж. Квентин Тарантино.

Автор:
Артем Колесов

Вопрос 2:
Печально известный с 60-х ИКС ИГРЕК был двойным и по составу, и по
действию. Назовите фильм, в котором Мистер ИГРЕК оказывается двойным
ИКСОМ.

Ответ:
"Бешеные псы".

Комментарий:
В фильме "Бешеные псы" Мистер Оранж (он же Мистер Оранжевый) оказывается
двойным агентом. Агент Оранж - это печально известная со времен войны во
Вьетнаме смесь дефолиантов и гербицидов в пропорции 1:1. Кстати, помимо
Агента Оранжа, использовались также Агент Блю, Агент Уайт и другие
"Агенты". Прямо как в "Бешеных псах".

Источник:
   1. http://ru.wikipedia.org/wiki/Агент_Оранж
   2. Фильм "Бешеные псы", реж. Квентин Тарантино.

Автор:
Дмитрий Сахаров, Артем Колесов

Вопрос 3:
В одном из ранних вариантов второстепенные героини пытаются обмануть
героя, но их выдают пятна крови, в результате чего выдают заглавную
героиню. Назовите эту героиню.

Ответ:
Золушка.

Зачет:
Синдерелла.

Комментарий:
Ее сестер выдают пятна крови на ступнях. А саму Золушку в итоге выдают
замуж за принца.

Источник:
Valerie Steele "Shoes and the Erotic Imagination".

Автор:
Антон Пинчук

Вопрос 4:
Рассказывают, что фильм "Интерстеллар" настолько не понравился
известному физику, что тот прямо во время сеанса СДЕЛАЛ ЭТО. Какое
выражение из трех слов мы заменили словами "СДЕЛАЛ ЭТО"?

Ответ:
Встал и вышел.

Зачет:
Встал и ушел.

Комментарий:
Известный физик - конечно, Стивен Хокинг. Надеемся, вы досидите тур до
конца.

Источник:
http://drugoe-kino.livejournal.com/3047266.html?thread=65087586#t65087586

Автор:
Артем Колесов

Вопрос 5:
Как пишет Евгений Морозов, в 90-х всемирная паутина казалась романтичной
и неизведанной, что отразилось в названии ИКСА. Назовите ИКС двумя
словами.

Ответ:
Internet Explorer.

Источник:
https://www.readability.com/articles/nxri0fo8

Автор:
Антон Пинчук

Вопрос 6:
Рассказывая о возможностях дракона, Огастес Браун упоминает скрип двери
и мяуканье кошки. Кого мы заменили драконом?

Ответ:
Пересмешника.

Зачет:
Пересмешник.

Комментарий:
Пересмешник может имитировать не только голоса птиц, но и скрип двери, а
также мяуканье кошки. "Убить дракона" и "Убить пересмешника" -
знаменитые экранизации.

Источник:
Огастес Браун. Почему панда стоит на голове?
http://www.flibusta.net/b/288520/read

Автор:
Дмитрий Сахаров

Вопрос 7:
В песне "Полька-байка" группы "Green Crow" повествуется о том, как
лирический герой, перебравшийся из Дублина в Бирмингем, спит с шестью
мужчинами. Напишите шесть букв, которые мы пропустили в предыдущем
предложении.

Ответ:
гастар.

Комментарий:
Песня называется "Полька-гастарбайка" и повествует о нелегкой судьбе
трудового народа, и спят герои песни всемером на одной кровати вовсе не
потому, почему вы могли подумать, а просто денег нет.

Источник:
http://webkind.ru/text/595958627_99362393p656475520_text_pesni_polka-gastarbajka.html

Автор:
Руслан Хаиткулов

Вопрос 8:
По словам Софии Губайдулиной, утверждение Брамса о том, что скорее
мужчина сумеет родить ребенка, чем женщина сможет создать хорошую
музыку, сейчас неактуально. В подтверждение своих слов Губайдулина
приводит историю альфы. Какое слово мы заменили в этом вопросе?

Ответ:
Арфы.

Зачет:
Арфа.

Комментарий:
София Губайдулина - композитор, что, впрочем, знать совсем не
обязательно. По ее словам, если, например, в Древней Греции на арфе
играли только мужчины, то в наши дни можно встретить только арфисток.
Соответственно, Губайдулина приводит в пример историю арфы.

Источник:
http://www.viperson.ru/wind.php?ID=277118

Автор:
Антон Волосатов

Вопрос 9:
Жена героя романа Сафрана Фоера сбегает с городским помещиком.
Прощальная записка жены сильно огорчает героя. В ходе дальнейших событий
записка начинает напоминать предмет из романа первой половины XX века.
Кому этот предмет принадлежал?

Ответ:
Фриде.

Комментарий:
Герой хочет спрятать записку или потерять ее, но она каждый раз без
всякой мистики снова попадается ему на глаза, заставляя раз за разом
переживать огорчения. Фриде, задушившей платком ребенка, каждый день
подкладывают этот самый платок на тумбочку.

Источник:
   1. Джонатан Сафран Фоер. Полная иллюминация.
   2. http://ru.wikipedia.org/wiki/Мастер_и_Маргарита#.D0.A4.D1.80.D0.B8.D0.B4.D0.B0

Автор:
Дмитрий Сахаров

Вопрос 10:
Отмечая назидательный характер мексиканской живописи, Петр Вайль писал,
что эпики были этиками. Кого мы заменили эпиками и этиками?

Ответ:
Муралисты, моралисты.

Комментарий:
Мурализм - движение в монументальной живописи Мексики второй четверти XX
века. Монументализм - в каком-то роде синоним эпичности, а мораль -
этики. :-) Кстати, одним из самых известных представителей мексиканского
мурализма был Диего Ривера, муж Фриды Кало.

Источник:
Петр Вайль, цикл телепередач "Гений места".

Автор:
Руслан Хаиткулов

Вопрос 11:
Герой одного фильма обращается к своему малознакомому собеседнику:
"Сударь, до [ПРОПУСК] нам еще полверсты". Восстанови пропущенное слово.

Ответ:
Ты.

Комментарий:
Герой фильма "Пена дней" говорит в рифму: "Сударь, до "ты" нам еще
полверсты". Своеобразный аналог фразы "Мы с вами на брудершафт еще не
пили".

Источник:
Фильм "Пена дней", реж. Мишель Гондри.

Автор:
Артем Колесов

Вопрос 12:
Слово "ОНИ" в вопросе заменяет несколько слов.
   В абсурдистском романе Бориса Виана фигурирует гардероб, в котором
вместо вешалок используются ОНИ. На чьих концертах в 2012 году также
использовались ОНИ?

Ответ:
Леди Гага.

Комментарий:
ОНИ - крюки для подвешивания мяса. Вместо вешалок у Виана использовались
крюки для мяса. В мае 2012 года во время тура в поддержку своего
студийного альбома "Born This Way" Леди Гага дала сольный концерт в
Токио. Одной из декораций послужили подвешенные на крюках туши коров.
Правда, туши были пластмассовые. :-)

Источник:
   1. Борис Виан. Пена дней. http://www.flibusta.net/b/133416/read
   2. http://ru.wikipedia.org/wiki/Мясное_платье_Леди_Гаги

Автор:
Дмитрий Сахаров

Вопрос 13:
В XIX веке французские повара изобрели способ отбивания мяса, не
нарушающий его внешнего вида. Назовите профессию людей, которые почти
сразу начали пользоваться этим способом в своих целях.

Ответ:
Жандармы.

Зачет:
Жандармерия, полиция, полицейские, дознаватели, следователи.

Комментарий:
Инструмент стало удобно использовать при пытках.

Источник:
Вильям Похлёбкин. Кухня века. http://www.flibusta.net/b/339182/read

Автор:
Антон Пинчук

Вопрос 14:
Дуплет.
   1. "Оценка позиции алгоритмизируется плохо". "Эффективная реализация
влияния - один из важнейших стратегических элементов, но столь же важным
является понимание момента, когда от схватки за влияние следует
переходить к окончательному оформлению территории". В каждой из этих
цитат мы пропустили одни и те же два коротких слова. Напишите их.
   2. "Самым большим барабаном является изготавливаемый из дзельквы
тайко высотой 14 см и диаметром 28 см". "Сценическая речь стилизована и
неразборчива". "Движения имеют медленный темпоритм". В каждой из этих
цитат мы пропустили одни и те же два коротких слова. Напишите их.

Ответ:
   1. В го.
   2. В но.

Комментарий:
Надеемся, вы еще сохраняете трезвость ума. Цитаты в первом вопросе
дуплета взяты из статьи об игре го, цитаты во втором вопросе дуплета -
из статьи о театре но.

Источник:
   1. http://ru.wikipedia.org/wiki/Го
   2. http://ru.wikipedia.org/wiki/Но

Автор:
Артем Колесов

Вопрос 15:
В одном из эпизодов романа "Прощай, оружие!" рядом с героем разрывается
снаряд, после чего слышен стук падающих ИХ. Чуть позже герой признаёт,
что действительно испугался. Назовите ИХ.

Ответ:
Кирпичи.

Зачет:
Кирпичики.

Комментарий:
Без комментариев.

Источник:
Эрнест Хемингуэй. Прощай, оружие! http://www.flibusta.net/b/153877/read

Автор:
Анна Бахарева

Вопрос 16:
Дуплет. Два вопроса по 30 секунд обсуждения на каждый. Ответ на первый
вопрос дуплета должен быть написан в левой части бланка, ответа на
второй вопрос дуплета - в правой.
   1. ЕГО история началась так: вЫрезали белую часть, оставив две
другие. Назовите ЕГО двумя словами.
   2. Крест на НЕМ подчеркивает, что бОльшая часть населения страны -
католики, в отличие от соседей. Назовите ЕГО тремя словами.

Ответ:
   1. Флаг Гаити.
   2. Флаг Доминиканской республики.

Зачет:
   1. Гаитянский флаг.

Комментарий:
   1. Красный и синий цвета полос флага восходят к цветам полос на флаге
Франции. В данном случае они выступают в качестве символического
воплощения союза мулатов и чернокожих. Восставшие на Гаити рабы вЫрезали
не только белую полосу, но и часть белого населения.
   2. Большинство населения Доминиканской республики - католики, а
гаитяне - вудуисты.
   Форма ответа - подсказка: государство Гаити находится на западе
острова Гаити, а Доминиканская Республика - на востоке.

Источник:
http://www.vokrugsveta.ru/vs/article/1305/

Автор:
Павел Бордачев, Дмитрий Сахаров

Вопрос 17:
(pic: 20150165.jpg)
   Чтобы использовать эту фотографию в рекламных целях, одна фирма
совершила над ней некое действие. Кто совершил аналогичное действие в
первой четверти XX века?

Ответ:
[Сергей] Эйзенштейн.

Комментарий:
Вам была роздана фотография покорителя Южного полюса норвежца Оскара
Вистинга. На черно-белой фотографии, если не знать, кто на ней
изображен, трудно определить, какой флаг перед нами - исландский или
норвежский. Норвежский производитель пищевых добавок для животных
"Obersten" раскрасил флаг на упаковках своей продукции. Эйзенштейн в
фильме "Броненосец "Потёмкин"" раскрасил флаг вручную.

Источник:
   1. "Вокруг света", 2014, N 11. - С. 100.
   2. http://ru.wikipedia.org/wiki/Броненосец_%C2%ABПотёмкин%C2%BB_(фильм)

Автор:
Дмитрий Сахаров

Вопрос 18:
(pic: 20150166.jpg)
   Эта пилотка, популярная в годы Второй мировой войны, была названа в
честь объекта, являющегося частью единственного национального парка....
Какой страны?

Ответ:
Словении.

Комментарий:
Эта пилотка словенских партизан-антифашистов из-за характерной формы
была прозвана триглавкой в честь национального символа Словении - горы
Триглав. Эта гора изображена, кстати, и на флаге Словении. :-)

Источник:
http://ru.wikipedia.org/wiki/Триглавка

Автор:
Дмитрий Сахаров

Вопрос 19:
На картинке, призывающей задуматься о судьбе вымирающего албанского
языка, изображен ОН. Своим названием ОН обязан президенту Американского
музея естественной истории. Назовите ЕГО.

Ответ:
Тираннозавр.

Зачет:
Т. Рекс, Тираннозавр Рекс.

Комментарий:
Название переводится как "королевский ящер-тиран". Президент
Американского музея естественной истории Осборн подчеркивал размеры
животного и доминирование над другими животными того времени. На
картинке-меме "задумчивый динозавр" тираннозавр задумался, не ждет ли
тезаурус Тираны участь тираннозавров.

Источник:
   1. В вопросе.
   2. http://ru.wikipedia.org/wiki/Тираннозавр

Автор:
Артем Колесов

Вопрос 20:
Георгий Михайлович в 2011 году удостоился медали за заслуги в освоении
космоса. При этом Гречко - глубоко религиозный человек: на февраль 2014
года в его доме были сотни икон, евангелий и мощей святых. Какое слово
мы заменили в этом вопросе?

Ответ:
Пшонка.

Зачет:
Через "е" и "ё" тоже зачтем. И с "о" на конце.

Комментарий:
В первом предложении вопроса замен не было, космонавт Георгий Михайлович
Гречко действительно удостоился медали за освоение космоса. А вот во
втором предложении мы заменили фамилию Пшонка фамилией Гречко. Виктор
Пшонка - украинский прокурор, тоже занимавшийся освоением. 24 февраля
2014 года активисты Евромайдана попали в особняк бывшего генерального
прокурора Украины Виктора Пшонки. А еще Гречко и Пшонка - это записанные
по-олбански гречка и пшёнка.

Источник:
   1. http://ru.wikipedia.org/wiki/Гречко,_Георгий_Михайлович
   2. http://ru.wikipedia.org/wiki/Пшонка,_Виктор_Павлович

Автор:
Артем Колесов

Вопрос 21:
Внимание, ПЕРВАЯ и ВТОРАЯ заменяют словосочетания из четырех слов.
   По мнению Александра Шевченко, характеристика "ПЕРВАЯ" больше
подходит к периоду до 1917 года, а "ВТОРАЯ" - после. "ПЕРВАЯ" - название
фильма времен развала Советского Союза. Напишите два глагола, которые
присутствуют, соответственно, в ПЕРВОЙ и ВТОРОЙ.

Ответ:
Потеряли, сломали.

Зачет:
В любой форме, в любом порядке.

Комментарий:
Россией, которую мы потеряли, можно назвать дореволюционную Россию. А
послереволюционный период - Россия, которую мы сломали. Сочетание
глаголов отсылает к известному анекдоту про два титановых шарика.

Источник:
   1. Разговор автора вопроса с Александром Шевченко, 24.05.2014 г.
   2. http://lurkmore.to/Россия,_которую_мы_потеряли
   3. http://www.anekdot.ru/id/-9937345/

Автор:
Антон Волосатов

Вопрос 22:
В одной статье рассказывается о том, что Яндексу приходится в украинской
версии своих карт показывать Крым территорией Украины, а в русской
версии - территорией России. Забавно, что в веб-адресе этой статьи
фигурируют два предмета. Какие?

Ответ:
Стулья.

Зачет:
Стульчики.

Комментарий:
Россия, которую мы обрели. :-) Статья то ли намекает на попытку Яндекса
усидеть на двух стульях, то ли просто на нелегкий выбор в ситуации,
когда есть два стула.

Источник:
https://tjournal.ru/50180-yandex-two-chairs

Автор:
Руслан Хаиткулов

Вопрос 23:
В названии статьи kommersant.ru о признании прав России на весь Охотский
шельф упоминается "ОНА" моря. "ОНА" - подпись к демотиватору, на котором
изображена свисающая с потолка петля. Назовите ЕЕ двумя словами.

Ответ:
Последняя капля.

Зачет:
Последняя капелька.

Комментарий:
Статья называлась "Последняя капля моря". А силуэт петли напоминает
каплю.

Источник:
   1. http://www.kommersant.ru/doc/2347321
   2. http://demotivators.to/p/70289/poslednyaya-kaplya-.htm

Автор:
Елена Сидорова, Павел Бордачев

Вопрос 24:
[Ведущему: обозначить цитату, кавычки не озвучивать.]
   "Вы слышите? На улице играют "ВсЕнощную!"" - в этой фразе пропущено
слово из трех букв. Назовите того, кто ее произнес незадолго до смерти в
одной из американских больниц.

Ответ:
[Сергей Васильевич] Рахманинов.

Комментарий:
Пропущенное слово - "моЮ". Россию в лице Рахманинова мы тоже потеряли.
Почти всю вторую половину жизни композитор провел на чужбине. Обработки
Рахманиновым церковной музыки обрели большую популярность как в
богослужениях, так и в светских выступлениях. Незадолго до смерти
композитор услышал, что играют его "Всенощное бдение". А наше с вами
бдение продолжается.

Источник:
http://www.kicl.ru/archives/1910

Автор:
Антон Волосатов

Тур:
18 тур. "Абажур Ожегова" (Москва)

Вопрос 1:
(pic: 20150167.jpg)
   На представленных двух картинах английский художник Джон Кольер
изобразил первую и вторую. Назовите обеих в правильном порядке.

Ответ:
Лилит, Ева.

Источник:
   1. http://www.wikiart.org/en/john-collier/lilith-with-a-snake-1886
   2. http://www.wikiart.org/en/john-collier/eve-1911

Автор:
Алина Нестеренко

Вопрос 2:
Рик Ли был большим поклонником Элвиса Пресли и, в частности, его хита
1956 года "Heartbreak Hotel". В 1966 году группа Рика получила название,
которое в переводе на соответствующий язык можно обнаружить в названии
известного романа. Назовите автора этого романа.

Ответ:
[Александр] Дюма.

Комментарий:
Группа называлась "Ten Years After".

Источник:
http://bandnaming.blogspot.ru/2013/04/ten-years-after-band-name-meaning.html

Автор:
Анастасия Грибановская

Вопрос 3:
Известной балерине Матильде Кшесинской в начале XX века был преподнесен
Высочайший подарок - браслет в виде некоего животного. А кто, согласно
распространенной версии, помогал Николаю II выбирать подарок?

Ответ:
Императрица.

Зачет:
Александра Федоровна.

Комментарий:
Согласно воспоминаниям фрейлины Анны Вырубовой, когда Николай II,
увлекшийся балериной, решил преподнести ей подарок, императрица сперва
приревновала, а потом предложила помочь с выбором и указала на браслет в
виде змеи с крупным сапфиром и бриллиантами.

Источник:
   1. И.В. Зимин. Царские деньги. Доходы и расходы Дома Романовых.
http://www.flibusta.net/b/242726/read
   2. А.А. Вырубова. Страницы моей жизни.
http://www.flibusta.net/b/242568/read

Автор:
Анастасия Грибановская

Вопрос 4:
(pic: 20150168.jpg)
   На раздатке социальная реклама Всемирного фонда дикой природы в
защиту животных. Подпись в правом верхнем углу: "Мода требует больше
жертв, чем вы думаете". Что мы закрыли от вас?

Ответ:
[Размеры] S, XL.

Зачет:
По смыслу.

Источник:
http://www.ontare.ru/shokiruyushhie-reklamnyie-kampanii-o-problemah-zhivotnyih/

Автор:
Алексей Авраменко

Вопрос 5:
Дуплет.
   1. Согласно "Словарю Сатаны" Амброза Бирса, на одном языке это -
красивая женщина, на другом - смертельный яд, что демонстрирует
удивительное сходство языков. Что это?
   2. Согласно "Словарю Сатаны" Амброза Бирса, ИКС - Бог ведущей религии
мира, храм которой находится в священном городе Нью-Йорке. Назовите ИКС.

Ответ:
   1. Белладонна.
   2. Маммона.

Источник:
Амброз Бирс. Словарь Сатаны. http://www.flibusta.net/b/293076/read

Автор:
Алина Нестеренко

Вопрос 6:
ИКС Вильгельма - часто используемый в кино и телевидении эффект. Помимо
"Звездных войн", его можно встретить в таких фильмах, как "Бешеные псы",
"Убить Билла", "Доказательство смерти", "Бесславные ублюдки". В одной из
серий сериала "Волшебники из Вэйверли Плэйс" при обсуждении "ИКСА"
появилась версия, что одному человеку просто очень сильно жмут ботинки.
Назовите ИКС.

Ответ:
Крик.

Источник:
   1. http://ru.wikipedia.org/wiki/Крик_Вильгельма
   2. http://lurkmore.to/Крик

Автор:
Илья Любич

Вопрос 7:
В тезисе из четырех слов отчетного доклада Л.И. Брежнева на XXVI съезде
КПСС, ставшем политическим лозунгом, несмотря на первое впечатление,
тавтологии нет, если правильно понимать использованные в нем термины. На
какое слово заменено первое слово этого тезиса в заголовке статьи о
специальном "зеленом" шрифте, на 20% сокращающем расход тонера?

Ответ:
Экология.

Комментарий:
Брежнев сказал: "Экономика должна быть экономной". Статья озаглавлена
"Экология должна быть экономной".

Источник:
http://www.brainbang.ru/note/489/

Автор:
Анастасия Грибановская

Вопрос 8:
(pic: 20150169.jpg)
   Давид Бонацци известен своими философскими иллюстрациями на
социальные темы. На представленной вам иллюстрации автор размышляет о
свободе. Во что превращаются птицы с проводами на закрытой от вас части
иллюстрации?

Ответ:
Колючая проволока.

Комментарий:
(pic: 20150170.jpg)

Источник:
http://www.adme.ru/tvorchestvo-hudozhniki/30-ironichnyh-nablyudenij-o-nashem-mire-v-illyustraciyah-692305/

Автор:
Алексей Авраменко

Вопрос 9:
Фенеки обладают очень маленьким телом и большими ушами-локаторами. Уши
фенеков занимают 20% поверхности тела и позволяют слышать перемещения
насекомых даже под землей, а также избежать перегрева благодаря большой
площади кровеносных сосудов. Как называется самое известное литературное
произведение, в котором прототипом одного из главных героев послужил
фенек?

Ответ:
Маленький Принц.

Источник:
   1. http://ru.wikipedia.org/wiki/Фенек
   2. Журнал "Geo", февраль 2015 г.

Автор:
Анастасия Кривенкова

Вопрос 10:
Киты, как известно, общаются друг с другом с помощью пения. Обычно киты
поют свои песни на частоте от 10 до 20 герц, но в мире существует
уникальная особь, чье пение раздается на частоте 52 герца. Этого кита
называют самым одиноким в мире, потому что его сородичи не могут его
понять. Ученым, изучавшим кита, приходили письма от ТАКИХ людей, в
которых они высказывали версию, что кит тоже является ТАКИМ. Каким?

Ответ:
Глухим.

Источник:
http://ru.wikipedia.org/wiki/Пятидесятидвухгерцевый_кит

Автор:
Алина Нестеренко

Вопрос 11:
В фильме Вуди Аллена "Разбирая Гарри" звучит такой диалог:
   - У тебя нет моральных ценностей, вся твоя жизнь - сплошной нигилизм,
цинизм, сарказм и оргазм.
   - Кстати, во Франции с таким лозунгом легко СДЕЛАТЬ ЭТО.
   Какие три слова мы заменили на "СДЕЛАТЬ ЭТО", если известный
голливудский актер делал это дважды?

Ответ:
Победить на выборах.

Комментарий:
Речь идет о Шварценеггере.

Источник:
   1. http://ru.wikiquote.org/wiki/Разбирая_Гарри_(фильм)
   2. http://ru.wikipedia.org/wiki/Шварценеггер,_Арнольд

Автор:
Анастасия Грибановская

Вопрос 12:
(pic: 20150171.jpg)
   Этот рекламный плакат был сделан по случаю одной популярной ежегодной
акции. Назовите ее.

Ответ:
Час Земли.

Комментарий:
Скрыта надпись "For once, the Dark Side saves the planet".

Источник:
http://www.brainbang.ru/note/490/

Автор:
Анастасия Грибановская

Вопрос 13:
По поводу происхождения названия этой американской рок-группы существует
множество версий. По одной из них, оно появилось, когда музыканты
подъехали поздно вечером к придорожной пиццерии с неисправно работающей
вывеской. Назовите группу.

Ответ:
"ZZ Top".

Комментарий:
Согласно этой версии, автобус с голодными музыкантами остановился у
неоновой вывески "PIZZA STOP" с выключенными "PI", "A" и "S".

Источник:
http://bandnaming.blogspot.ru/2013/04/zz-top-band-name-meaning.html

Автор:
Анастасия Грибановская

Вопрос 14:
В конце фильма "Залечь на дно в Брюгге" один из героев умирает в городе,
который успел возненавидеть за время пребывания в нем. Последние дни
своей жизни он сравнивает с ИКСОМ. ИКСОВАЯ версия тетриса имеет
закругленное дно. Что мы заменили словом "ИКС"?

Ответ:
Ад.

Источник:
   1. http://ru.wikiquote.org/wiki/Залечь_на_дно_в_Брюгге
   2. http://www.kongregate.com/games/banthar/hell-tetris

Автор:
Илья Любич

Вопрос 15:
В одном учебнике английского языка есть фраза, дословно переводящаяся
как "Если вы сидите на заборе, то вы ДЕЛАЕТЕ ЭТО. Однако рано или поздно
вам придется слезть справа или слева". Ответьте, что такое "ДЕЛАТЬ ЭТО",
тремя словами, начинающимися на три последовательные буквы.

Ответ:
Откладывать принятие решения.

Комментарий:
Sit on the fence = откладывать принятие решения.

Автор:
Роман Игла

Вопрос 16:
В фильме Вуди Аллена "Разбирая Гарри" один из персонажей появляется в
кадре на несколько минут, чтобы подосадовать на то, что он ТАКОЙ, и что
это очень мешает ему в его карьере актера. "Когда мир ТАКОЙ" - заголовок
статьи о тренировке зрения. Какие три слова мы заменили на "ТАКОЙ"?

Ответ:
Не в фокусе.

Источник:
   1. Фильм "Разбирая Гарри" (1997).
   2. Газета "Помоги себе сам", апрель 2005 г.

Автор:
Анастасия Грибановская

Вопрос 17:
В одном из выпусков юмористического проекта "Epic Rap Battles of
History" этот известный человек, ссылаясь на свою работу, заявляет, что
он крут как два обычных рэпера. Назовите этого человека.

Ответ:
Эйнштейн.

Комментарий:
E (instein) = MC^2. MC [эм-си] (сокр. англ. Master of Ceremonies), в
регги-культуре и хип-хопе - артист, в сопровождении электронной
танцевальной музыки произносящий со сцены слова - заранее сочиненные или
импровизированные.

Источник:
http://genius.com/408252

Автор:
Илья Любич

Вопрос 18:
По окончании перелетов на "Боингах" стюардессы российских авиалиний
иногда обнаруживают пропажу ИХ и находятся в недоумении. Назовите ИХ.

Ответ:
Спасательные жилеты.

Комментарий:
В журнале сообщается, что большинство людей при поимке заявляет, что
любят в них рыбачить.

Источник:
Журнал "Geo", февраль 2015 г.

Автор:
Анастасия Кривенкова

Вопрос 19:
(pic: 20150172.jpg)
   Доля резюме с ЭТИМ от общего количества представлена на диаграмме. С
чем?

Ответ:
С фотографией.

Источник:
http://www.career.ru/article/15908

Автор:
Дмитрий Лобанов

Вопрос 20:
Герой Орсона Скотта Карда пишет несколько политических статей под
псевдонимом, который он взял в честь мыслителя XVII века. Но переводчик
не понял хитрого авторского приема и решил, что псевдоним герой выбрал в
честь бога. Назовите и мыслителя, и бога.

Ответ:
[Джон] Локк, Локи.

Источник:
http://ru.wikipedia.org/wiki/Локк,_Джон

Автор:
Илья Любич

Вопрос 21:
Кельвин Кляйн считается одним из самых скандально известных новаторов в
моде. Он известен провокационной рекламой с участием практически
обнаженных несовершеннолетних и тем, что ввел в моду стиль "унисекс".
Одна из его идей была настолько возмутительной, что имела небывалый
успех. Она не давала модникам всего мира забыть его имя ни днем, ни
ночью. Какая?

Ответ:
Его имя на резинке трусов.

Зачет:
По смыслу.

Автор:
Анастасия Кривенкова

Вопрос 22:
В сериале "Сверхъестественное" в серии "Девушка с татуировкой драконов"
главные герои встречают хакершу Чарли, которая помогает им сражаться с
их врагами Левиафанами. Она постоянно думает, как бы поступили на ее
месте герои различных фентезийных миров, как бы примеряя их образы на
себя, чтобы не струсить и не убежать в самый опасный и ответственный
момент. Какие два слова мы пропустили в первом предложении?

Ответ:
Подземелий и.

Комментарий:
Серия называется "Девушка с татуировкой Подземелий и Драконов". В D&D
люди обычно, наоборот, представляют себя на месте других героев. Чарли
является сильным гиком.

Источник:
http://www.imdb.com/title/tt1995114/

Автор:
Илья Любич

Вопрос 23:
(pic: 20150173.jpg)
   Перед вами иллюстрация к одной карте из коллекционной карточной игры
"Magic: The Gathering". Действие карты ход за ходом снимает жизни у всех
игроков, однако игроки могут свести эти повреждения к минимуму. Название
карты является отсылкой к проекту, начатому в 1947 году. Назовите этот
проект.

Ответ:
"Часы Судного дня".

Источник:
http://ru.wikipedia.org/wiki/Часы_Судного_дня

Автор:
Илья Любич

Вопрос 24:
Один из героев фильма "Кадиллак Рекордз" говорит: "Когда первая девчонка
сняла трусы и бросила их на сцену, там пели блюз. Когда дело дошло до
НИХ, это назвали рок-н-ролл". Какие два слова мы заменили на "НИХ"?

Ответ:
Белых девчонок.

Зачет:
По смыслу.

Источник:
Фильм "Кадиллак Рекордз" (2008).

Автор:
Анастасия Грибановская

Тур:
19 тур. "Hangover" (Архангельск - Северодвинск - Санкт-Петербург)

Вопрос 1:
Некий дворянин был вынужден уволиться с государственной службы, когда
перепутал ИКС с АЛЬФОЙ. Впрочем, такие случаи происходят до сих пор.
Так, Артемий Лебедев, собирая свою фотоколлекцию под названием "6666
рублей", тоже сначала перепутал ИКС и АЛЬФУ. Что мы заменили на ИКС и
АЛЬФУ?

Ответ:
Архангельск, Астрахань.

Комментарий:
Коллекция Лебедева посвящена денежным знакам; на купюре в 500 российских
рублей присутствует город Архангельск, однако Лебедев не первый, кто
перепутал этот город с Астраханью. Дворянин, сделавший это до него, -
Обломов.

Источник:
   1. Иван Гончаров. Обломов. http://www.flibusta.net/b/217953/read
   2. http://tema.livejournal.com/1788181.html

Автор:
Екатерина Макоед (Архангельск)

Вопрос 2:
Роберт Персиг в своей книге упоминает реакционные указы губернатора
Монтаны Дональда Наттера и замечает, впрочем, что в его случае настоящий
политик-радикал может позволить себе делать что угодно, потому что
оппозиция уже выставила себя... Закончите фразу одним словом.

Ответ:
"... ослами".

Комментарий:
Оппозиция была представлена демократами, символом которых является осел.

Источник:
Роберт Пирсиг. Дзен и искусство ухода за мотоциклом.
http://www.flibusta.net/b/42253/read

Автор:
Денис Никитенко (Архангельск - Санкт-Петербург)

Вопрос 3:
Хотя это млекопитающее не является хищником и питается преимущественно
насекомыми и плодами, знакомый автора вопроса утверждал, что оно
неожиданно напало на его команду, прямо посреди турнира по "Что? Где?
Когда?". Назовите это животное.

Ответ:
Тупайя.

Источник:
   1. Шутка знакомого автора вопроса.
   2. http://ru.wikipedia.org/wiki/Тупайи

Автор:
Сергей Коновалов (Санкт-Петербург)

Вопрос 4:
В осовремененной постановке трагедии Еврипида "Медея" в первом акте на
сцене появляются два спальника, которые впоследствии будут использованы
как ИКСЫ. На сайте vkostume.ru [в костюме точка ру] можно заказать
хэллоуинский вариант ИКСА с прорезями для ног и головы, сшитый из
100%-ного полиэстера. Назовите ИКС тремя или двумя словами.

Ответ:
Мешок для трупа / патанатомичский мешок.

Зачет:
По смыслу.

Комментарий:
Медея упаковывает тела убитых сыновей в спальники, как в
патанатомические мешки.

Источник:
   1. Постановка Королевского Национального театра, 2014.
   2. http://www.vkostume.ru/item/Meshok_dlya_tela/

Автор:
Екатерина Макоед (Архангельск)

Вопрос 5:
На пьедестале памятника этому человеку выбиты слова, исходя из которых
можно предположить, что это памятник либо руководителю еврейского народа
времен завоевания Ханаана, либо французскому сенатору XIX века. Назовите
человека, которому в действительности поставлен этот памятник.

Ответ:
Николай Коперник.

Комментарий:
Надпись гласит: "Остановившему Солнце, сдвинувшему Землю". Иисус Навин
остановил Солнце, а Жорж Дантес убил Пушкина - "Солнце русской поэзии".

Источник:
http://ru.wikipedia.org/wiki/Коперник,_Николай

Автор:
Виталий Егоров (Мурманск)

Вопрос 6:
Дуплет.
   1. Леонид Мацих утверждает, что умирающий икс в эмблеме одной из
масонских лож - символ разгадки всех тайн природы. В одном из слов
предыдущего предложения мы пропустили три буквы. Напишите их.
   2. Петер Идлинг пишет, что нагрудный карман красного кхмера с разным
количеством шариковых ручек выполнял ту же функцию, что и он в других
странах. Какие три буквы мы пропустили?

Ответ:
   1. с, ф, н.
   2. п, о, г.

Комментарий:
Соответственно, СФиНкс и ПОГон.

Источник:
   1. Л.М. Мацих. Серия программ "Братья" на радио "Эхо Москвы".
   2. Петер Фреберг Идлинг. Улыбка Пол Пота. - М.: Астрель, 2014.

Автор:
Денис Никитенко (Архангельск - Санкт-Петербург)

Вопрос 7:
Внимание, в вопросе есть замены.
   "ДВОРОВЫЕ ПСЫ" стали устойчивым выражением в январе 1948 года.
Кропоткина и Бакунина вполне можно назвать ПОРОДИСТЫМИ ПСАМИ. Назовите
человека, который считается первым ПСОМ.

Ответ:
Диоген.

Комментарий:
Дворовые псы - безродные космополиты, породистые псы - родовитые
космополиты, а первым космополитом называли Диогена. Псом его, впрочем,
тоже называли.

Источник:
   1. http://www.bibliotekar.ru/encSlov/2/33.htm
   2. http://ru.wikipedia.org/wiki/Кропоткин,_Пётр_Алексеевич
   3. http://ru.wikipedia.org/wiki/Бакунин,_Михаил_Александрович
   4. Диоген Лаэртский. О жизни, учениях и изречениях знаменитых
философов. http://www.psylib.org.ua/books/diogenl/txt06.htm

Автор:
Александр Цыганов (Архангельск)

Вопрос 8:
В вопросе ИКСОМ заменены несколько слов.
   Американцам, скорее всего, не был известен сериал "Coronation
Street", поэтому ИКС, который в Британии воспринимался как пародия на
узнаваемые женские образы из этого сериала, в США был запрещен. Назовите
любого из снявшихся в ИКСЕ в главных ролях.

Ответ:
Фредди Меркьюри / Брайан Мэй / Роджер Тейлор / Джон Дикон.

Комментарий:
Речь идет о клипе на песню "I want to break free", содержащем
провокационные образы.

Источник:
http://ru.wikipedia.org/wiki/I_Want_to_Break_Free

Автор:
Екатерина Макоед (Архангельск)

Вопрос 9:
Один пользователь Рунета, говоря о защите компьютерных программ,
пошутил, что на полях борьбы правообладателей и пиратов было пролито
много крови. Какие два слова мы заменили в тексте вопроса?

Ответ:
Сломано копий.

Комментарий:
Замена связана со смыслом идиоматических выражений "пролито крови" и
"сломано копий". Игра слов с отсылкой к пиратским копиям.

Источник:
http://bash.im/quote/420736/

Автор:
Иван Беляев (Архангельск)

Вопрос 10:
Как известно, для бразильцев нет ничего важнее футбола. Перед финальным
матчем Кубка страны газета "Estado de Minas" вышла с большим постером на
первой полосе, посвященным предстоящему матчу. Назовите короткое слово,
встречающееся во всех остальных статьях в этом выпуске.

Ответ:
Bla.

Зачет:
Бла.

Комментарий:
Редакция газеты, видимо, посчитала, что ничего важнее финала кубка нет,
поэтому все остальные статьи газеты состояли из слов "bla bla bla".

Источник:
http://portalmakingof.com.br/estado_de_minas_coloca_quotbla_bla_blaquot_na_capa_para_focar_no_classico_mineiro.html

Автор:
Сергей Коновалов (Санкт-Петербург)

Вопрос 11:
В романе Джоан Роулинг "Шелкопряд" герои посещают старинное заведение с
традиционной британской кухней. Место настолько кичится своей
консервативностью, что "в пику французам" это слово там не
употребляется. Назовите это слово.

Ответ:
Меню.

Комментарий:
Даже привычное и почти обязательное слово для любого места общественного
питания, имеющее французские корни, британцы стараются избегать.

Источник:
Джоан Роулинг. Шелкопряд.

Автор:
Екатерина Макоед (Архангельск)

Вопрос 12:
Сидя на собрании анонимных алкоголиков, Гомер Симпсон спрашивает, где
пончики. Когда ему указывают, он говорит: "Слишком далеко. Целых...".
Закончите фразу двумя словами.

Ответ:
"... двенадцать шагов".

Комментарий:
Широко известна программа "12 шагов", которую также часто называют одной
из самых эффективных для лечения и реабилитации страдающих зависимостью
людей.

Источник:
"Симпсоны", s11e18 "Days of Wine and D'oh'ses".

Автор:
Екатерина Макоед (Архангельск)

Вопрос 13:
В книге "Огнем и мечом" герой, рассуждая о военных успехах Речи
Посполитой, говорит: "Разгромила их как АЛЬФА, но простит их как БЕТА".
АЛЬФА-БЕТА - единственный в своем роде носитель звания Почетного
гражданина города Волгограда за особые заслуги в период Второй мировой
войны. Что мы заменили словами "АЛЬФА" и "БЕТА"?

Ответ:
Королева, мать.

Комментарий:
Звание "Почетный гражданин города-героя Волгограда" присвоено
Королеве-матери Великобритании Елизавете Виндзорской постановлением
Волгоградского городского Совета народных депутатов от 6 января 2000 г.
за особые заслуги в организации помощи Сталинграду жителями
Великобритании в период Второй мировой войны и развитии дружественных
связей с Россией. На данный момент это единственный случай присвоения
звания почетного гражданина города Волгограда иностранцу.

Источник:
   1. Г. Сенкевич. Огнем и мечом. http://www.flibusta.net/b/243372/read
   2. http://ru.wikipedia.org/wiki/Почётные_граждане_города-героя_Волгограда#2000.C2.A0.D0.B3.D0.BE.D0.B4

Автор:
Александра Ермолина (Архангельск)

Вопрос 14:
Фильм, посвященный убийству Кеннеди, называется "ИКС". Первый лондонский
ИКС вышел на улицы в 1829 году. Назовите ИКСА.

Ответ:
Бобби.

Комментарий:
Фильм посвящен убийству Роберта Кеннеди; Бобби - также прозвище
английских полицейских.

Источник:
   1. http://www.kinopoisk.ru/film/31918/
   2. http://www.vokrugsveta.ru/quiz/104/

Автор:
Екатерина Макоед (Архангельск)

Вопрос 15:
Внимание, в вопросе есть замена.
   В театральных постановках АЛЬФУ сыграли Мехрангиз Гасанова и Алсу
Густова, а в фильме - Татьяна Федотова, которая попалась на глаза
режиссера случайно. "Если бы Танечка не сбежала с уроков, - пишет
Людмила Андреева, - у АЛЬФЫ было бы совсем другое...". Закончите
предложение одним словом абсолютно точно.

Ответ:
"... личико".

Комментарий:
АЛЬФА - Гюльчатай из "Белого солнца пустыни". Широкую известность
получила фраза из фильма "Гюльчатай, открой личико!".

Источник:
   1. http://ru.wikipedia.org/wiki/Гюльчатай
   2. http://www.smena.ru/media/numbers/24803/24803.pdf

Автор:
Денис Никитенко (Архангельск - Санкт-Петербург)

Вопрос 16:
Биатлонист Бьорн Ферри летом 2014 года презентовал автобиографическую
книгу. Название книги состоит из двух слов и на слух практически
неотличимо от "имени" рекордсменки, установившей свой рекорд в России в
мае 2009 года. Воспроизведите название книги.

Ответ:
"Ferry Tales".

Зачет:
"Ferry Tale".

Комментарий:
Рекордсменка - песня Александра Рыбака "Fairytale", набравшая на
конкурсе Евровидение-2009 рекордное число очков.

Источник:
   1. http://www.sports.ru/biathlon/1021218863.html
   2. http://ru.wikipedia.org/wiki/Fairytale

Автор:
Сергей Коновалов (Санкт-Петербург)

Вопрос 17:
Несмотря на вполне устоявшуюся традицию изображать ЕЕ определенным
образом, у НЕЕ, находящейся в Центральном административном округе
Москвы, нет одного из важнейших атрибутов. Этот факт стал причиной для
множества шуток и обсуждений до такой степени, что несколько лет назад
депутаты предложили президенту начать одну из проводимых реформ именно с
этого. Ответьте максимально точно: что предложили сделать депутаты?

Ответ:
Завязать Фемиде глаза.

Комментарий:
ОНА - это Фемида. Статуя Фемиды на фасаде Верховного суда РФ изображена
вопреки устоявшейся традиции без повязки на глазах. Депутаты на встрече
с тогдашним президентом Дмитрием Медведевым предложили начать судебную
реформу с изменения статуи.

Источник:
http://lenta.ru/news/2011/05/13/femida/

Автор:
Александра Ермолина (Архангельск)

Вопрос 18:
В своей статье Дмитрий Быков рассуждает о новой волне целенаправленных
нападок на нобелевского лауреата и упоминает французский вариант его
имени и АЛЬФУ, отличающиеся друг от друга только одной буквой. Назовите
АЛЬФУ.

Ответ:
Мишень.

Комментарий:
Нобелевский лауреат - Михаил Горбачёв, обыгрывается созвучие "Мишель -
мишень".

Источник:
http://www.sobesednik.ru/dmitriy-bykov/20140416-dmitriy-bykov-kak-razvlech-rossiyu-yarche-suda-nad-gorbachev

Автор:
Денис Никитенко (Архангельск - Санкт-Петербург)

Вопрос 19:
В своей статье о Владимире Тарасенко Артем Зырянов пишет, что этот
хоккеист то забьет кистевым броском, то щелкнет, то подставит клюшку на
пятачке, то выдаст хитрый финт. Далее голы Тарасенко сравниваются с
АЛЬФАМИ. В известной песне АЛЬФА сравнивается с транспортным средством.
Назовите это транспортное средство тремя словами.

Ответ:
Огромный седой вертолет.

Комментарий:
Голы Тарасенко, по мнению автора статьи, не похожи друг на друга, как
снежинки. В песне из спектакля "День радио" есть строки "Над костром
пролетает снежинка, как огромный седой вертолет".

Источник:
   1. http://www.sports.ru/tribuna/blogs/centre/709069.html
   2. http://www.muzland.ru/songs.html?auth=244&song=5

Автор:
Сергей Коновалов (Санкт-Петербург)

Вопрос 20:
В вопросе словами "СДЕЛАЛ ЭТО" заменены два других слова.
   Автор вопроса получил небольшую травму и впервые за 6,5 лет СДЕЛАЛ
ЭТО. Назовите литературного персонажа, который не впервые СДЕЛАЛ ЭТО и
понял, что страшно устал, но его воля окрепла и на сердце стало легче.

Ответ:
Фродо Бэггинс.

Комментарий:
Автор вопроса повредил безымянный палец и вынужден был снять обручальное
кольцо, которое до этого носил не снимая.

Источник:
   1. ЛОАВ.
   2. http://www.flibusta.net/b/220980/read

Автор:
Сергей Коновалов (Санкт-Петербург)

Вопрос 21:
Англоязычное идиоматическое выражение "СДЕЛАТЬ ЭТО" с середины XX века
использовалось в среде любителей бейсбола для того, чтобы отметить
хорошую игру игрока обороняющейся команды. В настоящее время фраза
"СДЕЛАТЬ ЭТО" используется в смысле "достичь грандиозного и неожиданного
успеха" или "решить очень сложную задачу". Назовите человека, которому
удалось первым СДЕЛАТЬ ЭТО и не погибнуть.

Ответ:
[Бенджамин] Франклин.

Комментарий:
Выражение - to catch lightning in a bottle - поймать молнию в бутылку,
которое, собственно, изначально и пошло от его опыта с лейденской банкой
и атмосферным электричеством. Подобный опыт пытался провести Рихман, но
погиб от удара шаровой молнией.

Источник:
   1. http://en.wiktionary.org/wiki/lightning_in_a_bottle
   2. http://en.wikipedia.org/wiki/Leo_Durocher
   3. http://ru.wikipedia.org/wiki/Франклин,_Бенджамин
   4. http://ru.wikipedia.org/wiki/Рихман,_Георг_Вильгельм

Автор:
Иван Беляев (Архангельск)

Вопрос 22:
Мастерская в городе Петрозаводске, занимающаяся производством
средневековых предметов быта аутентичным способом из аутентичных
материалов, называется "Как спичка". Какие три слова мы заменили в
вопросе двумя?

Ответ:
Кожа да кости.

Комментарий:
Логика замены: выражения, обозначающие худобу: "Худой, как спичка" и
"Худой - кожа да кости".

Источник:
   1. https://vk.com/kozhadakosti
   2. http://dic.academic.ru/dic.nsf/dic_synonims/179561/

Автор:
Сергей Коновалов (Санкт-Петербург)

Вопрос 23:
Сергей Шнуров, комментируя происходящую, по его мнению, девальвацию
патриотизма, сказал, что патриотизм стал "предметом" массовой
культуры... оболочка стала внутренним содержанием, и сравнил российский
патриотизм с ИКСОМ, который также очень часто используют не по
назначению. Мы не спрашиваем, какие слова были заменены на ИКС. Назовите
основанную в 1948 году компанию, производящую популярные в России ИКСЫ.

Ответ:
"Adidas".

Комментарий:
Патриотизм, по мнению Шнурова, стал подобен спортивному костюму, который
предназначен для тренировок и занятий спортом, а используется многими в
нашей стране в качестве повседневной одежды.

Источник:
   1. http://lenta.ru/news/2014/10/06/leningrad/
   2. http://ru.wikipedia.org/wiki/Adidas

Автор:
Сергей Коновалов (Санкт-Петербург)

Вопрос 24:
Внимание, в вопросе есть замена.
   Несмелый герой Джоан Роулинг сравнивает первую скамью в церкви с
первыми местами на ОЛИВЬЕ, где нужно принимать на себя все обрывы и
виражи. Что мы заменили на "ОЛИВЬЕ"?

Ответ:
Американские горки.

Зачет:
Русские горки.

Комментарий:
"Оливье" - салат, называемый в Европе русским, как и американские горки,
однако герой сравнил скамью с более привычным понятием.

Источник:
Дж. Роулинг. Случайная вакансия.
http://www.e-reading.link/bookreader.php/1015597/Rouling_Dzhoan_-_Sluchaynaya_vakansiya.html

Автор:
Екатерина Макоед (Архангельск)

Тур:
20 тур. "Ночь реабилитации" (Москва - Санкт-Петербург)

Редактор:
1-12 - Владислав Король; 13-24 - Серафим Шибанов

Вопрос 1:
В стихотворении Арсения Тарковского ОНИ ходят "по слепым глазам
старухи". Назовите ИХ.

Ответ:
Мухи, мухи, мухи.

Зачет:
Мухи.

Комментарий:
Стихотворение Тарковского называется "Портрет".

Источник:
Арсений Тарковский. Портрет. http://www.stihi-rus.ru/1/Tarkovskiy/76.htm

Автор:
Владислав Король

Вопрос 2:
Внимание, в вопросе есть замена.
   Представители МЧС отметили, что памятник комсомольцам Сталинграда
обрушился из-за сильного ветра и красной плесени. Какие два слова
заменены в тексте вопроса?

Ответ:
Коррозия металла.

Комментарий:
Сам памятник не стальной, а бронзовый - проржавели поддерживающие его
металлические конструкции. "Красная плесень" и "Коррозия металла" -
популярные группы.

Источник:
http://ria.ru/incidents/20090419/168625659.html

Автор:
Владислав Король

Вопрос 3:
Дуплет.
   1. Состоящий из десяти треков альбом "Некрофилия" группы "Сексуальные
меньшинства" имеет подзаголовок "ТАКАЯ десятка". Какая - ТАКАЯ?
   2. В каком белорусском городе был записан этот альбом?

Ответ:
   1. Холодная.
   2. Могилев.

Комментарий:
   1. В противовес многочисленным "горячим десяткам".
   2. Информация с обложки компакт-диска.

Источник:
http://www.lebedev.com/MusicPhone/Review/golubitsky/sm-disk.htm

Автор:
Владислав Король

Вопрос 4:
Прожив чуть более 17 тысяч дней и ночей, в 2007 году известный человек
СДЕЛАЛ ЭТО. Жан-Бедель Бокасса СДЕЛАЛ ЭТО, пообщавшись с Муаммаром
Каддафи. Какие два слова заменены словами "СДЕЛАЛ ЭТО"?

Ответ:
Принял ислам.

Комментарий:
"Двадцать тысяч дней и ночей пройдет - человек родился, человек умрет".
После смертей Александра Литвиненко и Ильи Кормильцева (автора текста
песни "20.000") словосочетание "принять ислам" в значении "умереть"
стало мемом. После встречи с ливийским лидером в сентябре 1976 года
Бокасса принял ислам, однако спустя три месяца снова стал добрым
католиком.

Источник:
   1. http://ru.wikipedia.org/wiki/Кормильцев,_Илья_Валерьевич
   2. http://www.nautilus.ru/SONGS/S820.shtml
   3. http://lurkmore.to/Принял_ислам
   4. http://ru.wikipedia.org/wiki/Бокасса,_Жан-Бедель

Автор:
Владислав Король, Серафим Шибанов

Вопрос 5:
В одной из песен с альбома "Некрофилия" ПРОПУСК поясняется словами
"живая ты не в кайф". У этого вопроса тоже ПРОПУСК. Заполните пропуск
тремя словами.

Ответ:
Позишн намбер файф.

Зачет:
Position Number Five.

Комментарий:
Сама песня является творческим переосмыслением нетленной работы Кая
Метова "Position Number Two".

Источник:
   1. http://www.lebedev.com/MusicPhone/Review/golubitsky/gol03.htm
   2. http://www.lebedev.com/MusicPhone/Review/golubitsky/smd/02-position-number-five-zhivaja-ty-ne-v-kaif.mp3

Автор:
Серафим Шибанов

Вопрос 6:
[Ведущему: немного выделить слово "помешала".]
   Внимание, словами "АЛЬФА" и "БЕТА" заменены два других слова.
   Героям одного романа, путешествовавшим из столицы в провинцию,
устроиться на ночлег помешала АЛЬФА, увившая фасад здания гостиницы.
Один из них потребовал, чтобы фасад был непременно увит БЕТОЙ. Назовите
АЛЬФУ и БЕТУ в любом порядке.

Ответ:
Повилика, жимолость.

Комментарий:
Во время другого путешествия из столицы в провинцию звучали слова:
"Приготовленную таким образом смесь надо двадцать минут помешивать
веткой жимолости. Иные, правда, утверждают, что в случае необходимости
можно жимолость заменить повиликой. Это неверно и преступно". Некоторые
литературоведы считают этот фрагмент поэмы Венедикта Ерофеева отсылкой к
эпизоду романа Джерома К. Джерома (перевод 1958 года).

Источник:
Джером К. Джером. Трое в лодке (не считая собаки), перевод Э. Линецкой и
М. Донского. http://www.flibusta.net/b/345347/read

Автор:
Владислав Король

Вопрос 7:
Узнав о планируемом телемосте между мэром Киева и одним из депутатов
российской Государственной Думы, автор вопроса заметил, что ОН вышел на
новый технологический уровень. Назовите ЕГО тремя словами.

Ответ:
Бокс по переписке.

Комментарий:
Депутат Николай Валуев и мэр Виталий Кличко должны были поговорить о
политике. Однако телемост не состоялся.

Источник:
   1. ЛОАВ.
   2. http://lenta.ru/news/2014/09/19/valuev/

Автор:
Владислав Король

Вопрос 8:
В романе Йозефа Рота "Марш Радецкого" посетитель трактира заметил, что
ЕГО "белое одеяние... казалось пробитым бесчисленными дробинками". Чье
одеяние?

Ответ:
[Государь император] Франц-Иосиф [Первый].

Комментарий:
В романе "Марш Радецкого" описано начало Первой мировой войны в
Австро-Венгрии. Как и в более известном романе, висевший в трактире
портрет государя императора был засижен мухами. Но Ярослав Гашек написал
об этом раньше.

Источник:
Йозеф Рот. Марш Радецкого. http://www.flibusta.net/b/175490/read

Автор:
Владислав Король

Вопрос 9:
Дмитрий Галковский пишет, что русские издавна предпочитали квас, репу,
капусту и ржаной хлеб, стремясь опираться не на внешний мир, а на
внутренний. Следствием этого стало первенство в некой области. Назовите
уроженца Волынской губернии, с которым связано это первенство.

Ответ:
Сергей Королёв.

Комментарий:
В философском романе "Бесконечный тупик" известный социогуманитарный
мыслитель отметил, что "природная склонность к реактивной форме
движения" привела к тому, что русские первыми вышли в космос.

Источник:
Дмитрий Галковский. Бесконечный тупик (примечание к N 902).
http://www.samisdat.com/3/311-902.htm

Автор:
Владислав Король, Серафим Шибанов

Вопрос 10:
В романтической комедии "Частная школа" урок по теме "Механика полового
акта" ведет преподавательница с подходящей фамилией. Назовите
произведение английской литературы, несколько героев которого носят
фамилию, созвучную с фамилией преподавательницы.

Ответ:
"Ромео и Джульетта".

Комментарий:
Половой акт - копуляция. Фамилия героини Сильвии Кристель - Копулетта
(Copuletta).

Источник:
Фильм "Private School".

Автор:
Владислав Король

Вопрос 11:
Жак часто развлекался, кидая из окна золотые монеты, а его младший брат
Ахмед как-то получил на день рождения автомобиль "Порше". Назовите их
фамилию.

Ответ:
Бокасса.

Комментарий:
Большинство из более чем сорока детей эксцентричного диктатора носит
французские имена, а вот Ахмед родился в тот короткий период, когда
Бокасса был мусульманином.

Источник:
http://gazeta.aif.ru/_/online/aif/1085/06_01

Автор:
Владислав Король

Вопрос 12:
Герой песни Валерия Зуйкова "Нюхаю трусы" уверен, что его супруга на
курорте чувствует себя отлично и на НИХ не смотрит. Назовите ИХ.

Ответ:
Часы.

Комментарий:
Счастливые часов не наблюдают.

Источник:
Песня Валерия Зуйкова "Нюхаю трусы".

Автор:
Владислав Король

Вопрос 13:
Согласно одной шутке, основанной на недавних наблюдениях, озвучку
главного злодея мультфильма "Пингвины Мадагаскара" стоило поручить
британскому певцу. Какому?

Ответ:
Seal.

Зачет:
Сил.

Комментарий:
Seal - это морской котик на английском. Последние исследования доказали,
что морские котики проявляют к пингвинам своеобычный интерес.

Источник:
   1. http://samir-befish.livejournal.com/937.html
   2. http://ru.wikipedia.org/wiki/Пингвины_Мадагаскара
   3. http://ru.wikipedia.org/wiki/Seal
   4. http://www.bbc.com/earth/story/20141117-why-seals-have-sex-with-penguins

Автор:
Серафим Шибанов

Вопрос 14:
Город Бобруйск был основан в 1387 году. Помня об этом факте, некоторые
местные жители используют эту дату вместо... Чего?

Ответ:
14/88.

Зачет:
С чертой и без.

Комментарий:
На стикере фанатов бобруйского футбольного клуба вместо традиционного
для правой сцены 14/88 крупно написано 13/87.

Источник:
Стикер ФК "Белшина" на плафоне эскалатора станции метро "Площадь
Восстания" (Санкт-Петербург).

Автор:
Владислав Король

Вопрос 15:
Один из альбомов группы "Коловрат", известной своими радикальными
политическими взглядами, называется "Эра ЕЕ". Вероятно, в течение
следующей минуты многие из здесь присутствующих задействуют ЕЕ. Назовите
ЕЕ двумя словами, начинающимися на соседние буквы алфавита.

Ответ:
Правая рука.

Комментарий:
Вскинутая вверх правая рука считается нацистским приветствием. При
записи вопроса многие из вас задействуют правую руку, так как правшей по
статистике больше.

Источник:
   1. http://ru.wikipedia.org/wiki/Эра_правой_руки
   2. http://ru.wikipedia.org/wiki/Коловрат_(группа)
   3. http://ru.wikipedia.org/wiki/Правша

Автор:
Серафим Шибанов

Вопрос 16:
Председатель Московской областной организации общества "Динамо" Виктор
Пауков говорит, что под его руководством существенно повысился средний
уровень результатов подмосковных спортсменов. Например, лыжников, часами
плутающих по НЕМУ, в командах, тащемта, уже нет. Назовите ЕГО двумя
словами.

Ответ:
Химкинский лес.

Комментарий:
Рок-музыкант Паук, баллотировавшийся на пост мэра Химок, в свое время
поддержал вырубку местного леса, обещая на его месте сделать русский
Лас-Вегас.

Источник:
Интервью с председателем Московской областной организации ОГО ВФСО
"Динамо", начальником ГУ МВД России по Московской области
генерал-майором полиции Виктором Кузьмичом Пауковым, журнал "Динамо",
январь 2015 г.

Автор:
Антон Снятковский

Вопрос 17:
(pic: 20150174.jpg)
   Перед вами инструкция, следуя которой вы сможете в буквальном смысле
СДЕЛАТЬ ЭТО. Ответьте двумя словами, начинающимися на соседние буквы
алфавита, что такое "СДЕЛАТЬ ЭТО".

Ответ:
Склеить телку.

Комментарий:
Бумажную телку.

Источник:
(pic: 20150175.jpg)

Автор:
Серафим Шибанов

Вопрос 18:
Дуплет.
   1. На гербе Гвинеи, действовавшем до 1993 года, можно было найти ЕГО.
Против другого ЕГО в свое время выступили участники "Союза офицеров
России". Назовите ЕГО двумя словами.
   2. Судя по тексту известной песни, ОНИ были у юных распутных девиц. В
фильме 2001 года каждая из НИХ стоила 50 долларов США. Назовите ИХ тремя
словами.

Ответ:
   1. Зеленый слоник.
   2. Пять бутылок водки.

Комментарий:
В фильме Светланы Басковой офицеры ведут себя несколько сомнительно. В
первой части вопроса речь идет о песне Андрея Алексина, в которых есть
строчки "Было к вечеру у них / две дешевых шоколадки, / пять бутылок на
троих". Автор предполагает, что речь идет именно о бутылках водки,
поскольку дальше есть строчки "Где-то в загородной бане / телки водку
будут жрать". Во второй части имеется в виду другой фильм Светланы
Басковой.

Источник:
   1. http://en.wikipedia.org/wiki/Coat_of_arms_of_Guinea
   2. http://www.ridus.ru/news/150661 со ссылкой на
https://vk.com/wall-7660419_503
   3. http://www.karaoke.ru/song/4644.htm
   4. http://www.imdb.com/title/tt2653232/

Автор:
Серафим Шибанов, идея дуплета - Владислав Король

Вопрос 19:
   <раздатка>
   По рублених стiнах бiгали червонi вiдблиски вогню, що палав у камiнi,
в кованих канделябрах горiли свiчки. Бiля широкого лiжка стояли два
дерев'яних стiльцi та маленький стiл пiд темним сукном. На стiльцi перед
вогнем сидiв гном, який вже скинув дощовика. У кiмнатi пахло його мiцним
тютюном.
   &nbsp;
   Перу какого писателя принадлежит отрывок, который мы перевели на
   мову?
   </раздатка>

Ответ:
Ника Перумова.

Комментарий:
Это была загадка в духе "на "Ля" начинается...". Начинается на "Перу",
кончается на "мову". Отрывок из романа "Алмазный меч, деревянный меч".

Источник:
   1. Ник Перумов. Эльфийский клинок.
http://books.rusf.ru/xussr_mr/perumn04/perumn04.3.html
   2. Перевод Владислава Короля.

Автор:
Владислав Король, Серафим Шибанов

Вопрос 20:
На днях наркополицейские привлекли к ответственности москвича,
торговавшего запрещенными препаратами. В сейфе задержанного нашли
марихуану, гашиш и амфетамины. Напишите трехзначный пароль, который тот
установил на свой сейф.

Ответ:
228.

Комментарий:
Именно такой номер имеет статья, предусматривающая ответственность за
оборот наркотиков.

Источник:
http://www.fsknmsk.ru/news/release/parol-228.html

Автор:
Серафим Шибанов

Вопрос 21:
Прослушайте четверостишие:
   ... Чтоб расширялася ОНА
   В два полушария округлых,
   Где дверь запретная видна
   Пленительно в долинах смуглых...
   Назовите ЕЕ.

Ответ:
Спина.

Комментарий:
Данное стихотворение Михаила Кузмина, сами понимаете о чем.

Источник:
http://az.gay.ru/authors/russian/kuzmin.html

Автор:
Серафим Шибанов

Вопрос 22:
Блиц.
   1. Сайт edim-doma.ru [едим дома точка ру] говорит, что для
бостонского ЕГО понадобятся: 50 г сахара, 3 яичных желтка, 20 г муки, 20
г кукурузного крахмала, 300 мл молока, 1 чайная ложка ванильной эссенции
и бисквит. Назовите ЕГО.
   2. Самое длинное ОНО, изготовленное в префектуре Миэ, имеет размер
222 метра и вес 20 килограммов. На шее вряд ли удержишь. Назовите ЕГО
двумя словами.
   3. В стихотворении Татьяны Матвеевой упоминается ОНА заката.
Станислав Малярчук пишет, что некоторые из групп, выступавшие на
Вудстоке, исчезли в НЕЙ кислотного тумана. Назовите ЕЕ двумя словами.

Ответ:
   1. Кремовый пирог.
   2. Жемчужное ожерелье.
   3. Пурпурная дымка.

Источник:
   1. http://www.edimdoma.ru/retsepty/49792-bostonskiy-kremovyy-pirog-boston-cream-pie
   2. http://lenta.ru/news/2013/04/12/necklace/
   3. https://vk.com/wall-59758826_355
   4. http://www.jazzquad.ru/index.pl?act=PRODUCT&id=3275

Автор:
Серафим Шибанов

Вопрос 23:
Фамилия одного из сценаристов фильма, в котором кавалерийский отряд
сопровождает пассажиров через земли, кишащие бандитами, совпадает с
названием города. Назовите этот город.

Ответ:
Лион.

Комментарий:
В фильме действие крутится вокруг дилижанса. Сам фильм называется
"Дилижанс ярости" (впрочем, знать это не нужно). Сценаристом был Эрл
Лион, такой вот вышел "лионский дилижанс". :-)

Источник:
http://www.kinonews.ru/movie_73207/stagecoach-to-fury

Автор:
Серафим Шибанов

Вопрос 24:
[Ведущему: слово "гоккун" обязательно повторить по буквам как указано.]
   Японское слово "гоккун" [Георгий, Олег, Константин, Кузьма, Устин,
Николай] является звукоподражанием глотку. Ответьте словом, также
содержащим удвоенную "К", разновидностью чего является гоккун.

Ответ:
Буккакэ.

Зачет:
Буккакэру, буккейк; также с "е" вместо "э".

Комментарий:
Неспроста все шесть имен мужские.

Источник:
   1. http://ru.wikipedia.org/wiki/Гоккун
   2. http://ru.wikipedia.org/wiki/Буккакэ

Автор:
Серафим Шибанов

Тур:
21 тур. "ЗаПущинная наука" (Пущино)

Вопрос 1:
Хотя на дворе ночь, расслабляться не время.
   Цитата с сайта bash.im: "Я пролила ИКС на ноутбук, теперь он не
уходит в ЗЕТ". Какое слово мы заменили на ИКС, а какие два слова - на
ЗЕТ?

Ответ:
Кофе, спящий режим.

Источник:
http://bash.im/quote/422320/

Автор:
Данила Аладин (Пущино)

Вопрос 2:
Во Вьетнаме водители грузовиков часто прикрепляют светоотражающие знаки
на передний бампер. С большой популярностью какого транспорта это
связано?

Ответ:
Мотоциклетного.

Зачет:
По упоминанию мотоцикла, мотороллера, велосипеда.

Комментарий:
Чтобы встречные не приняли грузовик за два мотоцикла и не попытались
"проскочить" между ними.

Источник:
ЛНА.

Автор:
Данила Аладин (Пущино)

Вопрос 3:
Закончите стихотворение-порошок двумя словами, начинающимися на одну и
ту же букву:
   в своей дилемме о футболе
   и сексе с ольгой на столе
   влад предпочтенье отдал ...
   ...

Ответ:
оле оле

Источник:
https://vk.com/sandalporoshki?w=wall-31481258_91608

Автор:
Данила Аладин (Пущино)

Вопрос 4:
(pic: 20150176.jpg)
   Два слова, которые мы от вас скрыли, обозначают всемирную проблему.
Напишите эти слова.

Ответ:
Глобальное потепление.

Источник:
http://demotivatorium.ru/demotivators/d/43473/

Автор:
Алексей Дубровский (Пущино)

Вопрос 5:
Закончите двумя словами цитату с сайта bash.im: "Накупили продуктов.
Российская капуста, израильская морковка, голландский лук, английская
свекла, египетская картошка, белорусское сало. Так вот ты какой - ...".

Ответ:
"... украинский борщ".

Источник:
http://bash.im/quote/427173/

Автор:
Данила Аладин (Пущино)

Вопрос 6:
На одной картинке человек раскрывает печенья с предсказаниями. Лежащие
перед ним на столе бумажки из печений складываются в слова
стихотворения-порошка, первые три строчки которого такие:
   сегодня ждите неудачу
   забудьте слово хорошо
   несчастье будет бесконечным
   Ответьте, что написано на четвертой, не до конца извлеченной из
печенья, бумажке.

Ответ:
вам жо

Источник:
(pic: 20150177.jpg)

Автор:
Алексей Дубровский (Пущино)

Вопрос 7:
В 2006 году в городе Аракатака прошел референдум. Предлагалось изменить
название города, но, к сожалению, явка не достигла необходимого минимума
в 7400 человек, хотя 90% проголосовавших были не против добавить в свою
жизнь магического реализма. Какое название должен был получить город?

Ответ:
Макондо.

Комментарий:
Аракатака - родной город Гарсиа Маркеса. Магический реализм -
литературное направление, в котором работал нобелевский лауреат.

Источник:
http://www.mk.ru/social/article/2014/04/18/1016713-7-faktov-iz-zhizni-gabrielya-garsia-markesa-korrespondent-v-moskve-zapret-na-vezd-v-ssha-i-obet-molchaniya.html

Автор:
Евгения Крутинина (Пущино)

Вопрос 8:
Внимание, в вопросе заменена одна буква.
   Один из товарищей по команде автора вопроса на фестивале "ишАчил", а
другой - "ажИл". Мы не спрашиваем вас, кто из них чем занимался.
Ответьте, какое слово сокращают до замененной буквы.

Ответ:
Жюри.

Комментарий:
Один из участников команды входил в ИЖ фестиваля, другой - в АЖ,
соответственно родились неологизмы "ИЖачил" и "АЖил".

Источник:
ЛНА.

Автор:
Никита Поздняков (Пущино)

Вопрос 9:
(pic: 20150178.jpg)
   Перед вами самый расистский из антирасистских плакатов. Скрытая от
вас надпись гласит: "Внутри мы все ПЕРПЕНДИКУЛЯРНЫЕ". Какое слово мы
заменили?

Ответ:
Нормальные.

Комментарий:
Под "нормальным" автор плаката понимает белый цвет кожи. :-)

Источник:
http://joyreactor.cc/post/1753904

Автор:
Алексей Дубровский (Пущино)

Вопрос 10:
Согласно шутке с сайта bash.im, "глава российского государства, который
завел себе блог, войдет в историю как Дмитрий". Какие две буквы латиницы
мы пропустили?

Ответ:
LJ.

Комментарий:
LJ Дмитрий - как обладатель Живого Журнала.

Источник:
http://bash.im/quote/407684/

Автор:
Данила Аладин (Пущино)

Вопрос 11:
(pic: 20150179.jpg)
   Автор вопроса считает, что в дудле от 9 сентября 2014 года большая
буква "G", как ни странно, очень напоминает нимб. Кого мы от вас скрыли?

Ответ:
Льва Толстого.

Комментарий:
Странно, что художник нарисовал букву "G", напоминающую нимб, несмотря
на то что Толстой был отлучен от церкви.

Источник:
http://www.ruscur.ru/themes/0/00/57/5751.shtml?news/0/05/21/52180

Автор:
Мария Томашевская (Пущино)

Вопрос 12:
Компания "Valtec" торгует инженерной сантехникой: трубами, счетчиками
для воды... К своему большому разочарованию, автор вопроса не нашел в их
каталоге ЭТО. Ответьте абсолютно точно, какие два слова мы заменили на
"ЭТО".

Ответ:
Водяной чип.

Комментарий:
Главый квест в культовой игре "Fallout" - найти водяной чип,
произведенный компанией "Vault-Tec".

Источник:
   1. http://www.valtec.ru
   2. http://ru.fallout.wikia.com/wiki/Волт-Тек

Автор:
Данила Аладин (Пущино)

Вопрос 13:
Вот и прозвучала половина вопросов тура.
   Прослушайте перевод отрывка одной из песен британской группы "Arena":
""Возьми ребенка. Он твой!" - единственное, что она могла сказать...".
Какое имя собственное носит эта песня?

Ответ:
"Соломон".

Комментарий:
В тексте песни обыгрывается известная притча о младенце и двух женщинах,
в которой Соломон предложил рассечь ребенка пополам.

Источник:
http://www.mp3lemon.net/lyric/50955/

Автор:
Данила Аладин (Пущино)

Вопрос 14:
(pic: 20150180.jpg)
   Назовите музыкальный инструмент, который мы от вас скрыли.

Ответ:
Балалайка.

Комментарий:
Вот такой вот "Битлз" по-русски.

Источник:
http://kunst-camera.livejournal.com/24238.html

Автор:
Данила Аладин (Пущино)

Вопрос 15:
Блиц. В рассказе Акутагавы Рюноскэ "О-Гин" упоминаются заимствованные
имена и понятия.
   1. Как по-русски мы называем того, кто в рассказе упоминается как
Сан-Дзеан Батиста?
   2. Как по-русски мы называем того, кто в рассказе упоминается как
Габуриэру?
   3. Как по-русски мы называем то, что в рассказе упоминается как
сагурамэнто?

Ответ:
   1. Иоанн Креститель.
   2. Гавриил.
   3. Крещение.

Зачет:
   3. Причастие.

Комментарий:
Рассказ повествует о первых христианах в Японии. И японские термины суть
искажения португальских, т.к. именно они первыми из европейцев
взаимодействовали с японцами.

Источник:
Акутагава Рюноскэ. О-Гин. http://www.flibusta.net/b/121845/read

Автор:
Данила Аладин (Пущино)

Вопрос 16:
(pic: 20150181.jpg)
   Воспроизведите надпись под демотиватором, несильно отличающуюся от
устойчивого выражения.

Ответ:
Пирамида непотребств [Маслоу].

Зачет:
Пирамида непотребностей [Маслоу].

Источник:
http://demotivators.to/p/968817/piramida-nepotrebstv-maslou.htm

Автор:
Алексей Дубровский (Пущино)

Вопрос 17:
Внимание, в вопросе есть замена.
   На обложке книжки "Жесткий тайм-менеджмент" циферблаты часов
заключены в ножницы. Какое слово мы заменили?

Ответ:
Наручники.

Комментарий:
(pic: 20150182.jpg)

Источник:
http://www.livelib.ru/book/1000596852

Автор:
Ирина Алисова (Пущино)

Вопрос 18:
Автор вопроса - не сторонник садомазохизма, но статья в журнале "Maxim"
его заинтересовала. В начале статьи журналист сокрушается: "Пусть ...
люди стали в целом добрее друг к другу, зато из их жизни ушла
изобретательность", а затем предлагает вспомнить лучшие из хитроумных и
жестоких пыток древности. Названием статьи стала русская пословица из
четырех слов, в которой заменили первую букву. Как называется эта
статья?

Ответ:
"Боль на выдумки хитра".

Источник:
"Maxim", май 2007 г. - С. 120.

Автор:
Олег Холодов (Серпухов)

Вопрос 19:
Приведенная на одном из рисунков пирамида потребностей содержит, как ни
странно, четыре уровня, верхний из которых называется "Уничтожение
человечества". Всё дело в том, что вместо правильного содержания второго
сверху уровня на рисунке показаны белые буквы на синем фоне. Третий же и
четвертый уровни этой пирамиды пункта приведены в соответствие ИМ.
Назовите ИХ тремя словами.

Ответ:
Три законами робототехники.

Зачет:
Три закона Азимова.

Комментарий:
(pic: 20150183.jpg)
   Белые буквы на синем фоне обозначают своего рода сбой мозга робота
при попытке выполнить первый закон робототехники.

Источник:
http://lol24.ee/post/101059/

Автор:
Никита Поздняков (Пущино)

Вопрос 20:
(pic: 20150184.jpg)
   Назовите двумя словами животное, изображение которого мы от вас
скрыли.

Ответ:
Большая панда.

Источник:
http://pikabu.ru/story/tatu_1752435

Автор:
Мария Томашевская (Пущино)

Вопрос 21:
В фильме "Внутри Льюина Дэвиса" с главным героем путешествует кот. После
множества приключений кот возвращается к хозяевам. Напишите состоящую из
одного слова кличку кота.

Ответ:
Одиссей.

Зачет:
Улисс.

Источник:
Х/ф "Внутри Льюина Дэвиса".

Автор:
Мария Томашевская (Пущино)

Вопрос 22:
(pic: 20150185.jpg)
   Восстановите скрытое от вас указание из пяти слов.

Ответ:
После свадьбы читать снизу вверх.

Источник:
http://demotivators.to/p/345533/posle-svadbyi-chitat-snizu-vverh.htm

Автор:
Алексей Дубровский (Пущино)

Вопрос 23:
Любимым увлечением его детства стали воздушные змеи, которых он сам
мастерил. Запустив в небо очередное творение, мальчик по нитке отправлял
"почту" - спичечный коробок с живым тараканом. Когда ему исполнилось 22
года, он построил первую в мире центробежную машину и записал в
дневнике: "Вес рыжего таракана был увеличен в 300 раз, а вес цыпленка -
в 10, без малейшего для них вреда". Назовите этого человека.

Ответ:
[Константин Эдуардович] Циолковский.

Источник:
"Maxim", май 2007 г. - С. 170.

Автор:
Олег Холодов (Серпухов)

Вопрос 24:
"ИКСЫ и ИГРЕКИ" - круглосуточный канал телевизионных игр и викторин.
Осенью 2014 года от многих из вас также требовали ИКСЫ, а зимой 2015
года от вас ждут уже ИГРЕКОВ. Назовите ИКС и ИГРЕК в правильном порядке.

Ответ:
Вопрос, ответ.

Источник:
http://tv.yandex.ru/5/channels/333

Автор:
Олег Холодов (Серпухов)

Тур:
22 тур. "Несомненный скрамасакс" (Москва)

Редактор:
Андрей Алдашев - автор всех вопросов тура; Серафим Шибанов - редактор

Вопрос 1:
Музыканты московской алко-трэш-метал-группы называют себя
"собутыльниками Дьявола". Если не считать пробел, название группы
состоит из 11 символов. Напишите это название.

Ответ:
"Портвейн 666".

Комментарий:
Более известен, конечно, портвейн "777".

Источник:
   1. http://www.portwine666.com/band/
   2. http://vk.com/portwine666

Автор:
Андрей Алдашев (Москва)

Вопрос 2:
В заголовке о продолжении рекламной акции "Зимний марафон" есть слова
"[ПРОПУСК 1]". В Москве 16 декабря 2013 года после ремонта [ПРОПУСК 2].
Заполните любой из пропусков.

Ответ:
Открылось второе дыхание.

Зачет:
Открылось "Второе дыхание".

Комментарий:
Культовая московская рюмочная была закрыта, но, как оказалось,
ненадолго.

Источник:
   1. http://www.tourprom.ru/profi/content/informer/15357/
   2. http://msk-zabegalovka.livejournal.com/492224.html

Автор:
Андрей Алдашев (Москва)

Вопрос 3:
(pic: 20150186.jpg)
   Внимание, АССА - это замена.
   Телебашню на розданной вам фотографии, как и множество ей подобных,
можно сравнить с АССОЙ. Назовите город, где находится эта телебашня.

Ответ:
Вена.

Комментарий:
АССА - игла. Название одного фильма с Виктором Цоем мы заменили другим.
Такая вот игла в Вене.

Источник:
   1. http://gjabu.livejournal.com/34048.html
   2. http://de.wikipedia.org/wiki/Donauturm

Автор:
Андрей Алдашев (Москва)

Вопрос 4:
Внимание, в вопросе есть замена.
   В Базе Вопросов ЧГК буквосочетание "ХЛЕБ" встречается более 900 раз.
Назовите марку ХЛЕБА, которая упоминается при этом чаще всего.

Ответ:
Гиннесс.

Комментарий:
Согласно анекдоту, если в слове "ХЛЕБ" сделать четыре ошибки, получится
"ПИВО". Чаще всего - это рекорд, так что имеет место очередной рекорд
Гиннесса.

Источник:
http://db.chgk.info/search/questions/пиво

Автор:
Андрей Алдашев (Москва)

Вопрос 5:
Почти все уже забыли, что в названии предприятия с турецким капиталом
когда-то фигурировал князь Рюрик. Какой город, помимо Москвы,
упоминается в нем сейчас?

Ответ:
Эфес.

Комментарий:
В названии ЗАО "Пивоварня Москва-Эфес" недолгое время после ее появления
вместо Москвы был князь Рюрик. Оборот "все забыли" намекает на
Герострата, который сжег храм Артемиды Эфесской.

Источник:
Этикетка "Старого мельника" из коллекции автора вопроса.

Автор:
Андрей Алдашев (Москва)

Вопрос 6:
На этикетке одного сорта эля изображены 15 одинаково одетых людей, и
специально подчеркнуто, что этот эль - светлый. Для его приготовления
использовался хмель, выведенный в стране из Южного полушария. В какой
стране?

Ответ:
Новая Зеландия.

Комментарий:
Пиво носит название "All blacks" в честь знаменитой новозеландской
регбийной сборной. В одной из версий этой игры на площадке находится 15
игроков. Черной является их форма, и, чтобы не сбить с толку покупателя
данного светлого эля, производители указали, что это не "черное" пиво.

Источник:
   1. http://www.1tonna.ru/beer/
   2. http://www.nubo.ru/pavel_egorov/moscow/m1t1801.jpg

Автор:
Андрей Алдашев (Москва)

Вопрос 7:
Домашний пивовар Михаил Венёвцев, вероятно, недолго мучился, придумывая
название для своего крепкого светлого сорта. Кто изображен на этикетке
указанного пива?

Ответ:
Фрекен Бок.

Зачет:
Кошка Матильда.

Комментарий:
Сваренное крепкое пиво относится к типу "бок". Название "Freken Bock"
для подобного пива придумать, как нам кажется, несложно. Слова
"домашний" и "мучился" - подсказка.

Источник:
http://www.nubo.ru/pavel_egorov/moscow/mzv0201.jpg

Автор:
Андрей Алдашев (Москва)

Вопрос 8:
Назовите марку российского пива, на этикетке которого была телеведущая
Маша Малиновская.

Ответ:
"Три медведя".

Комментарий:
Три медведя и Машенька - прямо как в сказке.

Источник:
http://www.nubo.ru/pavel_egorov/russian/rpi0519.jpg

Автор:
Андрей Алдашев (Москва)

Вопрос 9:
Дуплет.
   1. Согласно шутке автора вопроса, сидя в жюри телешоу "Голос", Дима
Билан услышал свою песню и прокомментировал ее исполнение цитатой. Чьей?
   2. Согласно другой версии шутки автора вопроса, один из членов жюри
телешоу "Голос" посчитал исполнение участником своей песни настолько
дурным, что раскритиковал того цитатой. Какую пьесу она завершает?

Ответ:
   1. [Константина] Станиславского.
   2. "На дне".

Комментарий:
Билан сказал "Не верю!", а неизвестный член жюри - "Эх... испортил
песню... дур-рак!".

Источник:
   1. ЛОАВ.
   2. http://ru.wikipedia.org/wiki/Не_верю
   3. Максим Горький. На дне.
http://az.lib.ru/g/gorxkij_m/text_0180.shtml

Автор:
Андрей Алдашев (Москва)

Вопрос 10:
Ожидая зимой общественного транспорта на остановках, например,
Савёловский вокзал или Хамовнический вал, автор вопроса характеризовал
состояние окружающей среды известной цитатой. Воспроизведите ее двумя
словами.

Ответ:
Ветер северный.

Комментарий:
Названия остановок звучат похоже на "Владимирский централ", а зимой
весьма прохладно. "Окружающей" - небольшая подсказка.

Источник:
   1. ЛОАВ.
   2. Михаил Круг - Владимирский централ.

Автор:
Андрей Алдашев (Москва)

Вопрос 11:
Певец Валентин Стрыкало стал известен благодаря Интернету. Закончите
одним словом строку из его песни: "Я ухожу к другому, его зовут...".

Ответ:
"... Рустем".

Комментарий:
В этой строчке можно усмотреть намек на блогера Рустема Адагамова,
известного под ником drugoi.

Источник:
   1. Валентин Стрыкало - Рустем.
   2. http://drugoi.livejournal.com/profile

Автор:
Андрей Алдашев (Москва)

Вопрос 12:
[Каждый из бланков раздатки должен быть порван пополам на месте пробела
между словами "Saxifraga" и "adscendens".]
   <раздатка>
   Saxifraga adscendens
   </раздатка>
   Лирический герой группы "Иван-царевич", собираясь в путешествие,
сулящее богатство, намерен запастись АЛЬФОЙ. Назовите АЛЬФУ.

Ответ:
Разрыв-трава.

Комментарий:
Разорванная раздатка должна служить намеком. Растение Saxifraga
adscendens, по одной из версий, является прототипом разрыв-травы из
сказок.

Источник:
   1. Иван-царевич - На острове Буян.
   2. http://ru.wikipedia.org/wiki/Камнеломка

Автор:
Андрей Алдашев (Москва)

Вопрос 13:
Цитата из журнала "Вокруг света": "Вначале НОЖКА была острой палкой,
потом шилом, потом в ее истории случилась ПОДНОЖКА". Что мы заменили
словами "НОЖКА" и "ПОДНОЖКА"?

Ответ:
Вилка, развилка.

Источник:
"Вокруг света", 2013, N 6. - С. 28.

Автор:
Андрей Алдашев (Москва)

Вопрос 14:
Известный в Советском Союзе "ИКС" скалил зубы, в том числе, и на
Гитлера. По некоторым источникам, ИКС Гитлера в 2012 году жил в Москве.
Назовите ИКСА.

Ответ:
Крокодил.

Комментарий:
Крокодилы живут по сотне лет, так что "личный крокодил Гитлера" по
кличке Сатурн, привезенный в Московский зоопарк, вероятно, жив до сих
пор. Сатирический журнал "Крокодил" неоднократно изображал Гитлера на
карикатурах.

Источник:
   1. http://www.journal-club.ru/?q=image/tid/88
   2. http://www.aif.ru/society/article/51397

Автор:
Андрей Алдашев (Москва)

Вопрос 15:
Однажды автор вопроса обратил внимание на дату упаковки и СДЕЛАЛ ЭТО.
Назовите супруга героини, которая СДЕЛАЛА ЭТО на дынных грядках.

Ответ:
Наг.

Комментарий:
СДЕЛАТЬ ЭТО - отложить яйца. Автор вопроса не постеснялся сказать об
этом. Имя кобры из рассказа Киплинга в разных переводах пишется
по-разному, поэтому мы решили спросить ее мужа.

Источник:
   1. ЛОАВ.
   2. Р. Киплинг. Рикки-Тикки-Тави.
http://www.flibusta.net/b/190177/read

Автор:
Андрей Алдашев (Москва)

Вопрос 16:
"Восемь гусениц" - это пример ЗАМЕНЫ. Управляемые силой мысли ЗАМЕНЫ в
XXI веке перестают быть фантастикой. Ответьте одним словом, что мы в
обоих случаях обозначили как ЗАМЕНЫ.

Ответ:
Протезы.

Комментарий:
Протезы используются как замены конечностей. А еще мы проиллюстрировали
лингвистическое явление протезы, когда перед одним звуком появляется
другой для удобства произношения.

Источник:
   1. http://ru.wikipedia.org/wiki/Протеза
   2. http://ru.wikipedia.org/wiki/Протезирование

Автор:
Андрей Алдашев (Москва)

Вопрос 17:
В СССР было [ПРОПУСК 1] для поставок их продукции в необходимых для
[ПРОПУСК 2] объемах. ПРОПУСК 1 и ПРОПУСК 2 малоразличимы на слух.
Заполните оба пропуска.

Ответ:
"... мало козоводов...", "... молокозаводов...".

Источник:
http://www.fermer.ru/book/export/html/15170

Автор:
Андрей Алдашев (Москва)

Вопрос 18:
Изначально словом "кобан" обозначались будки, в которых располагался
ЁКАРНЫЙ БАБАЙ. Напишите два слова, которые мы заменили словами "ЁКАРНЫЙ
БАБАЙ".

Ответ:
Японский городовой.

Комментарий:
Одно экспрессивное восклицание мы заменили другим.

Источник:
http://ru.wikipedia.org/wiki/Кобан_(полицейский_участок)

Автор:
Андрей Алдашев (Москва)

Вопрос 19:
В песне украинской группы "Лихолесье" медведи по утрам делают то же, что
и один из героев Антуана де Сент-Экзюпери. Намеком на попытки
российского руководства прекратить подачу газа это, однако, не является.
Ответьте двумя словами, что они делают.

Ответ:
Гасят фонари.

Зачет:
Синонимичные ответы с указанием фонарей.

Комментарий:
Упомянутый герой - не сам Маленький принц, а Фонарщик.

Источник:
   1. http://www.liholesie.ru/publ/statia_marchen
   2. А. Сент-Экзюпери. Маленький Принц.
http://www.flibusta.net/b/79276/read

Автор:
Андрей Алдашев (Москва)

Вопрос 20:
Дуплет.
   1. На гербе небольшого российского города изображены представители
бобовых, а название, по одной версии, означает "укрытие на
возвышенности". Назовите этот город.
   2. Назовите российский город, в котором действует пивоварня Sheep
Scull Brewery [шип скалл брЮэри].

Ответ:
   1. Гороховец.
   2. Череповец.

Комментарий:
В слове Гороховец некоторые исследователи выделяют корень "-хов-",
означающий "прятать" (сравните "ховать").

Источник:
   1. http://www.biografia.ru/about/moskva02.html
   2. https://vk.com/sheep_skull_brewery_club

Автор:
Андрей Алдашев (Москва)

Вопрос 21:
Улица Космонавтов ТАМ, вероятно, названа в честь МирОслава
ГермашЕвского. Московская улица Космонавтов находится не ТАМ. Ответьте
двумя словами, где ТАМ.

Ответ:
В Люблине.

Комментарий:
Мирослав Гермашевский - пока единственный польский космонавт. А топонимы
типа Люблино склоняются.

Источник:
   1. Карты Google Москвы и Люблина.
   2. http://ru.wikipedia.org/wiki/Гермашевский,_Мирослав

Автор:
Андрей Алдашев (Москва)

Вопрос 22:
Знакомый автора вопроса назвал День работников СИЗО и тюрем России
иноязычным словом, в котором выделил три последние буквы. Напишите
получившееся название.

Ответ:
ХэллоУИН.

Зачет:
ХеллоУИН.

Комментарий:
День работников СИЗО и тюрем приходится на 31 октября, как и Хэллоуин.
УИН - Управление исполнения наказаний, к которому относятся СИЗО и
тюрьмы в регионе России.

Источник:
Шутка знакомого автора вопроса.

Автор:
Андрей Алдашев (Москва)

Вопрос 23:
Дуплет.
   1. Гаубица - это и оружие супергероя, и название украинской компании,
предоставляющей тариф "Киноман". Какие три буквы мы заменили в
предыдущем предложении?
   2. Украинский психолог предостерегает, что "Град" или "Смерч" могут
привести к появлению магического стража. Какие три буквы мы заменили в
предыдущем предложении?

Ответ:
   1. П, т, н.
   2. П, н, х.

Зачет:
В любом порядке, в любом регистре.

Комментарий:
   1. Паутина, а не гаубица. Супергерой - Человек-паук, да и для
интернет-провайдера подходящее название.
   2. Не "магический страж", а "панический страх". Панический страх
нередко охватывает новобранцев при длительном обстреле.

Источник:
   1. http://www.pautina.ch.ua/tarifs.html
   2. http://www.ukrinform.ua/rus/news/v_boy___bez_paniki_psihologi_uchat_preodolevat_boevoy_stress_1686443

Автор:
Андрей Алдашев (Москва)

Вопрос 24:
Вероятно, что самоубийство стало для НЕГО поводом расслабиться. Назовите
ЕГО двумя словами, начинающимися на одни и те же пять букв.

Ответ:
Сфинктер Сфинкса.

Комментарий:
Теперь все могут расслабиться.

Источник:
Общие соображения.

Автор:
Андрей Алдашев (Москва)

Тур:
23 тур. "Яблоко в штанах" (Воронеж - Омск - Смоленск)

Инфо:
Команда благодарит за помощь в подготовке пакета Константина Науменко
(Киев), Юрия Мотькина (Самара), Юлию Лободу (Томск), Сергея Терентьева
(Санкт-Петербург), Максима Мерзлякова и команду "Сфинкс" (Воронеж).

Вопрос 1:
Заголовок заметки о рейтинге самых употребляемых слов русского
разговорного языка короче известной пословицы на одно слово. Напишите
эту пословицу.

Ответ:
Первый блин комом.

Комментарий:
Слово "блин" является самым употребительным в русской разговорной речи.
Заголовок "Первый - блин".

Источник:
"Русский репортер", 2014, N 23.

Автор:
Александр Нечаев (Воронеж)

Вопрос 2:
В этом вопросе мы использовали четыре источника.
   В музее Кьярамонти ОНА держит в руке рог изобилия. Другая ОНА
установлена в американском университете Пердью. Назовите ЕЕ двумя
словами.

Ответ:
Статуя Нила.

Комментарий:
Разливы Нила были рогом изобилия для Египта. А в университете Пердью
находится статуя Нила Армстронга. Фонтан четырех рек символизирует
четыре источника, одним из которых является Нил.

Источник:
   1. http://ru.wikipedia.org/wiki/Музей_Кьярамонти
   2. http://www.russianla.com/common/humor/story.php/474033
   3. http://ru.wikipedia.org/wiki/Армстронг,_Нил
   4. http://ru.wikipedia.org/wiki/Фонтан_Четырёх_рек

Автор:
Александр Сидоренков (Смоленск)

Вопрос 3:
Лозунг одной социальной рекламы гласит: "Не торопись за решетку, ДЕЛАЙ
ЭТО", а сама решетка отбрасывает тень в виде ИКСА. Мы не спрашиваем, что
такое "ДЕЛАТЬ ЭТО". Назовите ИКС двумя словами.

Ответ:
Пешеходный переход.

Комментарий:
Свет, прошедший сквозь решетку, проецируется на дорогу как "зебра"
пешеходного перехода. ДЕЛАТЬ ЭТО - пропускать пешехода.

Источник:
http://www.sostav.ru/publication/prizrak-rebenka-predupredil-toroplivykh-voditelej-5605.html

Автор:
Александр Нечаев (Воронеж)

Вопрос 4:
К вам обращается автор вопроса.
   В 2014 году я (а возможно, и кто-то из вас) отыграл несколько
вопросов с ответом "АЛЬФА". Ограниченность тем в вопросах сегодняшнего
ЧГК ужасно расстраивает. Вас ведь тоже? Из-за этого построить какие-то
новые ходы в этом вопросе не получилось. Простите.
   Назовите АЛЬФУ.

Ответ:
Четвертая стена.

Комментарий:
"Четвертая стена" - изначально театральный термин, обозначающий
воображаемую "стену" между актерами и зрителями, позднее
распространившийся на другие виды искусств. В тексте вопроса автор
всячески (насколько это возможно в отношении автора и игроков) пытается
эту стену сломать, прямо обращаясь к игрокам от лица игрока.
"Ограниченность", "построить" - подсказки. Как и номер вопроса.

Источник:
ЛОАВ.

Автор:
Дмитрий Тарарыков (Москва - Воронеж)

Вопрос 5:
(pic: 20150187.jpg)
   Перед вами английское название популярного советского фильма, в
котором полосками закрыты одни и те же четыре буквы. Назовите эти буквы.

Ответ:
trip.

Комментарий:
Фильм - "Полосатый рейс", а "stripe" в переводе с английского - полоска.

Источник:
http://en.wikipedia.org/wiki/Striped_Trip

Автор:
Андрей Яблонских (Воронеж)

Вопрос 6:
ОН был придуман в швейцарской компании. На постере документального
фильма о нем изображены 26 актеров. Назовите ЕГО точно.

Ответ:
Шрифт "Гельветика".

Зачет:
Гельветика, Гельвеция, Neue Haas Grotesk, Нойе Гельветика.

Комментарий:
На постере изображены 26 букв английского алфавита, а сам шрифт назван в
честь древнего латинского названия Швейцарии. Образец шрифта
"Гельветика" вы могли видеть на раздатке к прошлому вопросу.

Источник:
   1. http://ru.wikipedia.org/wiki/Гельветика
   2. http://en.wikipedia.org/wiki/Helvetica_(film)

Автор:
Андрей Кокуленко (Омск)

Вопрос 7:
На карикатуре, посвященной ЭТОМУ, белый медведь использует скотч.
Назовите ЭТО двумя словами.

Ответ:
Глобальное потепление.

Зачет:
Таяние льдин, таяние льдов, таяние ледников.

Комментарий:
Скотчем медведь пытается "склеить" таящие льдины.

Источник:
http://9gag.com/gag/aeNB4m5

Автор:
Аркадий Илларионов (Воронеж)

Вопрос 8:
Женщинам в Объединенных Арабских Эмиратах запретят делать макияж, а
мужчинам - надевать куфию. Есть, пить и писАть тоже скоро будет нельзя.
В тексте вопроса мы пропустили название, получившее известность с 1928
года. Напишите это название.

Ответ:
"За рулем".

Источник:
   1. http://news.bigmir.net/life/808171-Jenshinam-v-OAE-zapretyat-krasitsya-za-rylem-a-myjchinam-odevat-obrych
   2. http://ru.wikipedia.org/wiki/За_рулём

Автор:
Александр Нечаев (Воронеж)

Вопрос 9:
[Ведущему: "Лу Ю" прочитать отчетливо раздельно.]
   В этом вопросе слово "АЛЬФА" заменяет другое слово.
   Согласно "Чайному канону" Лу Ю АЛЬФА должна быть столь же блестящей,
как сугроб, и столь же роскошной, как весенний лотос. АЛЬФА входит в
название французского романа. Какого?

Ответ:
"Пена дней".

Комментарий:
В "Чайном каноне" упоминается поэма, которая говорит о пене как о
пламенеющем блеске. "Пена дней" - наиболее известный из романов
французского писателя Бориса Виана.

Источник:
   1. Лу Ю. Чайный канон.
http://www.e-reading.link/bookreader.php/1015344/Lu_-_Chaynyy_Kanon.html
   2. http://ru.wikipedia.org/wiki/Пена_дней

Автор:
Михаил Эктов (Воронеж)

Вопрос 10:
(pic: 20150188.jpg)
   [Ведущему: голосом слегка выделить "кораллово".]
   Звезды на Аллее славы сделаны из кораллово-розового композитного
материала. Мы не спрашиваем фамилию актера, звезда которого была открыта
18 августа 1997 года. Назовите его имя.

Ответ:
Патрик.

Комментарий:
Патрик Стар (персонаж мультсериала "Губка Боб Квадратные Штаны") -
розовая морская звезда. 18 августа 1997 года была открыта звезда Патрика
Суэйзи.

Источник:
   1. http://ru.wikipedia.org/wiki/Голливудская_%C2%ABАллея_славы%C2%BB
   2. http://www.imdb.com/event/ev0000693/1997
   3. http://ru.wikipedia.org/wiki/Патрик_Стар

Автор:
Аркадий Илларионов (Воронеж)

Вопрос 11:
В романе Стивена Кинга герой проверяет сочинения своих учеников.
Отмечая, что один из учеников очень сильно нажимает на ручку, он
упоминает фамилию. Какую?

Ответ:
Брайль.

Комментарий:
Выдавленные строки сочинения можно было читать тактильным способом, как
если бы оно было написано шрифтом Брайля.

Источник:
Стивен Кинг. 11/22/63.

Автор:
Андрей Яблонских (Воронеж)

Вопрос 12:
[Ведущему: немного выделить слова "Мюнхена" и "ребенок".]
   Брайан Аттбери считает, что название этих персонажей связано с гербом
Мюнхена, на котором изображен ребенок. Назовите их аналог в
произведениях советского писателя.

Ответ:
Жевуны.

Комментарий:
Они - манчкины. Возможно, "манчкин" произошло из немецкого слова,
означающего "карлик", "маленькое существо". Семья Баума имела немецкие
корни. Аттбери предполагает, что писатель мог видеть репродукцию герба в
детстве. Munch по-английски - жевать.

Источник:
   1. http://en.wikipedia.org/wiki/Munchkin#Origin_of_the_term
   2. http://ru.wikipedia.org/wiki/Манчкин

Автор:
Аркадий Илларионов (Воронеж)

Вопрос 13:
В уголовном деле по ограблению закусочной полицейские смогли доказать
вину подозреваемого с помощью АЛЬФЫ. В статье "Когда в СССР появилась
АЛЬФА" упоминаются гирлянды на шее. Назовите АЛЬФУ.

Ответ:
Туалетная бумага.

Комментарий:
На туалетной бумаге остались следы записки, которой преступник
шантажировал администрацию закусочной. В СССР был дефицит туалетной
бумаги. Обычно ей торговали по десять штук в одни руки, и покупатели шли
по городу с гирляндами на шее.

Источник:
   1. http://www.newsru.com/crime/30dec2014/robtoiletpaperus.html
   2. http://silver-fancy.livejournal.com/206741.html

Автор:
Александр Нечаев (Воронеж)

Вопрос 14:
Хирург Пирогов, пребывая в Риге, выкроил одному цирюльнику новый ИКС.
Хотя АЛЬФА вдохновила многих, сам классик хронологически никак не мог
выкроить ИКС из АЛЬФЫ. Мы не просим назвать классика. Назовите ИКС и
АЛЬФУ.

Ответ:
Нос, шинель.

Комментарий:
Пирогов выкроил цирюльнику новый нос, хотя читал ли Пирогов Гоголя -
автору неизвестно. "Шинель" появилась позже "Носа", поэтому Гоголь не
мог сам из нее сделать "Нос". Известно выражение "Все мы вышли из
гоголевской шинели".

Источник:
   1. http://www.booksite.ru/localtxt/sci/ent/ist/sto/45.htm
   2. http://ru.wikipedia.org/wiki/Гоголь,_Николай_Васильевич
   3. http://www.bibliotekar.ru/encSlov/3/182.htm

Автор:
Александр Сидоренков (Смоленск)

Вопрос 15:
Отмечая способности славян к торговле и предпринимательству, Борис
Акунин говорит, что даже ЭТО они делали не авансом, а только в случае
успеха. "ЭТО" - название фильма 1986 года. Назовите ЭТО сложным словом.

Ответ:
Жертвоприношение.

Комментарий:
Жертвы богам славяне приносили только когда уже получили их милость.
"Жертвоприношение" - фильм Андрея Тарковского.

Источник:
   1. Борис Акунин. История государства Российского. Том 1.
   2. http://ru.wikipedia.org/wiki/Жертвоприношение_(фильм,_1986)

Автор:
Александр Нечаев (Воронеж)

Вопрос 16:
Герой Виктора Пелевина, испытывая противоречивые чувства к школьной
программе по литературе, сравнивает ее с ИКСОМ. Британские ученые
недавно заявили о планах получения ИКСА из генномодифицированных
растений. Назовите ИКС двумя словами.

Ответ:
Рыбий жир.

Комментарий:
Пелевин замечает, что школьная программа по литературе это полезно, но
совершенно невкусно.

Источник:
   1. Виктор Пелевин. Любовь к трем цукербринам.
   2. http://medportal.ru/mednovosti/news/2014/04/17/082fishoil/

Автор:
Александр Нечаев (Воронеж)

Вопрос 17:
[Ведущему: "ИГРЕКОМ рассказывать о природе" прочитать как единую
конструкцию. Не <ИКС с ИГРЕКОМ> <рассказывать о природе>, а <ИКС> с его
<ИГРЕКОМ рассказывать о природе>.]
   На американской вилле, где собираются русские, Тимофей Пнин сожалеет,
что не пришел ИКС с его ИГРЕКОМ рассказывать о природе. Назовите ИКСА и
ИГРЕК.

Ответ:
Набоков, дар.

Комментарий:
Набоков увлекался энтомологией и написал произведение "Дар". "Пнин" -
также произведение Набокова. Писатель, как известно, жил и в Америке.

Источник:
   1. В. Набоков. Пнин. http://www.flibusta.net/b/158647/read
   2. http://ru.wikipedia.org/wiki/Дар_(роман)

Автор:
Александр Сидоренков (Смоленск)

Вопрос 18:
В стихотворении Натальи Ромашовой про барана с необычной кличкой ПРОПУСК
есть такие строчки:
   Стоит лишь в разговоре
   Ответить: "Да ну!",
   И вам сразу припомнят
   Знакомство с ПРОПУСК!
   Заполните пропуск.

Ответ:
Кигну.

Комментарий:
Баран Кигну - персонифицированная присказка баранки гну.

Источник:
http://www.artlebedev.ru/everything/izdal/baran-kignu/

Автор:
Аркадий Илларионов (Воронеж)

Вопрос 19:
Уважаемые знатоки, не печальтесь, если не возьмете этот вопрос.
   Путешественник Владимир Севриновский так пишет о своем походе к
курящемуся вулкану: "Пришлось ползти вниз по едва различимой тропинке. Я
то и дело оскальзывался и цеплялся за ветки, щедро ДЕЛАЯ ЭТО". Какое
устойчивое словосочетание было заменено словами "ДЕЛАТЬ ЭТО"?

Ответ:
Посыпать голову пеплом.

Источник:
http://www.vokrugsveta.ru/article/190618/

Автор:
Андрей Яблонских (Воронеж)

Вопрос 20:
В связи с особенностями ремонта дорог в России дорожное полотно иногда
сравнивают с НИМ. ОНО обычно упоминается при описании одного из периодов
отечественной истории. Назовите ЕГО двумя словами.

Ответ:
Лоскутное одеяло.

Комментарий:
Землю в иллюминаторе, дорожное полотно и карты различных государств
периода феодальной раздробленности довольно часто сравнивают с лоскутным
одеялом.

Источник:
http://ru.wikipedia.org/wiki/Лоскутное_одеяло

Автор:
Юлия Мещерякова (Воронеж)

Вопрос 21:
(pic: 20150189.jpg)
   [Ведущему: после слова "аллюзией" сделать смысловую паузу.]
   Дуплет.
   1. Розданный кадр из эпизода мультсериала "Симпсоны" сопровождает
хорошо известная всем вам музыка. В том же эпизоде за открывшейся дверью
туалета мы видим человека, держащего в руках... Что?
   2. Название упомянутого эпизода является аллюзией с поправкой на
характерную особенность персонажей сериала. Напишите это название.

Ответ:
   1. Винтовка.
   2. "A Clockwork Yellow".

Зачет:
   1. М14, автомат, карабин, ружье.
   2. "Заводной желтый", "Заводной лимон".

Комментарий:
Эпизод целиком посвящен творчеству Стэнли Кубрика. На розданной картинке
Гомер одет как персонаж "Заводного апельсина", а поднятая над головой
кость и музыка Рихарда Штрауса из симфонической поэмы "Так говорил
Заратустра" отсылают к фильму "Космическая одиссея 2001 года". Рядовой
Леонард "Куча" Лоуренс, сидящий в туалете с винтовкой, - одна из
наиболее известных сцен в фильме Кубрика "Цельнометаллическая оболочка".
Название эпизода обыгрывает то, что все персонажи "Симпсонов" имеют кожу
желтого цвета, что хорошо видно на раздаточном материале.

Источник:
"The Simpsons", s26e04.

Автор:
Дмитрий Тарарыков (Москва - Воронеж)

Вопрос 22:
Одна из цитат с сайта bash.im предполагает, что полное название
известного фильма 2010 года заканчивается словами "... только,
пожалуйста, не выкладывай всё это в Инстаграм". Назовите этот фильм.

Ответ:
"Ешь, молись, люби".

Комментарий:
Фильм снят по мотивам романа Элизабет Гилберт "Есть, молиться, любить"
(Eat, Pray, Love; 2006).

Источник:
http://bash.im/abyssbest/20150107

Автор:
Аркадий Илларионов (Воронеж)

Вопрос 23:
Пушпака, колесница Рамачандры, была, наверное, самая быстрая, да и
управлялась ЛОЖЬЮ. Что мы заменили ЛОЖЬЮ?

Ответ:
Мысль.

Комментарий:
Мысль изреченная есть ложь. Чтобы не солгать, слово "мысль" мы в вопросе
заменили. Согласно другому известному изречению, самое быстрое - это
мысль.

Источник:
   1. http://ru.wikipedia.org/wiki/Пушпака
   2. http://ru.wikiquote.org/wiki/Мысль
   3. http://www.gumer.info/bogoslov_Buks/Philos/Spirk/03.php

Автор:
Александр Сидоренков (Смоленск)

Вопрос 24:
Внимание, в вопросе есть замены.
   На рекламном плакате компании "Jeep" со слоганом "У каждого свой ритм
сердца!" изображен ИКС определенной формы. Назовите ИКС двумя словами,
одно из которых - английского происхождения.

Ответ:
След протектора.

Зачет:
Узор протектора, отпечаток протектора и другие синонимичные варианты со
словом "протектор".

Комментарий:
На плакате изображен след протектора шины в виде кардиограммы.

Источник:
   1. http://www.autogrodno.by/stati-partnerov/6826-luchshaya-avtomobilnaya-reklama.html
   2. http://ru.wikipedia.org/wiki/Протектор_(элемент_шины)

Автор:
Александр Нечаев (Воронеж)

Тур:
24 тур. "Сборная Царицына" (Волгоград)

Вопрос 1:
В немецкой мифологии есть злой дух по имени ИКС, который подсовывает
рудокопам металл, внешне похожий на медь. Как рассказал Виктор Бочаров,
один житель Воронежской области предложил вместо традиционного
обозначения ИКСА использовать череп с костями. Назовите ИКС.

Ответ:
Никель.

Комментарий:
В Воронежской области обнаружены крупнейшие месторождения никеля. Среди
жителей Прихоперья распространилось много слухов об опасности этого
металла.

Источник:
http://www.vlg.aif.ru/society/ecology/1071777

Автор:
Роман Мерзляков (Волгоград)

Вопрос 2:
Литературовед Вадим Борисов утверждает, что в мандельштамовской строке
"Успенье нежное - Флоренция в Москве" дважды упоминается ОНА. Назовите
ЕЕ.

Ответ:
Марина Цветаева.

Комментарий:
"Цветущее" название итальянского города содержит явную отсылку к фамилии
Марины Цветаевой. Скрытая аллюзия содержится в упоминании Успенского
собора, который построил известный итальянский зодчий Аристотель
Фиораванти. По-итальянски fiore - "цветок".

Источник:
Л. Видгоф. Французские мотивы. Нестрогие рассуждения о стихах и прозе О.
Мандельштама. // "Вопросы литературы", 2013, N 6. - С. 116.

Автор:
Роман Мерзляков (Волгоград)

Вопрос 3:
У представителей ЭТОГО подошва на левом ботинке из тефлона, а на правом
- из пористой резины. Какого деятеля российской истории первой половины
XX века упомянул наблюдавший за ЭТИМ журналист?

Ответ:
Чапаев.

Комментарий:
В керлинге у спортсменов один ботинок служит для хорошего сцепления со
льдом, а на тефлоновой подошве, наоборот, можно скользить при запуске
камня. Керлинг напоминает не только бильярд на льду, но и такую игру,
как "Чапаев".

Источник:
   1. http://gazeta.aif.ru/_/online/ss/245/vkl01_01
   2. Трансляция кёрлингового матча Россия - Дания, телеканал "Россия
2", 10.02.2014 г.

Автор:
Роман Мерзляков (Волгоград)

Вопрос 4:
Комментатор хоккейного матча с участием женской олимпийской сборной
России указывал, что не всегда наши хоккеистки находят верные решения.
При этом, оценивая важную роль в команде игрока под N 8, комментатор
вспомнил Константина Бескова. Назовите фамилию игрока N 8.

Ответ:
Гаврилова.

Комментарий:
Бесков говорил "спартаковцам": "Не знаешь, что делать с мячом, - отдай
его Гаврилову". Так же оценивал комментатор роль Ии Гавриловой.

Источник:
   1. Трансляция матча Россия - Германия, телеканал "Россия 2",
09.02.2014 г.
   2. http://ru.wikipedia.org/wiki/Гаврилова,_Ия_Викторовна
   3. http://ru.wikipedia.org/wiki/Гаврилов,_Юрий_Васильевич

Автор:
Роман Мерзляков (Волгоград)

Вопрос 5:
Говоря о движении прогресса вспять, Умберто Эко предрекает переход на
кабельное телевидение. Использование проводов, считает философ,
отодвинет нас к эпохе Антонио Меуччи - ученого, изобретшего в 1857 году
телефон. Однако до этого, как считает непоследовательный Эко,
человечество откатится к другой стадии. С чьим именем он связывает эту
стадию?

Ответ:
[Гульельмо] Маркони.

Комментарий:
Меуччи изобрел телефон на 19 лет раньше Александра Белла, однако из-за
неправильного оформления патента лишился права официального
первооткрывателя. Однако патриотичный Эко эпоху радиовещания связывает с
Маркони, который изобрел радио позже Александра Попова, но зато
позаботился оформить документы. Впрочем, термин "галактика Маркони" ввел
канадец Маршалл Маклюэн, а Умберто Эко лишь воспользовался им.

Источник:
У. Эко. Полный назад! - М., 2012. - С. 11-12.

Автор:
Роман Мерзляков (Волгоград)

Вопрос 6:
Вергилий из Бигорра пишет, как два ритора четырнадцать суток напролет
спорили о звательном падеже одного слова. Назовите это слово.

Ответ:
Я.

Комментарий:
Бессмысленность спора в том, что сам себя человек ни в каких ситуациях
не зовет.

Источник:
У. Эко. Полный назад! - М., 2012. - С. 562.

Автор:
Роман Мерзляков (Волгоград)

Вопрос 7:
(pic: 20150190.jpg)
   Этот морской рисунок разместил в своей книге Яков Перельман. Назовите
любую страну, у берегов которой мог быть сделан этот рисунок.

Ответ:
Любая из стран: Сан-Томе и Принсипи, Экваториальная Гвинея, Габон,
Конго, Кения, Сомали, Мальдивы, Индонезия, Кирибати, Эквадор, Колумбия,
Бразилия. Незачет: Уганда и Демократическая Республика Конго (у первой
нет выхода к морю, у второй он довольно далеко от экватора).

Комментарий:
Яков Перельман, как известно, занимался популяризацией точных наук. На
правдоподобном с астрономической точки зрения рисунке видно, что у
человека почти нет тени. Такое можно видеть на экваторе.

Источник:
   1. Я.И. Перельман. Занимательная астрономия. - М., 2011. - С. 30-31.
   2. http://www.vokrugsveta.ru/encyclopedia/index.php?title=Экватор

Автор:
Роман Мерзляков (Волгоград)

Вопрос 8:
(pic: 20150191.jpg)
   Надеемся, что этот вопрос доставит вам удовольствие.
   Яков Перельман, описывая лунную орбиту, говорит, что Луна в АЛЬФЕ
занимает такое же положение по отношению к Земле, как и в перигее. Что
мы заменили на АЛЬФУ?

Ответ:
Точка G.

Комментарий:
Перигей приходится на чертеже на точку A. А противоположная ей точка G
соответствует апогею.

Источник:
Я.И. Перельман. Занимательная астрономия. - М., 2011. - С. 90-91.

Автор:
Роман Мерзляков (Волгоград)

Вопрос 9:
   <раздатка>
   Возобновляемые источники (солнце, ветер, геотермальные) - чистые,
фотогеничные, но крайне непредсказуемы в поведении. Быстро наращивают
долю, но начинают с очень низкой базы. Атом - профессионал в летах,
всегда заканчивает оплаченную работу, за карьеру правда у него случалось
разное. Развивается 40 лет, но пока так и остался в аутсайдерах. Более
того, сейчас его доля даже сокращается. Гидроэнергетика - простая,
"грубая", но очень эффективная технология, вышедшая из деревенских
водяных мельниц.
   </раздатка>
   В блогерском сообществе "Мир вокруг нас" было опубликовано
экономическое исследование перспектив альтернативной энергетики.
Аналогию с каким фильмом проводят авторы, если его бюджет был на 600
тысяч долларов больше, чем у предыдущей картины того же режиссера?

Ответ:
"Хороший, плохой, злой".

Комментарий:
Возобновляемые источники - это хороший персонаж. Атом - плохой. А
гидроэнергетика - злой. Предыдущий фильм Серджо Леоне назывался "На
несколько долларов больше".

Источник:
   1. http://mirvn.livejournal.com/8149.html
   2. http://ru.wikipedia.org/wiki/Хороший,_плохой,_злой
   3. http://ru.wikipedia.org/wiki/На_несколько_долларов_больше

Автор:
Роман Мерзляков (Волгоград)

Вопрос 10:
У персонажа Виктора Пелевина сопоставление шансов ПЕРВОГО и ВТОРОГО
вызвало римско-пугачёвские ассоциации. Назовите ПЕРВОГО и ВТОРОГО.

Ответ:
"Спартак" и "Салават Юлаев".

Комментарий:
Речь о московском и уфимском хоккейных клубах, названных в честь
древнеримского гладиатора и башкирского бандита из армии Емельяна
Пугачёва.

Источник:
В. Пелевин. Желтая стрела.

Автор:
Роман Мерзляков (Волгоград)

Вопрос 11:
В финале рассказа Виктора Пелевина, изображающего социалистический быт,
главный герой видит ковш Большой Медведицы. Какое слово мы заменили в
тексте вопроса?

Ответ:
Совок.

Комментарий:
Заменили слово "совок" на "ковш". Советскую действительность часто
уничижительно называли совком.

Источник:
В. Пелевин. Желтая стрела.

Автор:
Роман Мерзляков (Волгоград)

Вопрос 12:
Когда пожилые прихожане пожаловались волгоградскому священнику Дмитрию
Нестерову, что не могут запомнить псалмы, он пошел по пути своего
западноевропейского коллеги и обучил молитвам десять ИХ. Назовите ИХ
словом арабского происхождения.

Ответ:
Попугаи.

Комментарий:
Монах Франциск Ассизский, как известно, читал Библию, в том числе, и
птицам. А Дмитрий Нестеров показал прихожанам попугаев, затвердивших
тексты молитв, как пример усердия. Слово "попугай" восходит к арабскому
"babagh&#257;", от которого попало во французский и испанский, а оттуда
уже в русский.

Источник:
   1. Пресс-релиз областного правительства о смерти иеромонаха Дидима
(Дмитрия Нестерова).
   2. http://put.ucoz.ru/index/0-76
   3. http://ru.wiktionary.org/wiki/попугай

Автор:
Роман Мерзляков (Волгоград)

Вопрос 13:
Согласно шуточной версии Ирины Антоновой, в 1937 году художник Аминадав
Каневский начал рисовать плюшевого медвежонка, но успел очертить только
его контуры. Набросок сбежал с мольберта и нырнул в заросли молодых
одуванчиков. А как оживший рисунок представился, когда пришел
устраиваться на работу?

Ответ:
Мурзилка.

Комментарий:
Сначала рисунок был похож на белого медвежонка, но в одуванчиковом море
он сам окрасился в ярко-желтый цвет. Персонаж пришел устраиваться
корреспондентом в журнал, где работал его автор.

Источник:
И. Антонова. Желтый и пушистый. // "Мурзилка", 2014, N 5. - С. 8-11.

Автор:
Роман Мерзляков (Волгоград)

Вопрос 14:
28 января 1925 года Иван Бунин написал стихотворение, в котором были
такие строки:
   И всё ж придет, придет пора
   И воскресенья и деянья,
   Прозрения и покаянья.
   Чьей памяти посвящены эти стихи?

Ответ:
Петр I.

Комментарий:
Дата написания стихотворения - это 200-я годовщина со дня смерти первого
российского императора. Последняя строка четверостишия содержит рифму к
слову "пора": "Россия! Помни же Петра".

Источник:
http://pesni.voskres.ru/poems/bunin17.htm

Автор:
Роман Мерзляков (Волгоград)

Вопрос 15:
Критик Тимофеев пишет: чтобы современные российские читатели этого
английского романа испытали те же ощущения, что и в свое время западная
публика, переводчику нужно было б некоторые русские слова заменить на
чеченские. Назовите этот роман.

Ответ:
"Заводной апельсин".

Комментарий:
По словам Тимофеева, "русский язык, особенно среди западных читателей
старшего поколения, прочно ассоциировался с угрозой советского
тоталитаризма". В "Заводном апельсине" Энтони Бёрджесс воспроизводит
молодежный жаргон, в котором очень много русских слов. Эти русские слова
в англоязычном тексте и предлагается при переводе заменить на чеченские.

Источник:
В. Тимофеев. Послесловие. // Э. Бёрджесс. Заводной апельсин. - СПб.,
2004. - С. 226.

Автор:
Роман Мерзляков (Волгоград)

Вопрос 16:
Персонаж Милорада Павича говорил, что ПЕРВОЕ идет в начале, ВТОРОЕ - в
конце. ПЕРВОЕ ассоциируется с жизнью, а ВТОРОЕ - с кладбищами и
могильными цветами. Назовите ВТОРОЕ двумя словами на одну и ту же букву.

Ответ:
Четное число.

Комментарий:
В рядах целых чисел нечетные идут раньше четных: 1, 2; затем 3, 4 и т.д.
Могильные букеты состоят из четного количества цветов, тогда как живым
дарят нечетное число бутонов.

Источник:
М. Павич. Русская борзая. - СПб., 2001. - С. 32.

Автор:
Роман Мерзляков (Волгоград)

Вопрос 17:
   <раздатка>
   f,sh
   </раздатка>
   Это набранное в английской раскладке первое слово, произнесенное ИМ.
Напишите ЕГО имя или фамилию.

Ответ:
Полиграф.

Зачет:
Шариков.

Комментарий:
Так в английской раскладке выглядит слово "абыр", т.е. "рыба" наоборот.

Источник:
http://bash.im/quote/419694/

Автор:
Денис Леонтьев (Волгоград)

Вопрос 18:
В одном документальном фильме о русском ученом использованы кадры
поздней режиссерской работы Василия Шукшина. Зрители могут видеть героя
"Калины красной" по прозвищу Горе. О каком ученом снят этот докфильм?

Ответ:
[Сергей] Прокудин-Горский.

Комментарий:
"Калина красная" - единственный фильм Василия Шукшина, снятый им на
цветную пленку. Главного героя картины зовут Егор Прокудин. В фильме
Леонида Парфёнова о пионере цветной фотографии Прокудине-Горском
происходит своеобразная встреча изобретателя и его кинематографического
однофамильца.

Источник:
   1. Документальный фильм "Цвет нации"
(http://www.youtube.com/watch?v=2yIWsXe8s2Y), 1:15:45.
   2. http://ru.wikipedia.org/wiki/Калина_красная_(фильм)

Автор:
Роман Мерзляков (Волгоград)

Вопрос 19:
На чемпионате мира по футболу корреспондент спортивного СМИ заметил на
матче сборной Бразилии Леонардо Ди Каприо. Журналист съязвил, что
никогда еще актер не был так близок к Халку. Какое слово мы заменили в
тексте вопроса?

Ответ:
Оскару.

Комментарий:
Халк и Оскар - игроки сборной Бразилии. Журналист имел в виду, что Ди
Каприо, несмотря на довольно успешную карьеру, до сих пор ни разу не
получил премию "Оскар".

Источник:
http://www.sports.ru/football/1021291329.html

Автор:
Роман Мерзляков (Волгоград)

Вопрос 20:
(pic: 20150192.jpg)
   Деревенские герои рассказа Юрия Коваля, гадая, как проводить
реанимационные действия в полевых условиях, решают, что нужно СДЕЛАТЬ
ЭТО. Назовите слово, которое мы заменили на "СДЕЛАТЬ ЭТО".

Ответ:
Вдуть.

Комментарий:
"Вдувай! Вдувай! - сразу обрадовался Генка. - В нас воздух живой. Он
войдет в него и оживит", - так персонажи рассуждают об искусственном
дыхании. Фотографии сексуальных девушек в Интернете часто сопровождаются
пользовательскими комментариями с однообразным содержанием: "Я бы вдул".

Источник:
   1. http://www.e-reading.link/chapter.php/105418/11/Koval'_-_Samaya_legkaya_lodka_v_mire_(sbornik).html
   2. http://www.gotowall.com/gallery/people/models_female/13376.html

Автор:
Роман Мерзляков (Волгоград)

Вопрос 21:
В июле 2014 года в российском Интернете появилась такая шутка. Некий
человек оказывается в глухом лесу и с облегчением говорит: "Наконец я
совсем один". Мы не спрашиваем, что он услышал в ответ. Назовите этого
человека.

Ответ:
Луис Фелипе Сколари.

Комментарий:
На домашнем чемпионате мира по футболу бразильцы проиграли в полуфинале
немцам. И эхо напомнило главному тренеру Бразилии счет матча: "семь -
один, семь - один, семь - один".

Источник:
http://wc2014.rsport.ru/wc2014/20140709/757078009.html

Автор:
Роман Мерзляков (Волгоград)

Вопрос 22:
Говоря о том, что жизнь Сократа еще до суда над ним оказалась под
угрозой, Михаил Гаспаров пишет, что ОНИ сгущаются. Назовите ИХ,
используя восемь символов.

Ответ:
"Облака".

Комментарий:
Первым предупреждением своевольному философу была комедия Аристофана
"Облака", в которой показывалось вредное влияние Сократа на молодежь. В
финале пьесы земледелец Стрепсиад поджигает школу мудреца.

Источник:
   1. М.Л. Гаспаров. Занимательная Греция: Рассказы о древнегреческой
культуре; Капитолийская волчица: Рим до цезарей. - М.: Эксмо, 2012. - С.
306-308.
   2. http://ru.wikipedia.org/wiki/Облака_(комедия)

Автор:
Роман Мерзляков (Волгоград)

Вопрос 23:
Нападающему команды "Ротор-Волгоград" Дмитрию Шишину после победы в
чемпионате России по пляжному футболу вручили премию "ИКС". На
германском телевидении ИКС присутствует в немецких аналогах передачи
"Спокойной ночи, малыши!". Назовите ИКСА двумя словами.

Ответ:
Песочный человек.

Комментарий:
В пляжный футбол играют на песке. А фольклорный персонаж Песочный
человек помогает детям заснуть.

Источник:
   1. http://www.volganet.ru/news/3060/
   2. http://ru.wikipedia.org/wiki/Песочный_человек

Автор:
Роман Мерзляков (Волгоград)

Вопрос 24:
Дуплет.
   У Хулио Кортасара есть серия ироничных инструкций, как совершать
какие-либо банальные действия, например, как правильно петь или как
правильно подниматься по лестнице.
   1. В инструкции, как правильно ДЕЛАТЬ ЭТО, рекомендуется подумать "о
селезне, подвергшемся нападению полчища муравьев". Также следует при
этом избегать сходства с улыбкой. Что учит делать эта инструкция?
   2. В инструкции, как правильно ДЕЛАТЬ ЭТО, рекомендуется сначала
приподнять головку. Далее следует фраза: "И вот начинаются новые сроки".
Что учит делать эта инструкция?

Ответ:
   1. Плакать.
   2. Заводить часы.

Зачет:
   2. Переводить часы, переводить время, переводить стрелки.

Комментарий:
   2. Кортасар советует поднять головку завода.

Источник:
Х. Кортасар. Собрание сочинений в четырех томах. - Т. 2. - СПб., 1992. -
С. 531, 532, 534.

Автор:
Роман Мерзляков (Волгоград)

Тур:
25 тур. "Сборная с. Тупино" (Ступино - Коломна)

Вопрос 1:
   <раздатка>
   Если вы читаете это, тогда это предупреждение для вас. Каждое слово,
прочитанное вами на этой бесполезной картинке, крадет у вас секунду
жизни. Вам больше нечем заняться? Ваша жизнь настолько пуста, что вы
действительно не можете придумать лучшего времяпрепровождения? Или вы
настолько впечатлены властью, что готовы уважать каждого и доверять
всем, кто этого требует? Вы читаете всё, что вам предлагают прочесть?
Думаете обо всём, о чем предлагают подумать? Покупаете то, что вам
навязывают? Выйдите из вашей квартиры. Встретьтесь с кем-нибудь
противоположного пола. Прекратите чрезмерно покупать и мастурбировать.
Уйдите с работы. Затейте драку. Докажите, что вы живы. Если вы не
докажете свое право быть человеком, вы превратитесь в статистику. Я вас
предупредил...
   </раздатка>
   Какое имя стоит в подписи к этому предупреждению?

Ответ:
Тайлер.

Зачет:
Дёрден.

Комментарий:
Еще одно предупреждение перед началом "Бойцовского клуба".

Источник:
http://www.adme.ru/tvorchestvo-kino/25-tajn-bojcovskogo-kluba-612655/

Автор:
Юлия Кощеева (Коломна), Виктор Плотников (Великий Новгород)

Вопрос 2:
Внимание, в вопросе есть одна замена.
   На логотипе интернет-магазина, продающего автомобильные шины и диски,
изображены две покрышки, лежащие друг на друге. Этот интернет-магазин
называется "Поршенон". В предыдущем предложении мы заменили одно слово.
Какое?

Ответ:
"Колесей".

Зачет:
"Колисей".

Комментарий:
Поршенон - Парфенон, Колесей - Колизей.

Источник:
http://www.kolesey.ru/

Автор:
Андрей Волков, Игорь Коршунов (Коломна)

Вопрос 3:
Внимание, в вопросе есть одна замена.
   Герой сериала "Интерны" венеролог Купитман говорит: "Ко мне пришел
депутат. Пойду, полюбуюсь, так сказать, на ИКС". Назовите ИКС двумя
словами, и не ошибитесь веткой.

Ответ:
Законодательный орган.

Источник:
Телесериал "Интерны", s4e33, 15'17".

Автор:
Юлия Кощеева (Коломна), Виктор Плотников (Великий Новгород)

Вопрос 4:
Во время Зимних Олимпийских игр 2014 года в Сочи многие интернет-сайты
вели текстовые трансляции происходящих событий. Онлайн-трансляция
мужской эстафеты в биатлоне на сайте Lenta.ru носила подзаголовок:
"Лыжи, палки, ОНИ". Напишите два слова, которые мы заменили словом
"ОНИ".

Ответ:
Четыре ствола.

Комментарий:
Мужская эстафета - 30 км (4 участника по 7,5 км). Ну и вопрос тоже
четвертый.

Источник:
http://sochi.lenta.ru/onlines/2014/02/22/thlon/

Автор:
Юлия Кощеева (Коломна), Виктор Плотников (Великий Новгород)

Вопрос 5:
[Ведущему: выделить голосом слово "ФОРМУЛУ".]
   Напоминаем ФОРМУЛУ стишка-порошка: 9+8+9+2, вторая и четвертая строки
рифмуются. Прослушайте первые две строки порошка:
   вот обнажившись кофеманка
   ныряет в море кофиЯ
   Напишите восьмибуквенное существительное, присутствующее в третьей
строке этого грустного порошка.

Ответ:
Капучина.

Зачет:
Капучино.

Комментарий:
   вот обнажившись кофеманка
   ныряет в море кофиЯ
   и капучина поглощает
   ея

Источник:
https://vk.com/perawki?w=wall-28122932_48096

Автор:
Олег Евстафьев (Ступино)

Вопрос 6:
(pic: 20150193.jpg)
   Воспроизведите подпись под этим изображением, совпадающую со строкой
из детской песни.

Ответ:
"Голубой вагон бежит, качается".

Источник:
(pic: 20150194.jpg)

Автор:
Борис Скоморохов (Ступино)

Вопрос 7:
В популярной сетевой игре "Годвилль" герой автора вопроса записал у себя
в дневнике: "Прошелся по овсяному полю, потом по пшеничному, по
ячменному. В общем, посетил кучу ИХ, а денег не потратил". Назовите ИХ
двумя словами.

Ответ:
Злачные места.

Источник:
http://www.godville.net

Автор:
Юлия Кощеева (Коломна), Виктор Плотников (Великий Новгород)

Вопрос 8:
Дуплет.
   1. В одном из эпизодов телевизионного шоу "Утка-детектив" из
мультсериала "Гравити Фолз" констебль в ответ на реплику УТКИ говорит,
что она фактически ПРОПУСК дело. Какое слово мы заменили на слово
"ПРОПУСК"?
   2. Когда в финале мультфильма "Шрек-2" фея превратила короля
Гарольда, один из второстепенных персонажей сказал: "Он ПРОПУСК". Какое
слово мы заменили на "ПРОПУСК"?

Ответ:
   1. Крякнула.
   2. Квакнулся.

Комментарий:
Вот такой вот птичий вопрос.

Источник:
   1. Телесериал "Гравити Фолз".
   2. М/ф "Шрек-2", 1h15'.

Автор:
Игорь Коршунов (Коломна), Олег Евстафьев (Ступино)

Вопрос 9:
Прослушайте четверостишие:
   Постигая дамские причуды,
   сделайте заметку, господа:
   иногда слова "иди отсюда"
   означают...
   Закончите последнюю строку двумя словами.

Ответ:
"Идиот, сюда".

Комментарий:
Важны не только запятые, но и пробелы.

Источник:
http://tamsyam.livejournal.com/406028.html

Автор:
Олег Евстафьев (Ступино)

Вопрос 10:
(pic: 20150195.jpg)
   Подпись к этому изображению всего на одну букву длиннее названия
популярной детской сказки. Воспроизведите подпись к рисунку.

Ответ:
"Храпунцель".

Источник:
https://vk.com/namalevich?w=wall-26217154_100517

Автор:
Юлия Кощеева (Коломна), Виктор Плотников (Великий Новгород)

Вопрос 11:
Внимание, в вопросе есть одна замена.
   Согласно твиттеру сайта Kinopoisk.ru [кинопоиск точка ру], одно из
правил создания постеров к фильмам с очень крутым героем таково: на
постере герой ПОКАЗЫВАЕТ СВОЕ НАСТОЯЩЕЕ ЛИЦО. До XIX века дирижеры не
ПОКАЗЫВАЛИ СВОЕ НАСТОЯЩЕЕ ЛИЦО. Напишите четыре слова, которые мы
заменили словами "ПОКАЗЫВАТЬ СВОЕ НАСТОЯЩЕЕ ЛИЦО".

Ответ:
Стоять спиной к зрителю.

Комментарий:
Герои на многих постерах стоят спиной к зрителю. До XIX века дирижеры
стояли лицом к зрителям.

Источник:
https://twitter.com/kinopoiskru/status/388609450711724032/

Автор:
Ксения Калинина (Великий Новгород - Санкт-Петербург)

Вопрос 12:
Одна из немногих FM-радиостанций, работающих в АнАдыре, носит название
"Радио Фигня". Какое слово мы заменили словом "Фигня"?

Ответ:
Пурга.

Комментарий:
Как еще назвать радиостанцию в снежном Анадыре?

Источник:
http://www.laradiofm.ru/station-950/

Автор:
Юлия Кощеева (Коломна), Виктор Плотников (Великий Новгород)

Вопрос 13:
Прослушайте цитату из песни:
   И сердце огнем пылает,
   И разум злой туман затмил,
   И снова дым извергает ...
   В последней строке четверостишия упоминается вымышленный топоним.
Назовите этот топоним.

Ответ:
Сайлент Хилл.

Источник:
КняZz - Пепел города грез.
http://www.knyazz.ru/releases/tajna-krivyx-zerkal

Автор:
Андрей Волков (Коломна)

Вопрос 14:
(pic: 20150196.jpg)
   Перед вами двое рабочих - один с шуруповертом, другой с молотком. В
какую игру они играют?

Ответ:
"Крестики-нолики".

Источник:
http://tango2010weibo.tumblr.com/post/86223935582

Автор:
Юлия Кощеева (Коломна), Виктор Плотников (Великий Новгород)

Вопрос 15:
В этом библейском сюжете старообрядцы усматривали современный им смысл:
Петр I упразднил патриаршество. Написанная в XVIII веке в Невьянске
икона на этот сюжет изображает Петра в виде воина с мечом. Назовите этот
сюжет.

Ответ:
Усекновение главы Иоанна Предтечи.

Зачет:
По смыслу.

Комментарий:
Петр I обезглавил церковь.

Источник:
http://roizman.livejournal.com/1690491.html

Автор:
Юлия Кощеева (Коломна), Виктор Плотников (Великий Новгород)

Вопрос 16:
В одном сериале полицейский собирается на пенсию. Он приобретает себе
ресторан. Зайдя в это заведение, другой герой сериала видит большое
количество посетителей-копов и говорит: "Отличная встроенная ОНА".
Назовите ЕЕ двумя словами.

Ответ:
Система безопасности.

Источник:
"Декстер", s07e08, 24'.

Автор:
Юлия Кощеева (Коломна), Виктор Плотников (Великий Новгород)

Вопрос 17:
Еще раз напомним формулу стишка-порошка: 9+8+9+2, вторая и четвертая
строки рифмуются. Закончите порошок:
   вошел король и всё затихло
   придворные упали ниц
   и только шарканье по полу
   ...

Ответ:
ресниц

Источник:
https://vk.com/sandalporoshki?w=wall-31481258_93111

Автор:
Олег Евстафьев (Ступино)

Вопрос 18:
Управляя внутренними двигателями старых дисководов, можно получать звуки
разной частоты. Пользователь YouTube собрал в одном корпусе несколько
дисководов и заставил их играть разные произведения мировой музыкальной
культуры. Видеоролик об этом он назвал "Призрак оперы". Какие три буквы
мы пропустили в этом вопросе?

Ответ:
ф, л, п.

Комментарий:
"Призрак флопперы" ("Phantom of the Floppera").

Источник:
http://www.youtube.com/watch?v=dmoDLyiQYKw

Автор:
Юлия Кощеева (Коломна), Виктор Плотников (Великий Новгород)

Вопрос 19:
В фильме 1972 года его персонаж на самом деле не болен, а только
притворяется больным. В фильме 1981 года его персонажа действительно
потрепало. В обоих случаях за ним ухаживают женщины, правда одна
чересчур молчаливая, а другая чересчур болтливая. Назовите исполнителя
этих ролей.

Ответ:
Никита Михалков.

Комментарий:
Ротмистр Минский из "Станционного смотрителя" и Генри Баскервиль из
"Собаки Баскервилей". Дуня Вырина за весь фильм ни слова не говорит, а
Эльза Бэрримор "трындычит" так, что у похмельного Баскервиля голова
раскалывается.

Источник:
Указанные фильмы.

Автор:
Юлия Кощеева (Коломна), Виктор Плотников (Великий Новгород)

Вопрос 20:
В современном фильме Карлосон в исполнении Михаила Галустяна задает
Малышу вопрос: "А что такое "не кипишуй"?". В ответ он слышит три слова.
Напишите эти три слова.

Ответ:
"Спокойствие, только спокойствие".

Источник:
"Тот еще Карлосон!", 11'30".

Автор:
Юлия Кощеева (Коломна), Виктор Плотников (Великий Новгород)

Вопрос 21:
Надеемся, вы еще не забыли, что такое стишок-порошок. Персонаж одного
стишка-порошка утонул в северной русской реке, так и не закончив свою
знаменитую фразу. Назовите фамилию этого персонажа.

Ответ:
Станиславский.

Зачет:
Алексеев.

Комментарий:
   неугомонный станиславский
   тонув в стремительной неве
   закончил жизнь на полуфразе
   не ве

Источник:
https://vk.com/perawki?w=wall-28122932_47968

Автор:
Олег Евстафьев (Ступино)

Вопрос 22:
В язык этого народа были введены несколько оригинальных слов. Например,
слово "беш-ло" означает "железная рыба". Назовите этот народ.

Ответ:
Навахо.

Комментарий:
Для обозначения военных терминов, которых не было в языке навахо, были
введены новые слова. "Беш-ло" - подводная лодка.

Источник:
http://ru.wikipedia.org/wiki/Шифровальщики_навахо

Автор:
Юлия Кощеева (Коломна), Виктор Плотников (Великий Новгород)

Вопрос 23:
Статья "Московского комсомольца" об успехах наших паралимпийцев в Сочи
призывает встать с дивана и начать заниматься спортом. В заголовок
статьи был вынесен вопрос, который с 1992 года некоторые телезрители
еженедельно задают с экрана телевизора другим телезрителям.
Воспроизведите этот вопрос из трех слов.

Ответ:
"А вам слабо?".

Комментарий:
Речь идет о рубрике в передаче "Сам себе режиссер".

Источник:
   1. http://www.mk.ru/daily/newspaper/2014/3/18/
   2. http://ru.wikipedia.org/wiki/Сам_себе_режиссёр

Автор:
Борис Скоморохов (Ступино)

Вопрос 24:
[Ведущему: выделить голосом слово "ВЕРНОЕ".]
   Внимание, в вопросе есть одна замена.
   Согласно популярной интернет-шутке, алкаша, который каждый день по
несколько часов ждет открытия винно-водочного магазина, продавцы ласково
прозвали "Бушарик". На самом деле его прозвали по-другому. Поняв, что мы
заменили в этом вопросе, напишите ВЕРНОЕ прозвище.

Ответ:
Бухатико.

Комментарий:
Одну кличку собаки - Шарик - заменили на другую - Хатико.

Источник:
http://www.anekdot.ru/id/706891/

Автор:
Олег Евстафьев (Ступино)

Тур:
26 тур. "Hookah Project" (Астрахань - Истра - Москва)

Инфо:
Команда благодарит за ценные замечания Алексея Хмелькова, Александра
Грачёва, Анатолия Полетаева, Веронику Лапину, Анастасию Вилкову и Сергея
Бакшаева.

Вопрос 1:
ИКСОМ в карьере Михаила Ботвинника стало школьное соревнование 1923
года, на котором, возможно, был объявлен его ИГРЕК. ИКС и ИГРЕК
различаются одной буквой. Назовите ИГРЕК двумя словами.

Ответ:
Первый шах.

Комментарий:
Первым шагом Ботвинника к шахматной короне был обычный школьный матч.

Источник:
http://ru.wikipedia.org/wiki/Ботвинник,_Михаил_Моисеевич

Автор:
Михаил Сенкевич (Астрахань)

Вопрос 2:
Во вселенной Вархаммера есть навигаторы - существа, использующие
нечеловеческую энергию, чтобы видеть и указывать путь кораблям, и
поэтому имеющие ЕГО. Героя Кинга излечивают от НЕГО, делая похожим на
Гарри Поттера. Назовите ЕГО.

Ответ:
Третий глаз.

Комментарий:
Наука на грани эзотерики. Мальчику в книге "Темная половина" удаляют
третий глаз, в результате чего у него появляется шрам на лбу.

Источник:
   1. http://ru.ffg.wikia.com/wiki/Навигатор
   2. С. Кинг. Темная половина.

Автор:
Михаил Сенкевич (Астрахань)

Вопрос 3:
В полусказке Феликса Кривина, когда зашел спор о прекрасном среди
растений, Колючка одобрила ЕЕ остротУ и проникновение до самых глубин,
но осудила за то, что ОНА представляет всё ТАК. Какие три слова мы
заменили на "ТАК"?

Ответ:
В розовом свете.

Зачет:
Сквозь розовые очки.

Комментарий:
Колючка критиковала розу.

Источник:
Ф. Кривин. Полусказки. http://www.flibusta.net/b/170551/read

Автор:
Вероника Лапина (Астрахань)

Вопрос 4:
В фильме "Сквозь горизонт" есть сцена: персонаж рисует на листе линию,
складывает лист пополам и протыкает его ручкой. Назовите фильм, в
котором эта сцена была полностью повторена.

Ответ:
"Интерстеллар".

Комментарий:
Персонаж показывает сущность кротовой норы и мгновенных перемещений в
пространстве.

Источник:
http://www.kinopoisk.ru/film/258687/

Автор:
Михаил Сенкевич (Астрахань)

Вопрос 5:
Герой Маяковского, бывший рабочий, бывший партиец, а ныне жених,
требовал, чтобы была ОНА и никаких богов. Другая ОНА состоялась в 299
году. Назовите ЕЕ двумя словами.

Ответ:
Красная свадьба.

Комментарий:
В 299 году по летоисчислению Семи Королевств.

Источник:
   1. В. Маяковский. Клоп.
http://www.feb-web.ru/feb/mayakovsky/texts/ms0/msb/msb-215-.htm
   2. Джордж Мартин. Буря Мечей.

Автор:
Вероника Лапина (Астрахань)

Вопрос 6:
Персонажи романа Бориса Виана "Пена дней" во время венчания едят
божественное мороженое и пьют газированную воду с сиропом. Какое слово в
предыдущем предложении мы пропустили?

Ответ:
Святую.

Комментарий:
Венчание происходит в церкви, где даже газированная вода с сиропом -
святая, а мороженое - божественное.

Источник:
Б. Виан. Пена дней. http://www.flibusta.net/b/133416/read

Автор:
Вероника Лапина (Астрахань)

Вопрос 7:
Евгения Ивановна, заглавная героиня повести Леонида Леонова, -
эмигрантка, которая бежала из царской России и вышла замуж за
англичанина. Запишите название повести, как это сделал автор.

Ответ:
"Evgenia Ivanovna".

Зачет:
Транслитом с незначительными ошибками.

Комментарий:
Такой заголовок подчеркивает, что героиня во многом и сама стала
англичанкой.

Источник:
Л. Леонов. Evgenia Ivanovna. http://www.flibusta.net/b/198326/read

Автор:
Вероника Лапина (Астрахань)

Вопрос 8:
Сразу после знакомства со своим спасителем заглавный герой детского
фильма попросил его назвать самое нежное из имен его отца. Назовите имя
отца заглавного героя.

Ответ:
Хоттаб.

Комментарий:
Потому он и называл Вольку ибн Алеша, а не ибн Алексей.

Источник:
Л. Лагин. Старик Хоттабыч. http://www.flibusta.net/b/266897/read

Автор:
Вероника Лапина (Астрахань)

Вопрос 9:
Саре Бернар приписывают следующее утверждение: "Жизнь похожа на лежание
на полке в бане.... Тебе мылят голову и ДЕЛАЮТ ЭТО". ДЕЛАЮТ ЭТО
работники некоторых музеев. Какие два слова мы заменили на "ДЕЛАТЬ ЭТО"?

Ответ:
Перебирать косточки.

Зачет:
Перебирать кости; перемывать косточки; перемывать кости.

Источник:
А.П. Чехов. Философские определения жизни.
http://www.feb-web.ru/feb/chekhov/texts/sp0/sp1/sp1-470-.htm

Автор:
Вероника Лапина (Астрахань)

Вопрос 10:
Барселонский клуб комедии установил на спинке каждого кресла планшет.
Эта технология позволила установить необычный, но справедливый способ
оплаты: теперь зритель платит по 30 центов за каждую АЛЬФУ. Какое слово
мы заменили АЛЬФОЙ?

Ответ:
Улыбка.

Комментарий:
Планшет считывает улыбку на лице зрителя, а после выступления улыбки
подсчитываются, и их количество определяет, сколько тот должен
заплатить. Максимальная сумма, которую можно потратить, - 24 евро, после
этого все улыбки бесплатны.

Источник:
http://www.theguardian.com/stage/2014/oct/14/standup-comedy-pay-per-laugh-charge-barcelona

Автор:
Илья Иванов (Пермь)

Вопрос 11:
В повести Теда Чана описывается строительство Вавилонской башни. При
этом не было недостатка в кирпичах, зато каждый строитель боялся
потерять то, что в советские времена было названием детского журнала о
творчестве и рукоделии. Что именно?

Ответ:
Мастерок.

Зачет:
Мастерки.

Комментарий:
Вавилонская башня была очень высокой. Утеря кирпича для каменщика была
пустяком, а вот без мастерка он на три-четыре месяца лишался работы,
пока ему наверх не доставляли новый.

Источник:
   1. Тед Чан. Вавилонская башня. http://www.flibusta.net/b/96583/read
   2. http://ru.wikipedia.org/wiki/Кельма

Автор:
Виктор Мялов (Днепропетровск)

Вопрос 12:
Некоторые муравьи рода Adetomyrma [адетомИрма] питаются гемолИмфой своих
личинок. За это их прозвали муравьями-акУлами. Одно из слов в предыдущем
предложении мы немного изменили. Напишите его в первоначальном виде.

Ответ:
Муравьями-дрАкулами.

Зачет:
ДрАкулами.

Комментарий:
Гемолимфа у насекомых является аналогом крови у животных.

Источник:
http://en.wikipedia.org/wiki/Adetomyrma

Автор:
Илья Иванов (Пермь)


FreeBSD-CVSweb <freebsd-cvsweb@FreeBSD.org>